Download as pdf or txt
Download as pdf or txt
You are on page 1of 150

NAME OF THE REGION:-

CHANDIGARH

NAME OF THE CLASS:-

IX

SUBJECT:- SOCIAL SCIENCE


अ याय : ांसीसी ां त
CHAPTER: THE FRENCH REVOLUTION
1. When did French Revolution occur ?
a) 14th July 1780 b) 14th July 1789
c) 15th August 1789 d) 14th July 1796
ांसीसी ां त कब ई ?
a) 14 जुलाई 1780 b) 14 जुलाई 1789
c) 15 अग त 1789 d) 14 जुलाई 1796
2. The National Assembly framed a Constitution in 1791 to limit the powers of:
a) Monarch b) Wealthy man
c) businessmen d) Press
रा ीय सभा ने 1791 म कसक श को काम करने के लए सं भधान का नमाण कया ?
a) स ाट b) अमीर आदमी
c) ापारी d) ेस
3. What was tithes ?
a) A tax levied by the Church b) A tax levied by the king
c) Unit of currency in France d) None of these
ताईद या था ?
a) चच ारा लगाया गया कर b) राजा ारा लगाया गया कर
c) ांस म मु ा क इकाई d) इनम से कोई नह

4. Who wrote an influential pamphlet ‘What is the third Estate’ ?


a) Mirabeau b) Jean Paul Marat
c) Abbe Sieyes d) Olympe de gouges

एक भावशाली लेख “तीसरा ए टे ट या है” कसने लखा ?


a) मरा यो b) जीन पॉल मारट
c) अ बे सए d) ओ ल पे द गाउज
5. Which period in the History of France is known as “Reign of Terror” ?
a) 1793 to 1798 b) 1793 to 1799
c) 1793 to 1794 d) 1794 to 1798
ांस के इ तहास म कस काल को आतंक का शासन कहा जाता है ?
a) 1793 से 1798 b) 1793 से 1799
c) 1793 से 1794 d) 1794 से 1798
6. Which group of the people did not join the Jacobin club ?
a) Men with property b) Artisans
c) Shopkeepers d) Daily wage workers
लोग का कौन-सा समूह जैको बन क ब म शा मल नह आ ?
a) संप वाले पु ष b) कारीगर
c) कानदार d) दै नक वेतन भोगी करमचारी
7. Who wrote “Two Treaties of Government” ?
a) Voltaire b) Rousseau
c) George Danton d) John Locke
“टू टाइजेज़ ऑफ गवनमट” कसने लखा ?
a) वो टाइर b) सो
c) जॉज दांत d) जॉन लॉक

8. French women demanded the right :


a) to vote b) to be elected to the assembly
c) to hold official office d) All of the above

ांसीसी म हला ने कन अ धकार क मांग क ?


a) मतदान करना b) सभा के लए चुने जाना
c) आ धका रक पद धारण करना d) सभी
9. A triangular slave trade took place between Europe, America and :
a) Asia b) Africa
c) Australia d) None of the above
यूरोप, अमे रका और----------------------- के बीच कोणीय दास ापार आ |
a) ए शया b) अ का
c) ऑ े लया d) इनम से कोई नह
10. Whose rule in the history of France is known as Reign of Terror ?
a) Maximilian Robespierre b) John Locke
c) Louis XVI d) George Danton
ांस के इ तहास म कसके काल को आतंक का शासन कहा जाता है ?
a) मै स म लयन रोबेसपयेर b) जॉन लॉक
c) लुइस 16 c) जॉज दांत
11. Upon becoming free, the slave wore :
a) Blue cap b) White cap
c) Red cap d) Green cap
आज़ाद होने पर दास या पहनते थे ?
a) नीली टोपी b) सफे द टोपी
c) लाल टोपी d) हरी टोपी

12. Who were not considered ‘passive citizens’ ?


a) Women b) Children
c) Non-propertied men d) Wealthy people

कसको न य नाग रक नह माना जाता ?


a) औरत को b) ब को
c) संप र हत पु ष d) अमीर लोग
13. In the context of France, what was ‘Taille’ ?
a) An indirect tax b) Direct tax
c) Custom duties d) None of these
ांस के संदभ म टाइल या है ?
a) एक अ य कर b) य कर
c) सीमा शु क d) इनम से कोई नह
14. Who among the following enjoyed certain privileges by birth in France during the late
th
18 century ?
a) Big businessmen and merchants b) Peasants and artisans
c) Clergy and Nobles d) Court officials and lawyers
न मन ल खत म से कसे 18व शता द म ांस म ज म के आधार पर कु छ वशेषा धकार ा त थे ?
a) बड़े वसायी और ापारी b) कसान और कलाकार
c) पादरी और रईस d) अदालत के अ धकारी और वक ल
15. Which of the following statements is untrue about the Third Estate ?
a) Within the Third Estate some were rich and some were poor
b) The Third Estate was made of the poor only
c) Peasants were obliged to serve in the army or bulid roads
d) Richer members of the Third Estate owned lands
तीसरे ए टे ट के बारे म न मन ल खत म से कौन सा बयान सही नह है ?
a) तीसरे ए टे ट म कु छ लोग अमीर थे और कु छ गरीब
b) तीसरा ए टे ट सफ गरीब से बना था
c) कसान सेवा करने और सड़क बनाने के लए बा य थे
d) तीसरे ए टे ट के अमीर लोग के पास ज़मीन थ |

16. Who wrote “The Spirit of Laws” ?


a) John Locke b) Montesquieu
c) Voltaire d) Rousseau
द रट ऑफ लॉज़ कसक रचना है ?
a) जॉन लॉक b) म टे स यू
c) वो टाइर d) सो
17. A guillotine was …………………………..
a) A device consisting of two poles and a blade with which a person was beheaded
b) A fine sword with which heads were cut off
c) A special noose to hang people
d) None of the above
गलो टन या था ?
a) एक उपकरण जसके दो डंडे और एक लेड होता है जसके साथ एक का सर काट दया जाता है
b) एक तलवार जससे सर काटे जाते ह
c) लोग को फांसी दे ने का खास फं दा
d) इनम से कोई नह
18. The word livres stands for:
a) Tax levied by the Church b) Direct tax
c) Unit of currency in France d) None of these
लवरेस श द का या अथ है ?
a) चच ारा लगाया गया कर b) य कर
c) ांस म मु ा क इकाई d) इनम से कोई नह

19. What was the ‘Subsistence Crisis’ ?


a) Subsidy in food grains
b) An extreme situation endangering the basic means of livelihood
c) Large scale production of foodgrains
d) None of these
नवाह संकट या था ?
a) खा ान म स सडी
b) आजी वका के बु नयाद साधन को खतरे म डालने वाली चरम त
c) खा ानो का बड़े तर पर उ पादन
d) इनम से कोई नह
20. What was Estate General ?
a) Post of Army General b) Head of all landed property
c) A political body d) Advisor of the king
ए टे ट जनरल या था ?
a) आम जनरल का पद b) सभी भू-संप का मु खया
c) एक राजनी तक सं ा d) राजा का सलाहकार
21. Where was Napoleon finally defeated in 1815 ?
a) Waterloo b) Paris
c) Tokyo d) Israel
नेपो लयन को आ खर म कहाँ परा जत कया गया था ?
a) वाटे रलू b) पे रस
c) टो यो d) इ ाइल
22. Who wrote ‘ The Social Contract’ ?
a) John Locke b) Voltaire
c) Jacques Rousseau d) George Danton

द सोशल का ै ट कसक रचना है ?


a) जॉन लॉक b) वो टाइर
c) जैकस सो d) जॉज दांत
23. The term ‘Old Regime’ was usually used to describe:
a) France before 1000 B.C. b) Society of France after 1789
c) Society and institutions of France before 1789 d) None of these
श द पुराना शासन कसका वणन करने के लए योग कया जाता है ?
a) 1000 ईसा पूव से पहले ांस b) 1789 के बाद का ांस का समाज
c) 1789 के पहले ांस का समाज और सं ाएं d) इनम से कोई नह
24. Which of these provisions were passed by the assembly on the night of 4 August,
1789 ?
a) Abolition of feudal system of obligations
b) Clergy had to give up its privileges
c) Tithes were abolished
d) All the above
इनम से कौन-स ावधान वधान सभा आरा 4 अग त 1789 को पा रत कया गया था ?
a) सामंती वसथा का उ मूलन
b) पाद रय को अपने वशेषा धकार यागने पड़े
c) ताईद समा त कर दया गया
d) उपरो सभी
25. Who said: The task of representing the people has been given to the rich ?
a) Mirabeau b) Jean Paul Marat
c) Rousseau d) George Denton
कसने कहा : लोग का त न धतव करने का काम अमीर को दया गया है ?
a) मरा यो b) जीन पॉल मारट
c) सो d) जॉज दांत
26. Who led the representatives of the Third Estate in Versailles on 20th June ?
a) Mirabeau b) Abbe Sieyes
c) Louis XVI d) Both a and b

20 जून को वसाइ के तीसरे ए टे ट के तनध क अगवाई कसने क ?


a) मरा यो b) अ बे सए
c) लुइस 16 d) पहला और सरा दोन
27. Which of the following is true about Bastille Storming ?
a) It was a fortress prison in France
b) It represent despotic powers
c) French common man hated Bastille
d) All the above
बास तल टोर मग के बारे म न न ल खत म से कौन-सा सही है ?
a) यह ांस म एक कले क जेल थी
b) यह नरंकुश श य का त न ध व करता है
c) ांसीसी आम आदमी बास तल से नफरत करता था
d) उपरो सभी
28. In the war against Prussia and Austria, the army sang which patriotic song ?
a) Liberty b) Marseillaise
c) Viva France d) None of these
शया और आ या के खलाफ यु म सेना ने कौन-सा दे श भ गीत गया ?
a) लबट b) मरसेलाइस
c) ववा ांस d) इनम से कोई नह
29. The new Constitution made France a :
a) Constitutional Monarchy b) Communist state
c) Fully Democratic state d) None of these
नए सं भधान ने ांस को या बनाया ?
a) संवैधा नक राजत b) क यु न ट रा य
c) पूरी तरह लोकता क रा य d) इनम से कोई नह
30. Name the most successful political club which became a rallying point for people
who wished to continue the Revolution in France.
a) Jacobin b) Arthur
c) Mirabeau d) Dauphine
सबसे सफल राजनी तक लब का नाम बताइए जो ांस म ां त को जारी रखने क इ ा रखने वाले लोग के
लए एक रैली ल बन गया |
a) जैकोबीन b) आथर
c) मरा यो d) डाउफाइन
31. The Assembly elected in 1792 was called :
a) Congress b) Council
c) Convention d) Congregation
1792 म चुनी गई सभा को या कहा गया ?
a) काँ ेस b) काउ सल
c) क वे न d) मंडली
32. On what charges was Louis XVI sentenced to death ?
a) Cruelty b) Treason
c) Incapability d) Misuse of powers
कस दोष के लए लुइस 16 को मौत क सजा द गई ?
a) ू रता b) दे श ोह
c) अयो यता d) स ा क रवत
33. Who among the following Indian individuals responded to the ideas coming from
Revolutionary France ?
a) Bhagat Singh b) Rammohan Rai
c) Mahatma Gandhi c) None of these
न न ल खत भारतीय म से कसने ां तकारी ांस से आने वाले वचार के त त या क ?
a) भगत सह b) राममोहन राय
c) महा मा गांधी d) इनम से कोई नह
34. Who among the following reintroduced slavery in France after it was abolished by
Jacobin regime ?
a) Louis XVI b) Robespierre
c) Napoleon d) Marat
न न ल खत म से कसने जैको बन शासन आरा ांस म दास था को समा त करने के बाद फर से शु कया ?
a) लुईस 16 b) रोबेसपीयर
c) नेपो लयन d) मारट
35. King Louis XVI belonged to which dynasty of kings ?
a) Hapsburg b) Bourbon
c) Romanov d) Windsor
रा य लुइस 16 राजा के कस वंश के थे ?
a) हैपसबुग b) बौरब
c) रोमनोव d) वदसोर
36. Austrian Princess Maria Antoniette was the queen of which of the following rulers ?
a) Louis XIII b) Louis XIV
c) Louis XV d) Louis XVI
आ या क राजकु मारी मा रया ए टो नयेत न न ल खत म से कस शासक क रानी थी ?
a) लुइस 13 b) लुइस 14
c) लुइस 15 d) लुइस 16

ASSERTION AND REASONING BASED QUESTIONS


For question numbers 37 to 40 two statements are given – one labelled Assertion (A)
and the other labelled Reason ( R). Select the correct answer to these questions from
the codes (a), (b), (c) and (d) as given below.
a) Both A and R are true and R is the correct explanation of the assertion
b) Both A and R are true and R is not the correct explanation of the assertion
c) A is true but R is false
d) A is False but R is true
अ भकथन और तक आधा रत
सं या 37 से 40 तक के लए दो कथन दए गए ह – एक को अ भकथन और सरे को कारण लेबल कया
गया है | नीचे दए गए कू ट (a), (b), (c) और (d) से इन के सही उ र का चयन कर |
a) A और R दोन कथन सही है और R कथन क सही ा या करता है |
b) A और R दोन कथन सही है और R कथन क सही ा या नह करता है |
c) A सही है ले कन R गलत है |
d) A गलत है ले कन R सही है |
37. Assertion: Upon his accession Louis XVI found an empty treasure
Reason : Long years od famine had drained the financial resources
अ भकथन : लुइस 16 को एक खाली खजाना मल था |
कारण : लंबे वष के अकाल ने व ीय संसाधन को ख म कर दया था |
38. Assertion : The members of the two estates enjoyed certain privileges by birth
Reason : They were respected by the society
अ भकथन : दो ए टे ट के सद य को ज म से वशेषा धकार ा त थे |
कारण : समाज आरा उनका स मान कया जाता था |
39. Assertion : The representatives of the First Estate viewed themselves as
spokesmen of the whole French nation
Reason : They enjoyed certain privileges by birth.
अ भकथन : पहले ए टे ट के त नधी खुद को पूरे ांसीसी रा के व ा के प म दे खते थे |
कारण : उ ह ने ज म से कु वशेषा धकार का आनंद लया |
40. Assertion : The period from 1793 to 1794 is referred aas the Reign of Terror.
Reason : The members of the Jacobin club belonged to the less prosperous society
अ भकथन : 1793 से 1794 तक क अव ध को आंतक के शासन के प म जाना जाता है |
कारण : जैको बन लब के सद य काम समृ समाज के थे |
SOURCE BASED QUESTIONS
Read the extract carefully and answer the following questions or choose the correct
option:
EXTRACT – 1
On 5 May 1789, Louis XVI called together an assembly of the Estates General to pass
proposals for new taxes. A resplendent hall in Versailles was prepared to host the
delegates. The first and second estates sent 300 representatives each, who were
seated in rows facing each other on two sides, while the 600 members of the Third
Estate had to stand at the back. The third estate was represented by its more
prosperous and educated members. Peasants, artisans and women were denied to
entry to the assembly. However their grievances and demands were listed in some
40000 letters which the representatives have brought with them.
उ रण को यानपूवक पढ़े और न न ल खत के सही वक प चुने |
5 मई 1789 को लुइस 16 ने नए कर के ताव को पा रत करने के लए ए टे ट जनरल क सभा को बुलाया |
त न धय क मेजबानी के लए वसाय म एक शानदार हॉल तैयार कया गया था | पहले और सरे ए टे ट ने 300
त न ध को भेज जो क दो तरफ एक सरे के सामने पं य म बैठे थे, जब क तीसरे ए टे ट के 300 सद य
को सबसे पीछे खड़ा होना पड़ा | तीसरे ए टे ट का त न ध व इसके अ धक समृ और श त सद य ारा कया
गया था | कसान , कारीगर और म हला को सभा म वेश करने से मना कर दया गया था | हालाँ क उनक
शकायत और माँग को लगभग 40000 प म सूचीब कर दया गया था जो क त न ध अपने साथ लाए थे |
41. When did Louis XVI call the assembly of the Estates General ?
a) On 5 April 1789 b) On 5 May 1789
c) On 15 May 1789 d) On 5 May 1798

लुइस 16 ने ए टे ट जनरल क सभा कब बुलाई ?


a) 5 अ ैल 1789 b) 5 मई 1789
c) 15 मई 1789 d) 5 मई 1798
42. Which Estate sent 600 members to the assembly ?
a) The First Estate b) The Second Estate
c) The Third Estate d) None of these
कस ए टे ट ने सभा म 600 त न ध भेजे ?
a) पहले ए टे ट ने b) सरे ए टे ट ने
c) तीसरे ए टे ट ने d) इनम से कोई नह

43. Who were denied to the entry to the assembly ?


a) Members of the First Estate
b) Members of the Second Estate
c) More prosperous and educated members of the Third Estate
d) Peasants, artisans and women
सभा म वेश करने से कसको मना कया गया ?
a) पहले ए टे ट के सद य को
b) सरे ए टे ट के सद य को
c) तीसरे ए टे ट के अ धक समृ और श त सद य को
d) कसान, कारीगर और औरत को
44. How many representatives were sent by the first estate ?
a) 200 members b) 300 members
c) 400 members d) 600 members
पहले ए टे ट के ारा कतने सद य को भेजा गया ?
a) 200 सद य b) 300 सद य
c) 400 सद य d) 600 सद य

45. The Third Estate was represented by whom ?


a) More prosperous and educated members
b) Peasants, artisans and women
c) Nobles
d) None of these
तीसरे ए टे ट का त न ध व कसके ारा कया गया था ?
a) अ धक समृ और श त सद य
b) कसान, कारीगर और म हलाएं
c) रईस
d) इनम से कोई

Extract – 2

Louis XVI had signed the Constitution , he entered into secret negotiations with the king
of Prussia. The National Assembly voted in April 1792 to declare war against Prussia
and Austria. Thousands of volunteers thronged from the provinces to join the army.
They saw this as a war of the people against kings and aristocracies all over Europe.
Among the patriotic songs they sang was the Marseillaise, composed by the poet Roget
de L’Isle ( It was sung for the first time by volunteers from Marseilles as they marched
into Paris and so got its name.) The Marseillaise is now the national anthem of France.
लुइस 16 ने सं भधान पर ह ता र कए थे, उसने शया के रा य के साथ गु त वाता क | अ ैल 1792 म रा ीय
सभा ने शया और ऑ या के खलाफ यु क घोषणा करने के लए मतदान कया | हजार क सं या म
वयंसेवक सेना म शा मल होने के लए ांत से उमड़ पड़े | उ होन इसे पूरे यूरोप म राजा और अ भजात वग के
खलाफ लोग ने यु के प म दे खा | उनके ारा गाए गए दे शभ गीत म क व रोजेट डी लसले ारा र चत
मार सलेइस था| यह पहली बार मार सले के के वयंसेवक ारा गाया गया था य क उ होन पे रस म माच कया
था और इस लए उसका नाम मला | मार सलेइस अब ांस का रा गान है |
46. The National Assembly voted in April 1792 to declare war against……………..
a) Germany b) Prussia and Austria
c) England d) Denmark
अ ैल 1792 म रा ीय सभा ने कसके खलाफ यु क घोषणा करने के लए मतदान कया ?
a) जमनी b) शया और ऑ या
c) इं लड d) डेनमाक
47. Who composed the song, the Marseillaise ?
a) Rousseau b) George Denton
c) Roget de L’Isle d) Voltaire
मार सलेइस गीत क रचना कसने क ?
a) सो b) जॉज डट
c) रोजेट द लसले d) वो टाइर
48. Who had signed the Constitution ?
a) Louis XIV b) Louis XV
c) Louis XVI d) None of these
सं भधान पर कसने ह ता र कए ?
a) लुइस 14 b) लुइस 15
c) लुइस 16 d) इनम से कोई नह
49. The Marseillaise is the national anthem of……………………..
a) France b) Germany
c) Italy d) Prussia
मार सलेइस कहाँ का रा गान है ?
a) ांस b) जमनी
c) इटली d) शया
50. Thousands of volunteers thronged from the -------------------- to join the army.
a) cities b) villages
c) provinces d) None of these
हजार क सं या म वयंसेवक सेना म शा मल होने के लए ---------------- से उमड़ पड़े |
a) शहर b) गाँव
c) ांत d) इनम से कोई नह

ANSWER KEY
1. b 2. a 3. a 4. c 5. c 6.a 7.d 8.d 9.b 10,a
11.c 12.d 13.b 14.c 15.b 16.b 17.a 18.c 19.b 20.c
21.a 22.c 23.c 24.d 25.b 26.d 27.d 28.b 29.a 30.a
31.c 32.b 33.b 34.c 35.b 36.d 37.a 38.c 39.d 40.b
41.b 42.c 43.d 44.b 45.a 46.b 47.c 48.c 49.a 50.c

LESSON:- INDIA SIZE AND LOCATION

Question 1. The Tropic of Cancer does not pass through


(a) Rajasthan
(b) Orissa (Now Odisha)
(c) Chhattisgarh
(d) Tripura
1 कक रेखा नह गुजरती है
(ए) राज ान
(बी) उड़ीसा (अब ओ डशा)
(सी) छ ीसगढ़
(डी) पुरा

Question 2. The eastern most longitude of India is


(a) 97°25’E
(b) 68°7′ E
(c) 77°6’E
(d) 82°32’E
2 भारत का सबसे पूव दे शांतर है
(ए) 97O25ई
(बी) 68O7′ ई
(सी) 77O6'ई
(डी) 82O32'ई

Question 3. Uttarakhand, Uttar Pradesh, Bihar, West Bengal and Sikkim have common
frontiers with
(a) China
(b) Bhutan
(c) Nepal
(d) Myanmar
3 उ राखंड, उ र दे श, बहार, प म बंगाल और स कम क सीमाएँ समान ह
(ए) चीन
(बी) भूटान
(सी) नेपाल
(डी) यांमार

Question 4. If you intend to visit Kavarati during your summer vacations, which one of
the following Union Territories of India you will be going to?
(a) Puducherry
(b) Lakshadweep
(c) Andaman & Nicobar
(d) Diu and Daman
4. य द आप अपनी गम क छु य के दौरान कावारती जाने का इरादा रखते ह, तो आप भारत के
न न ल खत म से कस क शा सत दे श म जाएंग?े
(ए) पुडुचरे ी
(बी) ल प
(सी) अंडमान और नकोबार
(डी) द व और दमन

Question 5. My friend hails from a country which does not share land boundary with
India. Identify the country.
(a) Bhutan
(b) Tajikistan
(c) Bangladesh
(d) Nepal
5. मेरा दो त एक ऐसे दे श से है जो भारत के साथ भू म सीमा साझा नह करता है। दे श को पहचानो।
(ए) भूटान
(बी) ता ज क तान
(सी) बां लादे श
(डी) नेपाल

Question 6. How many states and Union Territories does India have?
(a) India has 26 States and 7 Union Territories
(b) India has 25 States and 7 Union Territories
(c) India has 28 States and 7 Union Territories
(d) India has 28 States and 6 Union Territories
6. भारत म कतने रा य और क शा सत दे श ह?
(ए) भारत म 26 रा य और 7 क शा सत दे श ह
(बी) भारत म 25 रा य और 7 क शा सत दे श ह
(सी) भारत म 28 रा य और 7 क शा सत दे श ह
(डी) भारत म 28 रा य और 6 क शा सत दे श ह

Question 7. Which line divides India into approximately two equal parts?
(a) Equator
(b) Tropic of Cancer
(c) Tropic of Capricorn
(d) None of these

7. कौन सी रेखा भारत को लगभग दो बराबर भाग म वभा जत करती है?


(ए) भूम य रेखा
(बी) कक रेखा
(सी) मकर रेखा
(डी) इनम से कोई नह

Question 8. What is the position of India in the world in respect of area?


(a) 8th position
(b) 7th position
(c) 6th position
(d) 2nd position
8. े फल क से भारत का व म कौन सा ान है?
(ए) 8 वां ान
(बी) 7 वां ान
(सी) 6 वां ान
(डी) सरा ान

Question 9. Which meridian is fixed as a Standard Meridian of India?


(a) 82½° E
(b) 84½° E
(c) 86° E
(d) 81° E
9. भारत के मानक या यो र के प म कौन-सा म या रेखा नयत है?
(ए) 82½ ड ी ई
(बी) 84½ ड ी ई
(सी) 86 ड ी ई
(डी) 81 ड ी ई
Question 10. The latitudinal extent of India lies between
(a) 7° 5′ N and 26° 5′ N
(b) 8° 4′ N and 37° 6′ N
(c) 12° 5′ N and 27° 5′ N
(d) 12° 5′ N and 37° 6′ N
10. भारत का अ ांशीय व तार कसके बीच त है?
(ए) 7 ड ी 5' एन और 26 ड ी 5' एन'
(बी) 8 ड ी 4' एन और 37 ड ी 6' एन'
(सी) 12 ड ी 5' एन और 27 ड ी 5' एन'
(डी) 12 ड ी 5' एन और 37 ड ी 6' एन'

Question 11. Tropic of Cancer passes through which of these states


(a) Orissa
(b) Tripura
(c) Bihar
(d) Punjab
11. कक रेखा इनम से कस रा य से होकर गुजरती है
(ए) उड़ीसा
(बी) पुरा
(सी) बहार
(डी) पंजाब

Question 12. Which of the following longitudes is selected as the Standard Meridian for
India?
(a) 68°7′E
(b) 82°30′E
(c) 97°25′E
(d) 23°30′N
12. न न ल खत म से कौन सा दे शांतर भारत के लए मानक मे र डयन के प म चुना गया है?
(ए) 68°7′ई
(बी) 82 ड ी 30′ई
(सी) 97°25′ई
(डी) 23 ड ी 30′N

Question 13. Which of the following is the oldest route of contact between India and
other countries of the world?
(a) Ocean routes
(b) Maritime contact
(c) Land routes
(d) Air routes

13. न न ल खत म से कौन सा भारत और नया के अ य दे श के बीच संपक का सबसे पुराना माग है?
(ए) महासागर माग
(बी) समु संपक
(सी) भू म माग
(डी) हवाई माग

Question 14. India achieved multi-faceted socio-economic progress during which of the
following periods?
(a) Since ancient times
(b) During medieval period
(c) In the 21st century
(d) During the last five decades
14. भारत ने न न ल खत म से कस अव ध के दौरान ब आयामी सामा जक-आ थक ग त हा सल क ?
(ए) ाचीन काल से
(बी) म ययुगीन काल के दौरान
(सी) २१व सद म
(डी) पछले पांच दशक के दौरान

Question 15. Which of the following is the longitudinal extent of India?


(a) 8°4′N and 37°6′N
(b) 68°7′N and 97°25′E
(c) 68°7′E and 97°25′W
(d) 8°4′E and 37°6′E
15. न न ल खत म से कौन भारत का दे शांतरीय व तार है?
(ए) 8 ड ी 4'एन और 37 ड ी 6' एन'
(बी) 68°7′N और 97°25′E
(सी) 68°7′पूव और 97°25′W
(डी) 8 ड ी 4'ई और 37 ड ी 6'ई'
Question 16. The eastern-most longitude of India is
(a) 97°25′E
(b) 68°7′E
(c) 77°6′E
(D) 82°32′E
Question 17. Which latitude passes through the southern-most point of India’s mainland?
(a) 8°4′N
(b) 37°6′N
(c) 8°4′S
(d) 82°30′E
17. कौन सा अ ांश भारत क मु य भू म के सबसे द णी ब से होकर गुजरता है?
(ए) 8°4′N
(बी) 37°6′N
(सी) 8 ड ी 4'एस'
(डी) 82 ड ी 30′

Question 18. Which of the following groups of islands belonging to the Indian territory
lies in the Arabian Sea?
(a) Andaman and Nicobar Islands
(b) Sri Lanka
(c) Lakshdweep
(d) Maldives
18. भारतीय े से संबं धत प का न न ल खत म से कौन सा समूह अरब सागर म त है?
(ए) अंडमान और नकोबार प समूह
(बी) ीलंका
(सी) ल प
(डी) मालद व
Question 19. What is India’s size with respect to other countries of the world?
(a) First
(b) Third
(c) Fourth
(d) Seventh
19. व के अ य दे श क तुलना म भारत का आकार कतना है?
(पहला
(बी) तीसरा
(सी) चौथा
(डी) सातवां
Question 20. Which geographical feature bounds India’s mainland south of 22°N
latitude?
(a) Young Fold Mountains
(b) Sandy Desert
(c) Lava Plateaus
(d) Seas and Ocean
20. कौन सी भौगो लक वशेषता भारत क मु य भू म को 22°N अ ांश के द ण म बांधती है?
(ए) यंग फो पवत
(बी) सडी रे ग तान
(सी) लावा पठार
(डी) समु और महासागर

Question 21. If the local time at Dwarka (69°01′E) in Gujarat to the west of India is 6 am,
what will be the local time at Dibrugarh (94°58′E approximately 95°), in Assam, in the
east?
(a) 4.16 am
(b) 6 am
(c) 7.44 am
(d) 7.44 pm
21. य द भारत के प म म गुजरात म ारका (69°01′E) का ानीय समय सुबह 6 बजे है, तो पूव म असम
म ड ूगढ़ (94°58′E लगभग 95°) का ानीय समय या होगा?
(ए) 4.16 पूवा
(बी) सुबह 6 बजे
(सी) 7.44 पूवा
(डी) 7.44 अपरा
Question 22. The Standard Meridian of India, 82°30′E passes through which of the
following places?
(a) Kanniyakumari in Tamil Nadu
(b) Walong in Arunachal Pradesh
(c) Kachchh in Gujarat
(d) Mirzapur in Uttar Pradesh
22. भारत का मानक मे र डयन, 82°30′E न न ल खत म से कस ान से होकर गुजरता है?
(ए) त मलनाडु म क याकु मारी
(बी) अ णाचल दे श म वाल ग
(सी) गुजरात म क
(डी) उ र दे श म मजापुर
Question 23. Which of the following has reduced India’s distance from Europe by 7000
km?
(a) Suez Canal
(b) Panama Canal
(c) Indira Gandhi Canal
(d) Buckingham Canal
23. न न ल खत म से कसने यूरोप से भारत क री को 7000 कमी कम कर दया है?
(ए) वेज नहर
(बी) पनामा नहर
(सी) इं दरा गांधी नहर
(डी) ब कघम नहर

Question 24. The four states which are situated along the Himalayas are:
(a) Jammu and Kashmir, Himachal Pradesh, Uttarakhand, Arunachal Pradesh
(b) Gujarat, Maharashtra, Haryana, Rajasthan
(c) Gujarat, Chhattisgarh, Bihar, Jharkhand
(d) Bihar, West Bengal, Orissa, Uttar Pradesh 24 हमालय के
कनारे त चार रा य ह:
(ए) ज मू और क मीर, हमाचल दे श, उ राखंड, अ णाचल दे श
(बी) गुजरात, महारा , ह रयाणा, राज ान
(सी) गुजरात, छ ीसगढ़, बहार, झारखंड
(डी) बहार, प म बंगाल, उड़ीसा, उ र दे श
Question 25. A narrow channel of sea separating two land-masses is called:
(a) Gulf
(b) Strait
(c) Isthmus
(d) Bay
25.दो भू-भाग को अलग करने वाली समु क एक संकरी नहर कहलाती है:
(ए) खाड़ी
(बी) जलडम म य
(सी) इ तमुस
(डी) बे
Question 26. The neighbouring countries that share their boundaries with India are :
(a) Pakistan and Afghanistan
(b) Myanmar and Bangladesh
(c) China and Nepal
(d) All the above
26. भारत के साथ अपनी सीमा साझा करने वाले पड़ोसी दे श ह:
(ए) पा क तान और अफगा न तान
(बी) यांमार और बां लादे श
(सी) चीन और नेपाल
(डी) उपरो सभी

Question 27. India’s total area accounts for what percentage of the geographical area of
the world?
(a) 2.9%
(b) 3.2%
(c) 2.4%
(d) 4.2%
27.भारत का कु ल े फल व के भौगो लक े फल का कतना तशत है?
(ए) 2.9%
(बी) 3.2%
(सी) 2.4%
(डी) 4.2%

Question 28.Which country among the India’s neighbours is the smallest?


(a) Nepal
(b) Bhutan
(c) Sri Lanka
(d) Bangladesh
28.भारत के पड़ो सय म सबसे छोटा दे श कौन सा है?
(ए) नेपाल
(बी) भूटान
(सी) ीलंका
(डी) बां लादे श

Question 29. Which meridian is fixed as a Standard Meridian of India?


(a) 82½° E
(b) 84½° E
(c) 86° E
(d) 81° E
29. भारत क मानक या यो र के प म कौन-सी म या रेखा नयत है?
(ए) 82½ ड ी ई
(बी) 84½ ड ी ई
(सी) 86 ड ी ई
(डी) 81 ड ी

Question 30. In which of the following places, would you find the least difference in the
duration between day time and night time?
(a) Kanyakumari
(b) Leh
(b) Srinagar
(d) Itanagar
30. न न ल खत म से कस ान पर आप दन और रात के समय क अव ध म सबसे कम अंतर पाएंग?े
(ए) क याकु मारी
(बी) लेह
(बी) ीनगर
(डी) ईटानगर

Question 31. Which of the following Union Territories is located along the west coast of
India?
(a) Andaman and Nicobar Islands
(b) Chandigarh
(c) Dadra and Nagar Haveli
(d) Puducherry
31. न न ल खत म से कौन सा क शा सत दे श भारत के प मी तट के साथ त है?
(ए) अंडमान और नकोबार प समूह
(बी) चंडीगढ़
(सी) दादरा और नगर हवेली
(डी) पुडुचरे ी

Question 32. Which of the following places of India is located on the three seas?
(a) Port Blair
(b) Kavaratti
(c) Kanyakumari
(d) Kochi
32. भारत का न न ल खत म से कौन सा ान तीन समु पर त है?
(ए) पोट लेयर
(बी) कवर ी
(सी) क याकु मारी
(डी) को
Question 33. Which of the following parallels of latitude divides India into two almost
equal parts?
(a) Equator
(b) Tropic of Capricorn
(c) Tropic of Cancer
(d) Prime Meridian
33.अ ांश के न न ल खत म से कौन सा समानांतर भारत को लगभग दो बराबर भाग म वभा जत करता है?
(ए) भूम य रेखा
(बी) मकर रेखा
(सी) कक रेखा
(डी) ाइम मे र डयन

Question 34. Which geographical feature bounds India’s mainland south of 22°N
latitude?
(a) Young Fold Mountains
(b) Sandy Desert
(c) Lava Plateaus
(d) Seas and Ocean
34. कौन सी भौगो लक वशेषता भारत क मु य भू म को 22°N अ ांश के द ण म बांधती है?
(ए) यंग फो पवत
(बी) सडी रे ग तान
(सी) लावा पठार
(डी) समु और महासागर

Question 35. Which of the following figures shows the total area of India’s landmass?
(a) 2.4 million square km
(b) 3.28 million square km
(c) 32.8 million square km
(d) 3.28 million km
35. न न ल खत म से कौन सा आंकड़ा भारत के कु ल े फल को दशाता है?
(ए) 2.4 म लयन वग कमी
(बी) 3.28 म लयन वग कमी
(सी) 32.8 म लयन वग कमी
(डी) 3.28 म लयन कमी

Question 36. Both the latitudinal and longitudinal extent of India’s mainland is about 30°.
But on looking at the map of India which of the following alternatives do you observe
about India’s size?
(a) East-west extent appears to be smaller than north-south extent
(b) East-west extent appears to be larger than north-south extent
(c) East-west and north-south extent appears equal
(d) North-south extent appears to be smaller than east-west extent
36. भारत क मु य भू म का अ ांशीय और दे शांतर दोन ही व तार लगभग 30° है। ले कन भारत के न े
को दे खने पर आपको भारत के आकार के बारे म न न ल खत म से कौन सा वक प दखाई दे ता है?
(ए) पूव-प म व तार उ र-द ण सीमा से छोटा तीत होता है
(बी) पूव-प म व तार उ र-द ण सीमा से बड़ा तीत होता है
(सी) पूव-प म और उ र-द ण सीमा बराबर दखाई दे ती है
(डी) उ र-द ण व तार पूव-प म सीमा से छोटा तीत होता है

Question 37. Which of the following influences the duration of the day and night, as one
moves from south to north?
(a) Longitudinal extent
(b) Latitudinal extent
(c) Standard Meridian
(d) All the above
37. न न ल खत म से कौन द ण से उ र क ओर बढ़ने पर दन और रात क अव ध को भा वत करता है?
(ए) अनुदै य व तार
(बी) अ ांशीय व तार
(सी) मानक मे र डयन
(डी) उपरो सभी
Question 38. Which one of the following water bodies separate Sri Lanka from India?
(a) Palk Strait and Gulf of Khambhat
(b) Palk Strait and Gulf of Mannar
(c) Gulf of Mannar and 10° Channel
(d) 10° Channel and Gulf of Khambhat
38. न न ल खत म से कौन सा जल नकाय ीलंका को भारत से अलग करता है?
(ए) पाक जलडम म य और खंभाट क खाड़ी
(बी) पाक जलडम म य और म ार क खाड़ी
(सी) म ार क खाड़ी और 10 ड ी चैनल
(डी) 10 ड ी चैनल और खंभात क खाड़ी

Question 39. What is the length of Indian coastline?


(a) 8716 km
(b) 7165 km
(c) 9515 km
(d) 7516 km
39. भारतीय समु तट क लंबाई कतनी है?
(ए) 8716 कमी
(बी) 7165 कमी
(सी) 9515 कमी
(डी) 7516 कमी

Question 40. Which one of the following straits separates India from Sri Lanka?
(a) Sunda Strait
(b) Johor Strait
(c) Bering Strait
(d) Palk Strait
40. न न ल खत म से कौन सा जलडम म य भारत को ीलंका से अलग करता है?
(ए) सुंडा जलडम म य
(बी) जोहोर े ट
(सी) बे रग जलडम म य
(डी) पाक जलडम म य
Question 41. India has land boundary of about
(a) 18,200 km
(b) 7516.6 km
(c) 15,200 km
(d) 2458 km
41. भारत क भू म सीमा लगभग है
(ए) 18,200 कमी
(बी) 7516.6 कमी
(सी) 15,200 कमी
(डी) 2458 कमी
Question 42. Which one of the following forms the southern-most tip of the Indian
mainland?
(a) Indira Point
(b) Kanyakumari
(c) Palk Strait
(d) Kavaratti

42. न न ल खत म से कौन भारतीय मु य भू म का सबसे द णी छोर बनाता है?


(ए) इं दरा वाइंट
(बी) क याकु मारी
(सी) पाक जलडम म य
(डी) कवर ी

Question 43. What is the position of India in the world in respect of area?
(a) 8th position
(b) 7th position
(c) 6th position
(d) 2nd position
43. े फल क से भारत का व म कौन सा ान है?
(ए) 8 वां ान
(बी) 7 वां ान
(सी) 6 वां ान
(डी) सरा ान
Question 44. Tropic of Cancer passes through which of these states
(a) Orissa
(b) Tripura
(c) Bihar
(d) Punjab
44. कक रेखा इनम से कस रा य से होकर गुजरती है
(ए) उड़ीसा
(बी) पुरा
(सी) बहार
(डी) पंजाब
Question 45. Due to which of the following reasons is the Indian Ocean named after
India?
(a) India has a strategic location along the trans-Indian Ocean routes
(b) No other country has a long coastline on the Indian Ocean as India
(c) India is centrally located at the head of the Indian Ocean
(d) All the above
45. न न ल खत म से कस कारण से हद महासागर का नाम भारत के नाम पर पड़ा?
(ए) ांस- हद महासागर माग के साथ भारत का एक रणनी तक ान है
(बी) हद महासागर पर भारत के प म कसी अ य दे श क लंबी तटरेखा नह है
(सी) भारत हद महासागर के शीष पर त है
(डी) उपरो सभी

Question 46. Which of the following is the oldest route of contact between India and
other countries of the world?
(a) Ocean routes
(b) Maritime contact
(c) Land routes
(d) Air routes
46. न न ल खत म से कौन सा भारत और नया के अ य दे श के बीच संपक का सबसे पुराना माग है?
(ए) महासागर माग
(बी) समु संपक
(सी) भू म माग
(डी) हवाई माग

Question 47. India achieved multi-faceted socio-economic progress during which of the
following periods?
(a) Since ancient times
(b) During medieval period
(c) In the 21st century
(d) During the last five decades
47. भारत ने न न ल खत म से कस अव ध के दौरान ब आयामी सामा जक-आ थक ग त हा सल क ?
(ए) ाचीन काल से
(बी) म ययुगीन काल के दौरान
(सी) २१व सद म
(डी) पछले पांच दशक के दौरान
Question 48. Which of the following is the western-most longitude of India?
(a) 97°25′E
(b) 68°7′E
(c) 68°7′E
(d) 82°32′E
48. न न ल खत म से कौन भारत का सबसे प मी दे शांतर है?
(ए) 97°25′ई
(बी) 68°7 ′ई
(सी) 68°7′ई
(डी) 82 ड ी 32'ई'

Question 49. By which geographical feature is India bounded in the north-west, north
and north-east?
(a) Seas
(b) Lave Plateaus
(c) Young Fold Mountains
(d) Sandy Desert
49. भारत कस भौगो लक वशेषता से उ र-प म, उ र और उ र-पूव म घरा है?
(ए) समु
(बी) लव पठार
(सी) यंग फो पवत
(डी) सडी रे ग तान

Question 50. If the local time at Dwarka (69°01′E) in Gujarat to the west of India is 6 am,
what will be the local time at Dibrugarh (94°58′E approximately 95°), in Assam, in the
east?
(a) 4.16 am
(b) 6 am
(c) 7.44 am
(d) 7.44 pm
50. य द भारत के प म म गुजरात म ारका (69°01′E) का ानीय समय सुबह 6 बजे है, तो पूव म असम
म ड ूगढ़ (94°58′E लगभग 95°) का ानीय समय या होगा?
(ए) 4.16 पूवा
(बी) सुबह 6 बजे
(सी) 7.44 पूवा
(डी) 7.44 अपरा
Question 51. Indian Standard Time or I.S.T. is how many hours ahead or behind of
G.M.T. or
Universal Time?
(a) 5 hrs 30 min behind G.M.T.
(b) 15 hrs ahead of G.M.T.
(c) 5 hrs 30 min ahead of G.M.T.
(d) None of the above
51. भारतीय मानक समय या आई.एस.ट . G.M.T से कतने घंटे आगे या पीछे है? या
यू नवसल टाइम?
(ए) जीएमट के पीछे ५ घंटे ३० मनट।
(बी) जीएमट से 15 घंटे आगे।
(सी) जीएमट से 5 घंटे 30 मनट आगे।
(डी) उपरो म से कोई नही

Question 52. Read the extracts and answer the question that follows
The Himalayas, geologically young and structurally fold mountains stretch over the
Himalayas northern borders of India. These mountain ranges run in a west-east
Direction from the Indus to the Brahmaputra. The Himalayas represent the loftiest and
one of the most rugged mountain barriers of the world. They form an arc, which covers
a distance of about 2,400 Km. Their width varies from 400 Km in Kashmir to 150 Km in
Arunachal Pradesh. The altitudinal variations are greater in the eastern half than those
in the western half. The Himalaya consists of three parallel ranges in Its longitudinal
extent. A number of valleys lie between these ranges. The northernmost range is known
as the Great or Inner Himalayas. It is the most continuous range consisting of the
loftiest peaks with an average height of 6,000 meters. It contains all the prominent
Himalayan peaks. The folds of Great Himalayas are asymmetrical in nature. The core of
this part of Himalayas is composed of granite. It is perennially snowing bound, and a
number of Glaciers descend from this range.
हमालय, भूगभ य प से युवा और संरचना मक प से मुड़े ए पहाड़ भारत क हमालय क उ री सीमा पर
फै ले ए ह। ये पवत ृंखलाएं सधु से पु तक प म-पूव दशा म चलती ह। हमालय नया के सबसे ऊंचे
और सबसे ऊबड़-खाबड़ पवतीय अवरोध म से एक का त न ध व करता है। वे एक चाप बनाते ह, जो लगभग
2,400 कमी क री तय करता है। इनक चौड़ाई क मीर म 400 कलोमीटर से लेकर अ णाचल दे श म 150
कलोमीटर तक है। प मी आधे ह से क तुलना म पूव ह से म ऊंचाई क भ ताएं अ धक ह। हमालय अपनी
अनुदै य सीमा म तीन समानांतर े णय से मलकर बना है। इन े णय के बीच कई घा टयाँ ह। सबसे उ री ेणी
को महान या आंत रक हमालय के प म जाना जाता है। यह ६,००० मीटर क औसत ऊंचाई के साथ सबसे ऊंची
चो टय से यु सबसे नरंतर ेणी है। इसम हमालय क सभी मुख चो टयाँ शा मल ह। टे हमालय क तह क
कृ त वषम होती है। हमालय के इस ह से का कोर ेनाइट से बना है। यह बारहमासी बफ़बारी से बंधा आ है,
और कई ले शयर इस सीमा से उतरते ह।
Questions:-
i. The Great or Inner Himalayas is also known as?
महान या भीतरी हमालय को कस नाम से भी जाना जाता है?
ii. Give two features of the folds of Great Himalayas.
महान हमालय क तह क कोई दो वशेषताएँ बताइए।
iii. Give two features of the Inner Himalayas.
आंत रक हमालय क कोई दो वशेषताएँ बताइए।
ANSWER KEY
Answer 1: (b) Orissa (Now Odisha)
Answer2 : (a) 97°25’E
Answer 3: (c) Nepal
Answer 4: (b) Lakshadweep
Answer 5: (b) Tajikistan
Answer 6: (c) India has 28 States and 7 Union Territories
Answer 7: (b) Tropic of Cancer
Answer 8: (b) 7th position
Answer 9: (a) 82½°
Answer 10: (b) 8° 4′ N and 37° 6′ N
Answer 11: (b) Tripura
Answer 12: (b) 82°30′E
Answer 13: (c) Land routes
Answer 14: (d) During the last five decades
Answer 15: (b) 68°7′N and 97°25′E
Answer 16: (a) 97°25′E
Answer 17: (a) 8°4′N
Answer 18: (c) Lakshdweep
Answer 19: (d) Seventh
Answer 20 (d) Seas and Ocean
Answer 21: (c) 7.44 am
Answer 22: (d) Mirzapur in Uttar Pradesh
Answer 23: (a) Suez Canal
Answer 24 (a) Jammu and Kashmir, Himachal Pradesh, Uttarakhand, Arunachal
Pradesh
Answer 25: (b) Strait
Answer 26: (d) All the above
Answer 27: (c) 2.4%
Answer 28: (b) Bhutan
Answer 29: (a) 82½° E
Answer 30: (a) Kanyakumari
Answer 31: (c) Dadra and Nagar Haveli
Answer 32: (c) Kanyakumari
Answer 33: (c) Tropic of Cancer
Answer 34: (d) Seas and Ocean
Answer 35: (b) 3.28 million square km
Answer 36: (a) East-west extent appears to be smaller than north-south extent
Answer 37: (b) Latitudinal extent
Answer 38: (b) Palk Strait and Gulf of Mannar
Answer 39: (d) 7516 km
Answer 40: (d) Palk Strait
Answer 41: (c) 15,200 km
Answer 42: (b) Kanyakumari
Answer 43: (b) 7th position
Answer 44: (b) Tripura
Answer 45: (d) All the above
Answer 46: (c) Land routes
Answer 47: (d) During the last five decades
Answer 48: (b) 68°7′E
Answer 49: (c) Young Fold Mountains
Answer 50: (c) 7.44 am
Answer 51: (c) 5 hrs 30 min ahead of G.M.T.
Answer 52 ( i) greater
(ii) The folds of Great Himalayas are asymmetrical in nature . The core of this part of
Himalayas is composed of granite.
(iii) It is the most continuous range consisting of the loftiest peaks with an average
height of 6,000 meters .It contains all the prominent Himalayan peaks, The folds of
Great Himalayas are asymmetrical in nature

PHYSICAL FEATURES OF INDIA


भारत क भौ तक व प

CASE STUDY BASED QUEST


के स टडी आधा रत :
It is a table land composed of the old crystalline, igneous and metamorphic rocks. It
was formed due to the breaking and drifting of the Gondwana land and thus, making it a
part of the oldest landmass. The plateau has broad and shallow valleys and rounded
hills. This plateau consists of two broad divisions, namely, the Central Highlands and
the Deccan Plateau.

यह पुरानी टलीय, आ नेय और कायांत रत च ान से बनी एक टे बल भू म है। इसका नमाण ग डवाना भू म के


टू टने और बहने के कारण आ था और इस कार, इसे सबसे पुराने भूभाग का ह सा बना दया गया था। पठार म
चौड़ी और उथली घा टयाँ और गोल पहा ड़याँ ह। इस पठार म दो ापक वभाजन ह, अथात् म य उ भू म और
द कन का पठार।
1. Identify the physiographic division described in the given extract.
a) The Himalayan mountains
b) The Peninsular Plateau
c) The Indian Desert
d) The Coastal Plains
1. दए गए उ रण म व णत भौगो लक वभाजन क पहचान कर।
क) हमालय पवत
ख) ाय पीय पठार
ग) भारतीय रे ग तान
घ) तट य मैदान

2. Which of the following is not true with respect of the Physiographic Division identified
by
You?
a) It is an interplay of three major river systems namely the Indus, the Ganga and the
Brahmaputra.
b) The Chambal, the Sind and the Betwa are the rivers which drain this region.
i) Only a is true
ii) Only b is true
iii) Both a and b are true
iv) Neither a nor b is true

2. न न ल खत म से कौन-सा कसके ारा पहचाने गए फ जयो ा फक डवीजन के संबंध म


सही नह है?
a) यह तीन मुख नद णा लय अथात् सधु, गंगा और नद क पर र या है
पु ।
b) चंबल, सध और बेतवा न दयाँ ह जो इस े को बहाती ह।
i) के वल a स य है
ii) के वल b स य है
iii) a और b दोन स य ह
iv) न तो a और न ही b स य है

3. The part of the Peninsular plateau lying to the north of the Narmada river, covering a
major area of Malwa plateau is known as..........
a) Western Ghats
b) Eastern Ghats
c) The Deccan Plateau
d) The Central Highland

3. ाय पीय पठार का वह भाग जो नमदा नद के उ र म त है, मालवा पठार के एक बड़े े को कवर करता
है ......… के प म जाना जाता है।
क) प मी घाट
ख) पूव घाट
ग) द कन का पठार
घ) स ल हाइलड

4. Which of the following is the prominent soil found in the Peninsular plateau?
a) Red soil
b) Laterite soil
c) Black soil
d) Alluvial soil

4. ाय पीय पठार म पाई जाने वाली मुख मृदा न न म से कौन-सी है?


क) लाल म
ख) लैटेराइट म
ग) काली म
घ) जलोढ़ म

2. CASE STUDY QUESTIONS:

These, geologically young and structurally fold mountain stretch over the northern
borders of India. These mountain ranges run in a west- east direction from the Indus to
the Brahmaputra. These represent the loftiest and one of the most rugged mountain
barriers of the world. They form an arc, which covers a distance of about 2400 km.
There width varies from 400 km in Kashmir to 150 km in Arunachal Pradesh. The
altitudinal variations are greater in the Eastern half than those in the Western half.
These consist of three parallel ranges in its longitudinal extent. A number of valleys lie
between these ranges.

ये, भूगभ य प से युवा और भारत क उ री सीमा पर संरचना मक प से मुड़े ए पवत ह। ये पवत ृंखलाएं
सधु से पु तक प म-पूव दशा म चलती ह। ये नया के सबसे ऊंचे और सबसे ऊबड़-खाबड़ पवतीय
अवरोध म से एक का त न ध व करते ह। वे एक चाप बनाते ह, जो लगभग 2400 कमी क री तय करता है।
क मीर म इसक चौड़ाई 400 कमी से लेकर अ णाचल दे श म 150 कमी तक है। प मी आधे ह से क
तुलना म पूव ह से म ऊंचाई भ ताएं अ धक ह। इनम अनुदै य व तार म तीन समानांतर े णयां शा मल ह। इन
े णय के बीच कई घा टयाँ ह।
1. Which of the following physiographic divisions have been described in the given
extract?
a) The Coastal Plains
b) The Islands
c) The Himalayan Mountains
d)The Peninsular Plateau

1. दए गए उ रण म न न ल खत म से कौन से भौगो लक वभाजन का वणन कया गया है?


क) तट य मैदान
ख) प समूह
ग) हमालय पवत
घ) ाय पीय पठार
2. Which of the following three parallel ranges belong to the Physiographic division
identified
by you?

a) Garo, Khasi, Jantia


b) Aravali, Vindhya and Satpura
c) Himadri, Himachal and Shivalik
d) Sulaiman, Hindukush and Kunlun

2. न न ल खत तीन समानांतर े णय म से कौन-सी पहचान कए गए भौ तक


व ान भाग से संबं धत है?
क) गारो, खासी, जं तया
ख) अरावली, व य और सतपुड़ा
ग) हमा , हमाचल और शवा लक
घ) सुलेमान, ह कु श और कु नलुन
3. .................and.............. peaks are located in the Himalayas.

a) Anai Mudi and Kanchenjunga


b) Kanchenjunga and Dhaulagiri
c) Dhaulagiri and Anai Mudi
d) Mahendragiri and Anai Mudi

3.................और............ शखर हमालय म त ह।


क) अनाई मुडी और कं चनजंगा
ख) कं चनजंगा और धौला गरी
ग) धौला गरी और अनाई मु दक
घ) मह ग र और अनाई मु दक
4. The lofiest peak lying in India is
a) Mt.Everest
b) Kanchenjunga
c) Nanda Devi
d) Anai Mudi
4. भारत क सबसे ऊँची चोट है
क) माउं ट एवरे ट
ख) कं चनजंगा
ग) नंदा दे वी
घ) अनाई मु दक

3. CASE STUDY QUESTIONS:

The Northern plains have been formed by the interplay of the three major river systems
namely the Indus the Ganga and the Brahmaputra along with their tributaries. This plain
is formed of alluvial soil. The deposition of Alluvium in a vast basin lying at the foothills
of the Himalaya over millions of years formed this fertile plain. It spread over an area of
7 lakh sq. km. The plain being about 2400 km long and 240 to 320 km broad, is a
densely populated physiographic division. With a rich soil cover combined with
adequate water supply and favourable climate, it is agriculturally a productive part of
India.
The rivers coming from northern mountains are involved in depositional work. In the
lower course, due to a gentle slope, the velocity of the river decreases, This results in
the formation of riverine islands. Majauli, in the Brahmaputra river, is the highest
inhabited riverine island in the world. The rivers in the lower course splits into numerous
channels due to the deposition of silt. These channels are known as distributaries.

उ री मैदान का नमाण तीन मुख नद णा लय सधु गंगा और पु के साथ-साथ उनक सहायक न दय के


पर र या ारा कया गया है। यह मैदान जलोढ़ म से बना है। लाख वष से हमालय क तलहट म पड़े एक
वशाल बे सन म जलोढ़ के जमाव ने इस उपजाऊ मैदान का नमाण कया। यह 7 लाख वग कमी के े म फै ला
है। लगभग २४०० कमी लंबा और २४० से ३२० कमी चौड़ा होने के कारण यह मैदान एक घनी आबाद वाला
भौगो लक े है। पया त जल आपू त और अनुकूल जलवायु के साथ समृ म के आवरण के साथ, यह कृ ष
प से भारत का एक उ पादक ह सा है।
उ री पवत से आने वाली न दयाँ न ेपण के काय म लगी होती ह। नचले माग म कोमल ढलान के कारण नद का
वेग कम हो जाता है, जसके प रणाम व प नद के प का नमाण होता है। मजौली, पु नद म, नया का
सबसे ऊंचा बसा आ नद प है। गाद के जमाव के कारण नचले माग क न दयाँ कई चैनल म वभा जत हो
जाती ह। इन चैनल को वतरक के प म जाना जाता है।
1. The source given above relates to which of the following options?
a) The Coastal plains
b) The Islands
c) The Northern plains
d) Drainage system

1. ऊपर दया गया ोत न न ल खत म से कस वक प से संबं धत है?


क) तट य मैदान
ख) प समूह
ग) उ री मैदान
घ) ेनेज स टम
2. The Northern Plains have been formed by which of the following rivers?
a) The Indus
b) The Ganga
c) The Brahmaputra
d) All of these.

2. उ री मैदान का नमाण न न ल खत म से कस नद से आ है?


क) सधु
ख) गंगा
ग) पु
घ) ये सभी।
3. The rivers in the lower course split into numerous channels due to deposition of........ ,
these channels are known as........
a) silt, distributaries
b) clay, gulf
c) gravel, strait
d) sound, isthmus

3. नचले माग क न दयाँ ......... के जमाव के कारण कई चैनल म वभा जत हो जाती ह, इन चैनल को ......... के
प म जाना जाता है।
क) गाद, वत रकाएं
ख) म , खाड़ी
ग) बजरी, जलडम म य
घ) व न, इ मुस
4. Which of the following is the largest inhabited riverine island in the world?
a) Majuli island
b) Bhoka island
c) Richards island
d) Sumba island

4. न न ल खत म से कौन सा व का सबसे बड़ा बसा आ नद प है?


क) माजुली प
ख) भोका प
ग) रचड् स प
घ) सुंबा प
PHYSICAL FEATURES OF INDIA
भारत क भौ तक व प
MULTIPLE CHOICE QUESTION ANSWERS:
ब वक पीय उ र:
Question1.
The northern most range of the Himalayas is known as
(a) Himadri
(b) lesser Himalaya
(c) Shivalik
(d) none of the above.
1
हमालय क सबसे उ री सीमा को के प म जाना जाता है
(क) हमा
(ख) कम हमालय
(ग) शवा लक
(घ) उपरो म से कोई नह ।

Question 2.
The Kangra and Kullu valley are located in
(a) Uttarakhand
(b) Jammu and Kashmir
(c) Himachal Pradesh
(d) Uttar Pradesh
2
कांगड़ा और कु लू घाट . म त ह
(क) उ राखंड
(ख) ज मू और क मीर
(ग) हमाचल दे श
(घ) उ र दे श

Question 3.
Majuli is a riverine island located in the river
(a) Ganga
(b) Brahmaputra
(c) Yamuna
(d) Kaveri

3
माजुली नद म त एक नद प है
(क) गंगा
(ख) पु
(ग) यमुना
(घ) कावेरी

Question 4.
Dudhwa National Park is located in the state of
(a) Assam
(b) Uttar Pradesh
(c) Madhya Pradesh
(d) Rajasthan
4
धवा रा ीय उ ान रा य म त है
(क) असम
(ख) उ र दे श
(ग) म य दे श
(घ) राज ान

Question 5.
The soil containing calcareous deposits is locally known as
(a) Bhangar
(b) Khadar
(c) Bhabar
(d) Kankar
5
चने क मा ा वाली म को ानीय प से जाना जाता है
(क) भंगारी
(ख) खादरी
(ग) भाबरी
(घ) कांकरो

Question 6.
The highest peak in the Eastern Ghats is
(a) Anai Mudi
(b) Doda Betta
(c) Ooty
(d) Mahendragiri

6
पूव घाट क सबसे ऊँची चोट है
(क) अनाई मु दक
(ख) डोडा बे ा
(ग) ऊट
(घ) मह ग र

Question 7.
Jaisalmer is in
(a) Himachal Pradesh
(b) Rajasthan
(c) Gujarat
(d) Uttar Pradesh
7
जैसलमेर म है
(क) हमाचल दे श
(ख) राज ान
(ग) गुजरात
(घ) उ र दे श

Question 8.
Barchans are found in
(a) desert regions
(b) plains
(c) plateaus
(d) hilly regions
8
बरचन पाए जाते ह
(क) रे ग तानी े
(ख) मैदानी
(ग) पठार
(घ) पहाड़ी े

Question 9.
The northern part of the western coast is called the
(a) Konkan
(b) Kannad plain
(c) Malabar coast
(d) Coromandel coast
9
प मी तट के उ री भाग को कहा जाता है
(क) क कण
(ख) क ड़ मैदान
(ग) मालाबार तट
(घ) कोरोमंडल तट

Question 10.
Lake Chilika lies in the state of
(a) Jharkhand
(b) Telangana
(c) Odisha
(d) Tamil Nadu
10
च का झील कस रा य म त है
(क) झारखंड
(ख) तेलंगाना
(ग) ओ डशा
(घ) त मलनाडु

Question 11.
A landmass bounded by sea on three sides is referred to as
(a) Coast
(b) Island
(c) Peninsula
(d) None of the above
11
तीन ओर से समु से घरे भू-भाग को या कहते ह?
(क) एक तट
(ख) प
(ग) ाय प
(घ) उपरो म से कोई नह
Question 12.
Mountain ranges in the eastern part of India forming its boundary with Myanmar are
collectively called as
(a) Himachal
(b) Uttarakhand
(c) Purvachal
(d) none of the above
12
भारत के पूव भाग म यांमार के साथ अपनी सीमा बनाने वाली पवत ृंखला को सामू हक प से कहा जाता है
(क) हमाचल
(ख) उ राखंड
(ग) पूवाचल
(घ) उपरो म से कोई नह

Question 13.
The western coastal strip, south of Goa is referred to as;
(a) Coromandel
(b) Konkan
(c) Kannad
(d) Northern Circar
13
गोवा के द ण म प मी तट य प को कहा जाता है;
(क) कोरोमंडल
(ख) क कण
(ग) क ड़
(घ) उ री सकार
Question 14.
The highest peak in the Eastern Ghats is
(a) Anai Mudi
(b) Kanchenjunga
(c) Mahendragiri
(d) Khasi
14
पूव घाट क सबसे ऊँची चोट है
(क) अनाई मु दक
(ख) कं चनजंगा
(ग) मह ग र
(घ) खासी

Question 15.
A narrow gap in a mountain range providing access to the other side is:
(a) Mound
(b) Pass
(c) Strait
(d) Valley
15
एक पवत ंखला म एक संकरी खाई जो सरी ओर प ँच दान करती है :
(क) माउं ड
(ख) पास
(ग) जलडम म य
(घ) घाट
Question 16.
Which two hills are located in the south-east of Eastern Ghats?
(a) Mizo Hills and Naga Hills
(b) Javadi Hills and Shevroy Hills
(c) Patkoi Hills and Manipuri Hills
(d) Mizo Hills and Patkoi Hills
16
पूव घाट के द ण-पूव म कौन सी दो पहा ड़याँ त ह?
(क) मजो ह स और नागा ह स
(ख) जावड़ी ह स और शेवरॉय ह स
(ग) पटकॉय ह स और म णपुरी ह स
(घ) मजो ह स और पटकॉय ह स

Question 17.
Match the following:
LIST 1 LIST 2
A. Punjab Himalayas 1. Teesta – Dihang rivers
B. Nepal Himalayas 2. Satluj –Kali rivers
C. Assam Himalayas 3. Kali – Teesta rivers
D. Kumaon Himalayas 4. Indus – Satluj rivers
Codes:-
A B C D
a) 4 3 1 2
b) 3 2 1 4
c) 1 2 3 4
d) 1 4 3 2
17
न न ल खत को मलाएं:
सूची 1 सूची 2
क. पंजाब हमालय 1. ती ता- दहांग न दयाँ
ख. नेपाल हमालय 2. सतलुज-काली न दयां
ग. असम हमालय 3. काली-ती ता न दयां
घ. कु माऊं हमालय 4. सधु-सतलुज न दयां
कोड:-
क ख ग घ
a) 4 3 1 2
b) 3 2 1 4
c) 1 2 3 4
d) 1 4 3 2
Question 18.
Assertion (A) The Chambal, Sind, Betwa and Ken rivers flow from South West to North –
east.
Reason (R) The Central highland slopes from South West to North – East direction.

Codes:
a) Both A and R are true and R is the correct explanation of A
b) Both A and R are true, but R is not the correct explanation of A
c) A is true, but R is false
d) A is false, but R is true

18
अ भकथन (A) चंबल, सध, बेतवा और के न न दयाँ द ण प म से उ र-पूव क ओर बहती ह।
कारण (आर) द ण प म से उ र-पूव दशा म म य उ भू म ढलान।
कोड:
(क) A और R दोन सही ह और R, A क सही ा या करता है
(ख) A और R दोन सही ह, ले कन R, A क सही ा या नह करता है
(ग) ए सच है, ले कन आर झूठा है
(घ) ए झूठा है, ले कन आर सच है

Question 19.
Assertion (A) The Himalyas forms the highest and the rugged mountains in the world.
Reason (R) The Mount Everest is the highest peak in the world lying in the Himalayas.
Codes:
a)Both A and R are true and R is the correct explanation of A
b) Both A and R are true, but R is not the correct explanation of A
c) A is true, but R is false
d) A is false, but R is true

19
अ भकथन (ए) हमालय नया म सबसे ऊंचे और ऊबड़ पहाड़ का नमाण करता है।
कारण (R) माउं ट एवरे ट हमालय म त व क सबसे ऊँची चोट है।
कोड:
(क) A और R दोन सही ह और R, A क सही ा या करता है
(ख) A और R दोन सही ह, ले कन R, A क सही ा या नह करता है
(ग) ए सच है, ले कन आर झूठा है
(घ) ए झूठा है, ले कन आर सच है
Question 20.
Consider the following statements about Deccan Plateau
1. Deccan Plateau has a triangular landmass.
2. Deccan Plateau lies south of the Narmada river.
3. Satpura Range lies north of the Deccan Plateau.
Which one is correct?
codes
(a) Only 1
(b) Only 2
(c) 1 and 2
(d) all of these
20
द कन के पठार के बारे म न न ल खत कथन पर वचार कर
1. द कन के पठार म एक भुजाकार भूभाग है।
2. द कन का पठार नमदा नद के द ण म त है।
3. सतपुड़ा रज द कन के पठार के उ र म त है।
कौनसा सही है?
कोड
(क) के वल 1
(ख) के वल 2
(ग) 1 और 2
(घ) ये सभी
Question 21.
CORRECT AND REWRITE
The Lakshadweep island groups are volcanic Islands of India while the Andaman and
Nicobar islands are Coral Islands.
(a) The Lakshadweep Island Groups volcanic island of India while the Andaman and
Nicobar
islands are also volcanic Islands of India.
(b) The Lakshadweep Island groups are Coral islands while the Andaman and Nicobar
islands are volcanic Islands of India.
(c) The Lakshadweep Islands Group are volcanic islands while the Andaman and
Nicobar islands
are volcanic Islands of India.
21
सही कर और फर से लख
ल प प समूह भारत के वालामुखी प समूह ह जब क अंडमान और नकोबार प समूह कोरल प
समूह ह।
(क) ल प समूह भारत के वालामुखी प समूह जब क अंडमान और नकोबार
प भी भारत के वालामुखी प ह।
(ख) ल प प समूह कोरल प समूह ह जब क अंडमान और नकोबार प समूह
भारत के वालामुखी प ह।
(ग) ल प प समूह वालामुखी प ह जब क अंडमान और नकोबार प समूह
भारत के वालामुखी प ह।
Question 22.
Which of the following are young-fold Mountains?
(a) The Aravalis
(b) The Nilgiris
(c) The Himalayas
(d) The Sahyadri
22
न न ल खत म से कौन-से युवा व लत पवत ह?
(क) अरावली
(ख) नील गरी
(ग) हमालय
(घ) स ा
Question 23.
Geologically, which of the following physiographic divisions of India is supposed to be
one of the most stable land blocks?
(a) The Himalayas
(b) The Northern Plains
(c) The Peninsular Plateau
(d) The Indian Desert
23.
भूगभ य प से, भारत के न न ल खत म से कौन सा भौगो लक वभाजन सबसे र भू म लॉक म से एक माना
जाता है?
(क) हमालय
(ख) उ री मैदान
(ग) ाय पीय पठार
(घ) भारतीय रे ग तान
Question 24.
Which of the following physiographic divisions of India was formed out of
accumulations in the Tethys geosynclines?
(a) The Himalayas
(b) The Northern Plains
(c) The Peninsular Plateau
(d) The Indian Desert
24
भारत के न न ल खत म से कौन सा भौगो लक वभाजन टे थस जयो सकलाइन म संचय से बना था?
(क) हमालय
(ख) उ री मैदान
(ग) ाय पीय पठार
(घ) भारतीय रे ग तान
Question 25.
Which of the following countries or continents was not a part of the ancient landmass
of Gondwanaland?
(a) India
(b) Australia
(c) Europe
(d) South America
25
न न ल खत म से कौन सा दे श या महा प ग डवानालड के ाचीन भूभाग का ह सा नह था?
(क) भारत
(ख) ऑ े लया
(ग) यूरोप
(घ) द ण अमे रका

Question 26.
Match the following:
LIST 1 LIST 2
A. Makalu 1. 8078 m
B. Annapurna 2. 8481 m
C. Kanchenjunga 3. 8598 m
D. Nanda Devi 4.7817 m

Codes:-
A B C D
a) 2 1 3 4
b) 3 2 1 4
c) 1 2 3 4
d) 1 4 3 2
26
न न ल खत को मलाएं:
सूची 1 सूची 2
क. मकालू 1. ८०७८ मी
ख. अ पूणा 2. ८४८१ मी
ग. कं चनजंगा 3. ८५९८ मी
घ. न दा दे वी 4. ७८१७ मी
कोड:-
क ख ग घ
a) 2 1 3 4
b) 3 2 1 4
c) 1 2 3 4
d) 1 4 3 2
Question 27.
Which type of climate the Aravali region has?
a) Temperate climate
b) Arid climate
c) Tropical climate
d) Mediterranean climate
27
अरावली े क जलवायु कस कार क है?
क) समशीतो ण जलवायु
ख) शु क जलवायु
ग) उ णक टबंधीय जलवायु
घ) भूम यसागरीय जलवायु
Question 28.
Find the incorrect option.
(a) Luni - river in the Indian desert
(b) Kaveri river - forms delta in the Northern circar
(c) Anai Mudi - Highest peak in the Western Ghats
(d) Baghelkhand - Eastward extension of the Central highlands.
28
गलत वक प खोज।
(क) लूनी - भारतीय रे ग तान म नद
(ख) कावेरी नद - उ री सकार म डे टा बनाती है
(ग) अनाई मुडी - प मी घाट म सबसे ऊंची चोट
(घ) बघेलखंड - क य उ भू म का पूव क ओर व तार।

Millions of years ago, the Peninsular Plateau region (the oldest landmass) was a part of
the Gondwana Land which covered India, Australia, South Africa, and South America. Over
hundreds of years of shifting landmass and ocean currents broke this landmass into
multiple pieces.

One such piece- the Indo-Australian plate started shifting northwards, where it collided
with the Eurasian plate (now Europe). Consequently, this collision caused the landmass to
fold and become, what we know as the Himalayas today. Thereafter, many such
geological events led to the formation of each of the varied physical features of India.

लाख साल पहले, ाय पीय पठार े (सबसे पुराना भूभाग) ग डवाना भू म का एक ह सा था जो भारत,
ऑ े लया, द ण अ का और द ण अमे रका को कवर करता था। सैकड़ वष से अ धक समय तक बदलते
भूभाग और महासागरीय धारा ने इस भूभाग को कई टुकड़ म तोड़ दया।
ऐसा ही एक टुकड़ा- इंडो-ऑ े लयाई लेट उ र क ओर खसकने लगी, जहां यह यूरे शयन लेट (अब यूरोप) से
टकरा गई। नतीजतन, इस ट कर ने भूभाग को मोड़ दया और बन गया, जसे आज हम हमालय के प म जानते
ह। इसके बाद, ऐसी कई भूगभ य घटना ने भारत क व भ भौ तक वशेषता म से येक का नमाण कया।
Q29. Choose the correct option
Assertion: India is a vast country with varied landforms.
Reason: It has practically all major physical features of the earth i.e., mountains, plains,
deserts, plateau and islands.
Codes:
a)Both A and R are true and R is the correct explanation of A
b) Both A and R are true, but R is not the correct explanation of A
c) A is true, but R is false
d) A is false, but R is true
29. सही वक प चुन
अ भकथन: भारत व वध भू-आकृ तय वाला एक वशाल दे श है।
कारण: इसम ावहा रक प से पृ वी क सभी मुख भौ तक वशेषताएं ह, जैसे पहाड़, मैदान, रे ग तान, पठार
और प।
कोड:
(क) A और R दोन सही ह और R, A क सही ा या करता है
(ख) A और R दोन सही ह, ले कन R, A क सही ा या नह करता है
(ग) ए सच है, ले कन आर झूठा है
(घ) ए झूठा है, ले कन आर सच है

Q30. From geological point of view, one of the ancient landmasses in India is
a) Central Highland
b) Peninsular plateau
c) Himalayas
d) None
30. भूवै ा नक कोण से, भारत के ाचीन भूभाग म से एक है
(क) स ल हाइलड
(ख) ाय पीय पठार
(ग) हमालय
(घ) कोई नह

Q31. Which of the following countries or continents was not a part of the ancient land
mass of
Gondwanaland?
a) India
b) Australia
c) Europe
d) South America

31. न न ल खत म से कौन सा दे श या महा प ाचीन भू म का ह सा नह था


ग डवानालड?
(क) भारत
(ख) ऑ े लया
(ग) यूरोप
(घ) द ण अमे रका

Q32. Which continents of today were parts of the Gondwanaland?


a) Asia and Africa
b) Europe and Africa
c) Europe and Asia
d) Australia and South America
32. आज के कौन से महा प ग डवानालड के ह से थे?
(क) ए शया और अ का
(ख) यूरोप और अ का
(ग) यूरोप और ए शया
(घ) ऑ े लया और द ण अमे रका
Q33. Which of the following is a plausible theory presented by Earth Scientist to explain
the
formation of continents and ocean and various landforms?
a) Theory of Motion
b) Theory of Plate tectonics
c) Theory of Evolution
d) Theory of Relativity

33. न न ल खत म से कौन सा पृ वी वै ा नक ारा तुत कया गया एक शंसनीय स ांत है जसे


समझाने के लए महा प और महासागर और व भ भू-आकृ तय का नमाण?
(क) ग त का स ांत
(ख) लेट ववत नक का स ांत
(ग) वकास का स ांत
(घ) सापे ता का स ांत
Topography and wind play an important role in the distribution of precipitation. The
Ghats, which mean Western and Eastern Ghats, are two mountain ranges forming the
eastern and western edges, respectively, of the Deccan plateau of peninsular India.
The terms Ghats is derived from Hindi which means “river landing stairs” or “mountain
pass”. Western Ghats extends from the mount of the river Tapi to the Cape of
Kanyakumari for a distance of 1600 km. Its average height is 1200 m. It is not a real hill
range; rather it is a rift side of the peninsular plateau. The height of the Western Ghats
increases from North to South. Nilgiris Hills, Annamalai Hills, and Cardamom Hills are
major hills. Eastern Ghats located along the east coast in Tamil Nadu, Andhra Pradesh
and Odisha. The average height of the Eastern Ghats is 600m. Stanley Shevaroy Hills,
Javadi Hills, Palkonda Range and Nallamala Hills are major hills.
The Western Ghats block rain-bearing winds which cause rainfall on the western slopes.
Whereas South-west monsoon moves parallel to the Eastern Ghats, which cause less
rainfall because in the Eastern Ghats unable to block moisture-laden winds. The
Western Ghats lies in rain-fed area of the Arabian Sea branch of the south-west
monsoon whereas Eastern Ghats lies in the rain shadow area of the Arabian Sea branch
of the south-west monsoon.

लाकृ त और पवन वषा के वतरण म मह वपूण भू मका नभाते ह। घाट, जसका अथ है प मी और पूव घाट,
ाय पीय भारत के द कन पठार के मशः पूव और प मी कनार का नमाण करने वाली दो पवत ृंखलाएं
ह।
घाट श द हद से लया गया है जसका अथ है "नद उतरने क सी ढ़याँ" या "माउं टेन पास"। प मी घाट तापी नद
के पवत से क याकु मारी के के प तक 1600 कमी क री तक फै ला आ है। इसक औसत ऊंचाई 1200 मीटर
है। यह एक वा त वक पहाड़ी ृंखला नह है; ब क यह ाय पीय पठार का एक दरार प है। प मी घाट क
ऊंचाई उ र से द ण क ओर बढ़ती है। नील गरी पहा ड़याँ, अ ामलाई पहा ड़याँ और इलायची पहा ड़याँ मुख
पहा ड़याँ ह। पूव घाट त मलनाडु , आं दे श और ओ डशा म पूव तट पर त है। पूव घाट क औसत ऊंचाई
600 मीटर है। टे नली शेवरॉय ह स, जावडी ह स, पालक डा रज और न लामाला ह स मुख पहा ड़यां ह।
प मी घाट बा रश वाली हवा को रोकते ह जो प मी ढलान पर वषा का कारण बनती ह। जब क द ण-
प म मानसून पूव घाट के समानांतर चलता है, जससे कम वषा होती है य क पूव घाट म नमी से भरी हवा
को अव करने म असमथ होते ह। प मी घाट द ण-प म मानसून क अरब सागर शाखा के वषा आधा रत
े म त है जब क पूव घाट द ण-प म मानसून क अरब सागर शाखा के वषा छाया े म त है।
Q34. The average elevation of Western Ghats is
a) 600 m to 1000 m
b) 900 m to 1600 m
c) 700 m to 1200 m
d) 900 m to 2000 m

34. प मी घाट क औसत ऊंचाई है


(क) ६०० मीटर से १००० मीटर
(ख) 900 मी से 1600 मी
(ग) 700 मीटर से 1200 मीटर
(घ) 900 मी से 2000 मी

Q35. The height of Western Ghats increases from....


a) North to West
b) North to East
c) North to South
d) South to North

35 . प मी घाट क ऊंचाई कससे बढ़ती है?...


(क) उ र से प म
(ख) उ र से पूव
(ग)) उ र से द ण
(घ) द ण से उ र

Q36. Assertion: South - west monsoon moves parallel to the Eastern Ghats which cause
less
rainfall.
Reason: Eastern Ghats unable to block moisture - laden winds.
Codes:
a)Both A and R are true and R is the correct explanation of A
b) Both A and R are true, but R is not the correct explanation of A
c) A is true, but R is false
d) A is false, but R is true

36. अ भकथन: द ण-प म मानसून पूव घाट के समानांतर चलता है जससे कम


वषा होती है।
कारण: पूव घाट नमी से लद हवा को रोक नह पा रहे ह।
कोड:
(क) A और R दोन सही ह और R, A क सही ा या करता है
(ख) A और R दोन सही ह, ले कन R, A क सही ा या नह करता है
(ग) ए सच है, ले कन आर झूठा है
(घ) ए झूठा है, ले कन आर सच है

Q37. Which Ghat lies in the rain shadow area of Bay of Bengal branch of the south - west
monsoon?
a) Western Ghats
b) Eastern Ghats
c) Both A and B
d) None of these

37. कौन सा घाट द ण-प म मानसून क बंगाल क खाड़ी शाखा के वषा छाया े म त है?
a) प मी घाट
b) पूव घाट
सी) ए और बी दोन
घ) इनम से कोई नह
Q38. Assertion: The Western Ghats block rain-bearing winds which cause rainfall.
Reason: Western Ghats unable to block moisture-laden winds.
Codes:
a)Both A and R are true and R is the correct explanation of A
b) Both A and R are true, but R is not the correct explanation of A
c) A is true, but R is false
d) A is false, but R is true

38. अ भकथन: प मी घाट वषा करने वाली हवा को रोकते ह जो वषा का कारण बनती ह।
कारण: प मी घाट नमी से लद हवा को रोकने म असमथ ह।
कोड:
(क) A और R दोन सही ह और R, A क सही ा या करता है
(ख) A और R दोन सही ह, ले कन R, A क सही ा या नह करता है
(ग) ए सच है, ले कन आर झूठा है
(घ) ए झूठा है, ले कन आर सच है
PHYSICAL FEATURES OF INDIA
भारत क भौ तक व प
ANSWER KEY
CASE STUDY 1
1 b) The Peninsular Plateau
2 i) Only a is true
3 d) The Central Highland
4 c) Black soil
CASE STUDY 2
1 c) The Himalayan Mountains
2 c) Himadri, Himachal and Shivalik
3 b) Kanchenjunga and Dhaulagiri
4 b) Kanchenjunga
CASE STUDY 3
1 c) The Northern plains
2 d) All of these.
3 a) silt, distributaries
4 a) Majuli island
MULTIPLE CHOICE QUESTION ANSWERS:
ब वक पीय उ र:

1 (a) Himadri
2 (c) Himachal Pradesh
3 (b) Brahmaputra
4 (b) Uttar Pradesh
5 (d) Kankar
6 (d) Mahendragiri
7 (b) Rajasthan
8 (a) desert regions
9 (a) Konkan
10 (c) Odisha
11 (c) Peninsula
12 (c) Purvachal
13 (c) Kannad
14 (c) Mahendragiri
15 (b) Pass
16 (b) Javadi Hills and Shevroy Hills
17 e) 4 3 1 2
18 e) Both A and R are true and R is the correct explanation of A
19 a)Both A and R are true and R is the correct explanation of A
20 (c) 1 and 2
21 (b) The Lakshadweep Island groups are Coral islands while the Andaman and
Nicobar islands are volcanic Islands of India.
22 (c) The Himalayas
23 (c) The Peninsular Plateau
24 (a) The Himalayas
25 (c) Europe
26 e) 2 1 3 4
27 b) Arid climate
28 (b) Kaveri river - forms delta in the Northern circar
29 a)Both A and R are true and R is the correct explanation of A
30 b) Peninsular plateau
31 c) Europe
32 d) Australia and South America
33 b) Theory of Plate tectonics

34 b) 900 m to 1600 m

35 c) North to South

36 a)Both A and R are true and R is the correct explanation of A

37 d) None of these

38 c) A is true, but R is false

WHAT IS DEMOCRACY? WHY DEMOCRACY?

I. Multiple Choice Questions(MCQs)

Q1. The word ‘Democracy’ comes from the Greek word –


(a) Democracia (b) Demokratia
(b) (c) Demos (d) Kratia
'डेमो े सी' श द ीक श द से आया है –
(ए) डेमो े शया (बी) डेमो े टया
(सी) डेमोस (डी) े टया

Q2. What kind of government is there in Myanmar?


(a) Government elected by the people (b) Communist government
(c) Army rule (d) Monarchy

यांमार म कस तरह क सरकार है?


(ए) लोग ारा चुनी गई सरकार (बी) क यु न ट सरकार
(सी) सेना शासन (डी) राजशाही

Q3. The head of the government in Nepal before 1990 was the:
(a) President (b) Prime Minister
(b) (c) King (d) Vice President

1990 से पहले नेपाल म सरकार का मु खया था:


(ए) रा पत (बी) धान मं ी
(सी) राजा (डी) उपरा प त

Q4. Who led a military coup in Pakistan in 1999?


(a) Benazir Bhutto (b) Nawaz Sharif
(b) (c) Pervez Musharraf (d) None of the above

1999 म पा क तान म सै य त तापलट का नेतृ व कसने कया?


(ए) बेनजीर भु ो (बी) नवाज शरीफ
(सी) परवेज मुशरफ (डी) उपरो म से कोई नह

Q5. In which case was the real power with external powers and not with the locally
elected representatives?
(a) India in Sri Lanka (b) US in Iraq
(b) (c) USSR in Communist Poland (d) Both (b) and(c)

कस मामले म वा त वक श बाहरी श य के पास थी न क ानीय प से नवा चत त न धय के पास?


(ए) ीलंका म भारत (बी) इराक म अमे रका
(सी) क यु न ट पोलड म यूएसएसआर (डी) दोन (बी) और (सी)
Q6. ‘One person, one vote’ means
(a) One person is to be voted by all (b) One person has one vote and each
vote has one value (c) A person can vote only once in his life (d) both (a) and (c)

'एक , एक वोट' का अथ है
(ए) एक को सभी ारा वोट दया जाना है
(बी) एक के पास एक वोट है और येक वोट का एक मू य है
(सी) एक अपने जीवन म के वल एक बार मतदान कर सकता है
(डी) दोन (ए) और (सी)

Q.7. How many members are elected to the National People’s Congress from all over
China?
(a) 3050 (b) 3000 (c) 4000 (d) 2000

पूरे चीन से नेशनल पीपु स कां ेस के लए कतने सद य चुने जाते ह?


(ए) 3050 (बी) 3000 (सी) 4000 (डी) 2000

Q8. Why can the Chinese government not be called a democratic government even
though elections are held there?
(a) Army participates in election
(b) Government is not accountable to the people
(c) Some parts of China are not represented at all
(d) Government is always formed by the Communist Party

चीनी सरकार को वहां चुनाव होने के बावजूद लोकतां क सरकार य नह कहा जा सकता है?
(ए) सेना चुनाव म भाग लेती है
(बी) सरकार लोग के त जवाबदे ह नह है
(सी) चीन के कु छ ह स का त न ध व ब कु ल नह है
(डी) सरकार हमेशा क यु न ट पाट ारा बनाई जाती है

Q9.Which party always won elections in Mexico since its independence in 1930 until
2000?
(a) Revolutionary Party (b) Mexican Revolutionary Party
(c) Institutional Revolutionary Party (d) Institutional Party

कस पाट ने 1930 म अपनी वतं ता के बाद से 2000 तक मे सको म हमेशा चुनाव जीता?
(ए) रवो यूशनरी पाट (बी) मै सकन रवो यूशनरी पाट
(सी) सं ागत ां तकारी पाट (डी) सं ागत पाट

Q10. Democracy must be based on


(a) One-party system (b) Free and fair election
(c) Choice from only the ruling party (d) All the above

लोकतं पर आधा रत होना चा हए


(ए) एक दलीय णाली (बी) वतं और न प चुनाव
(सी) के वल स ा ढ़ दल से वक प (डी) उपरो सभी

Q11. When did Zimbabwe attain independence and from whom?


(a) 1970, from Black minority rule (b) 1880, from White minority rule
(c) 1980, from Americans (d) 1980, from White minority rule

ज बा वे को वतं ता कब और कससे मली?


(ए) 1970, अ ेत अ पसं यक शासन से (बी) 1880, ेत अ पसं यक शासन से
(c) 1980, अमे र कय से (डी) 1980, ेत अ पसं यक शासन से

Q12.Which party has ruled Zimbabwe since its independence and who is its ruler?
(a) ZANU-PF, Robert Mugabe (b) ZANU-PF, Kenneth Kaunda
(c) Zimbabwe Freedom Party, Nelson Mandela (d) Zimbabwe Party, P Johnson

ज बा वे म आजाद के बाद से कस पाट ने शासन कया है और उसका शासक कौन है?


(ए) ज़ानू-पीएफ, रॉबट मुगाबे (बी) ज़ानू-पीएफ, के नेथ क डा
(सी) ज बा वे डम पाट , ने सन मंडेला (डी) ज बा वे पाट , पी जॉनसन
Q13. Which of these features is/are necessary to provide the basic rights to the
citizens?
(a) Citizens should be free to think (b) should be free to form associations
(c) Should be free to raise protest (d) All the above

नाग रक को मूल अ धकार दान करने के लए इनम से कौन सी वशेषताएँ आव यक ह/ह?


(ए) नाग रक को सोचने के लए वतं होना चा हए
(बी) संघ बनाने के लए वतं होना चा हए
(सी) वरोध करने के लए वतं होना चा हए
(डी) उपरो सभी
Q14. Which organ of the government is required to protect the rights of the citizens?
(a) Executive (b) Legislature (c) Independent judiciary (d) Police

नाग रक के अ धकार क र ा के लए सरकार के कस अंग क आव यकता है?


(ए) कायपा लका (बी) वधा यका (सी) वतं यायपा लका (डी) पु लस

Q15. What is Constitutional Law?


(a) Provisions given in the Constitution (b) Law to make Constitution
(c) Law to set up Constituent Assembly (d) none of the above
संवैधा नक कानून या है?
(ए) सं वधान म दए गए ावधान (बी) सं वधान बनाने के लए कानून
(सी) सं वधान सभा क ापना के लए कानून (डी) उपरो म से कोई नह

Q16.Which of these is permitted under the Rule of Law?


(a) Prime Minister can be punished for violating the Constitution
(b) Police has a right to kill anybody.
(c) Women can be paid lesser salaries
(d) President can rule for as long as he wants.

कानून के शासन के तहत इनम से कसक अनुम त है?


(ए) सं वधान का उ लंघन करने के लए धान मं ी को दं डत कया जा सकता है
(बी)पु लस को कसी को भी मारने का अ धकार है।
(सी) म हला को कम वेतन दया जा सकता है
(डी) रा प त जब तक चाह शासन कर सकते ह।

Q17. Some of the drawbacks of democracy is


(a) Instability and delays (b) corruption and hypocrisy
(c) Politicians fighting among themselves (d) all the above

लोकतं क कु छ क मयां ह:
(ए) अ रता और दे री (बी) ाचार और पाखंड
(सी) राजनेता आपस म लड़ रहे ह (डी) उपरो सभी

Q18. In which of these cases can democracy not provides a complete solution?
(a) Removing poverty completely (b) Providing education to all
(c) Giving jobs to all (d) All the above

इनम से कस मामले म लोकतं पूण समाधान दान नह कर सकता है?


(ए) गरीबी को पूरी तरह से हटाना (बी) सभी को श ा दान करना
(सी) सभी को नौकरी दे ना (डी) उपरो सभी

Q19. In which period did China face one of the worst famines that have occurred in the
world?
(a) 1932-36 (b) 1958-61 (c) 2001-2002 (d) 2004-2007

कस कालखंड म चीन को नया के सबसे भयानक अकाल म से एक का सामना करना पड़ा?


(ए) 1932-36 (बी) 1958-61 (सी) 2001-2002 (डी) 2004-2007

Q20. A democratic government is better than a non-democratic government because


(a) It may or may not be accountable (b) It always responds to the needs of the
people
(c) It is a more accountable form of government (d) None of the above

एक लोकतां क सरकार गैर-लोकतां क सरकार से बेहतर है य क


(ए) यह जवाबदे ह हो सकता है या नह
(बी) यह हमेशा लोग क ज रत का जवाब दे ता है
(सी) यह सरकार का एक अ धक जवाबदे ह प है
(डी) उपरो म से कोई नह

Q21. Democracy improves the quality of decision-making because


(a) Decisions are taken by educated people
(b) Decisions are taken by consultation and discussion
(c) Decisions are taken over a long period of time
(d) All decisions are approved by judiciary

लोकतं नणय लेने क गुणव ा म सुधार करता है य क


(ए) श त लोग ारा नणय लए जाते ह
(बी) नणय परामश और चचा ारा लए जाते ह
(सी) नणय लंबी अव ध म लए जाते ह
(डी) सभी नणय यायपा लका ारा अनुमो दत होते ह
Q22. How does democracy allows us to correct its own mistakes?
(a) Mistakes are hidden and cannot be corrected
(b) Re-electing the same government to enable it to correct its mistakes
(c) The rulers can be changed
(d) None of the above
लोकतं हम अपनी गल तय को सुधारने क अनुम त कै से दे ता है?
(ए) गल तयाँछपी ई ह और उ ह ठ क नह कया जा सकता है
(बी) अपनी गल तय को सुधारने म स म बनाने के लए उसी सरकार को फर से चुनना
(सी) शासक को बदला जा सकता है
(डी) उपरो म से कोई नह

Q23.The most common form that democracy takes in our time is that of
(a) Limited democracy (b) representative democracy
(c) Maximum democracy (d) none of the above

हमारे समय म लोकतं का सबसे सामा य प है


(ए) सी मत लोकतं (बी) त न ध लोकतं
(सी) अ धकतम लोकतं (डी) उपरो म से कोई नह

Q24.Which body in Indian political system is an example of direct democracy?


(a) Zila Parishad (b) Panchayat Samiti (c) Gram Sabha (d) Vidhan Sabha

भारतीय राजनी तक व ा म कौन सा नकाय य लोकतं का उदाहरण है?


(ए) जला प रषद (बी) पंचायत स म त (सी) ाम सभा (डी) वधानसभा

Q25.Which of these is an example of perfect democracy?


(a) USA (b) UK (c) India (d) None of the above

इनम से कौन पूण लोकतं का उदाहरण है?


(ए) यूएसए (बी) यूके (सी) भारत (डी) उपरो म से कोई नह

Q26.Which of these is not a feature of monarchy, dictatorship or one-party rule?


(a) Censorship of press (b) No opposition party or parties
(c) Citizens taking part in politics (d) One-man rule

इनम से कौन राजतं , तानाशाही या एकदलीय शासन क वशेषता नह है?


(ए) ेस क ससर शप (बी) कोई वप ी दल या दल नह
(सी) राजनी त म भाग लेने वाले नाग रक (डी) एक शासन

Q27.The most common form that democracy takes in our time is that of
(a) Unlimited democracy (b) Representative democracy
(c) Direct democracy (d) None of the above

सबसे सामा य प जो लोकतं हमारे समय म लेता है, वह है


(ए) असी मत लोकतं (बी) त न ध लोकतं
(सी) य लोकतं (डी) उपरो म से कोई नह

Q28. Which of these is an example of perfect democracy?


(a) USA (b) France
(c) India (d) None of the above

इनम से कौन पूण लोकतं का उदाहरण है?


(ए) यूएसए (बी) ांस
(सी) भारत (डी) उपरो म से कोई नह

Q29. Which body in Indian political system is an example of direct democracy?


(a) Municipal Corporation (b) Panchayat Samiti
(c) Gram Sabha (d) Legislative Assembly

भारतीय राजनी तक व ा म कौन सा नकाय य लोकतं का उदाहरण है?


(ए) नगर नगम (बी) पंचायत स म त
(सी) ाम सभा (डी) वधान सभा

Q30. A democratic government is better than a non-democratic government because


(a) It may and can respond to the people’s needs
(b) It fulfill all the wish of people
(c) It is a more accountable form of government
(d) None of the above

एक लोकतां क सरकार गैर-लोकतां क सरकार से बेहतर होती है य क


(ए) यह लोग क ज रत का जवाब दे सकता है और कर सकता है
(बी) यह लोग क सभी इ ा को पूरा करता है
(सी) यह सरकार का एक अ धक जवाबदे ह प है
(डी) उपरो म से कोई नह

Q31. The drawbacks of democracy are


(a) Instability and delays (b) Politicians fighting among themselves
(c) Corruption (d) All the above

लोकतं क क मयां ह
(ए) अ रता और दे री (बी) राजनेता आपस म लड़ रहे ह
(सी) ाचार (डी) उपरो सभी

Q32. Democracy must be based on


(a) One-party system (b) Free and fair election
(c) Choice from only the ruling party (d) All the above

लोकतं पर आधा रत होना चा हए


(ए) एक दल णाली (बी) वतं और न प चुनाव
(सी) के वल स ा ढ़ दल से वक प (डी) उपरो सभी
Q33. Which organ of the government is required to protect the rights of the citizens?
(a) Executive (b) Legislative
(c) Independent judiciary (d) Police

नाग रक के अ धकार क र ा के लए सरकार के कस अंग क आव यकता है?


(ए) कायकारी (बी) वधायी
(सी) वतं यायपा लका (डी) पु लस

Q34. Some of the drawbacks of Democracy is


(a) Instability and delays (b) Corruption and hypocrisy
(c) Politicians fighting among themselves (d) All the above

लोकतं क कु छ क मयां ह:
(ए) अ रता और दे री (बी) ाचार और पाखंड
(सी) राजनेता आपस म लड़ रहे ह (डी) उपरो सभी

Q35. Which of these is not a feature of monarchy, dictatorship or one -party rule?
(a) Censorship of press (b) No opposition party or parties
(c) Citizens taking part in politics (d) One-man rule

इनम से कौन राजतं , तानाशाही या एकदलीय शासन क वशेषता नह है?


(ए) ेस क ससर शप (बी) कोई वप ी दल या दल नह
(सी) राजनी त म भाग लेने वाले नाग रक (डी) एक शासन

MULTIPLE CHOICE QUESTIONS [1 MARK]

1 (b) 2 (c) 3 (a) 4 (c) 5 (d) 6 (b) 7 (b) 8 (d) 9 (c) 10 (b) 11 (d) 12 (a) 13 (d) 14 (c) 15 (a)
16 (a) 17 (d) 18 (d) 19 (b) 20 (c) 21(b) 22 (c) 23 (b) 24 (c) 25 (d) 26 (c) 27 (b) 28 (c)
29(c) 30 (c) 31 (d) 32(b) 33(c) 34(d) 35(c)

VERY SHORT QUESTIONS


Q1.The word democracy comes from Greek word.........?

० 1 लोकतं श द ीक श द ……… से आया है?

Q2. Who led a military coup in Pakistan in 1999?

० 2.1999 म पा क तान म सै य त तापलट का नेतृ व कसने कया?

Q3.Which party always won election in Mexico since its independence in 1930 until
2000?

० 3. 1930 म अपनी वतं ता के बाद से 2000 तक कस पाट ने हमेशा मे सको म चुनाव जीता?

Q4. When did Zimbabwe attain independence and from whom?


० 4. ज बा वे को वतं ता कब और कससे मली?

Q5. What is constitutional Law?


० 5. संवैधा नक कानून या है?

Q6. A democratic government is better than a non democratic government because......


०6. एक लोकतां क सरकार एक गैर-लोकतां क सरकार से बेहतर होती है य क...

Q7. How does democracy allows us to correct its own mistakes?


० 7. लोकतं हम अपनी गल तय को सुधारने क अनुम त कै से दे ता है?

Q8. Which body in Indian political system is an example of direct democracy?

० 8. भारतीय राजनी तक व ा म कौन सा नकाय य लोकतं का उदाहरण है?

Q9. How do people elect their representatives in China?


० 9. चीन म लोग अपने त न धय का चुनाव कै से करते ह?

Q10. After how many years does Mexico hold elections for electing the President?

०10. मे सको म रा प त के चुनाव के लए कतने वष बाद चुनाव होते ह?

II. Assertion and Reasons Questions


1. Assertion (A) : In India, people elect their own representatives.
Reason (R) : India is a democracy.

a) Both A and R are true and R is the correct explanation of A


b) Both A and R are true and R is not the correct explanation of A
c) A is true but R is false
d) A is false but R is true

अ भकथन (1): भारत म, लोग अपने त न धय का चुनाव करते ह।


कारण (2) : भारत एक लोकतं है।

क) 1 और 2 दोन सही ह और 2, 1 क सही ा या करता है


ख) 1 और 2 दोन सही ह और 2, 1 क सही ा या नह करता है
ग) 1 सच है ले कन 2 झूठा है
घ) 1 झूठा है ले कन 2 सच है

2. Assertion (A) : A referendum was held in Pakistan in the year 2002.


Reason (R) : Pervez Mushraf was granted five year extension as President

a) Both A and R are true and R is the correct explanation of A


b) Both A and R are true and R is not the correct explanation of A
c) A is true but R is false
d) A is false but R is true

अ भकथन (1):पा क तान म वष 2002 म एक जनमत सं ह आयो जत कया गया था।


कारण (2) : परवेज मुशरफ को रा प त के प म पांच साल का व तार दया गया था।

क) 1 और 2 दोन सही ह और 2, 1 क सही ा या करता है


ख) 1 और 2 दोन सही ह और 2, 1 क सही ा या नह करता है
ग) 1 सच है ले कन 2 झूठा है
घ) 1 झूठा है ले कन 2 सच है

3. Assertion (A) : In China elections don’t offer any choice.


Reason (R) : In Mexico the people seem to have a choice but practically they did
not have any choice.
a) Both A and R are true and R is the correct explanation of A
b) Both A and R are true and R is not the correct explanation of A
c) A is true but R is false
d) A is false but R is true

अ भकथन (1): चीन के चुनाव म कोई वक प नह होता है।


कारण (2) : मे सको म लोग के पास एक वक प लगता है ले कन ावहा रक प से
उनके पास कोई वक प नह था।
क) 1 और 2 दोन सही ह और 2, 1 क सही ा या करता है
ख) 1 और 2 दोन सही ह और 2, 1 क सही ा या नह करता है
ग) 1 सच है ले कन 2 झूठा है
घ) 1 झूठा है ले कन 2 सच है

4. Assertion (A) : Pakistan is considered as a democratic country.


Reason (R) : In Pakistan final powers rested with military officers

a) Both A and R are true and R is the correct explanation of A


b) Both A and R are true and R is not the correct explanation of A
c) A is true but R is false
d) A is false but R is true

अ भकथन (1): पा क तान को एक लोकतां क दे श माना जाता है।


कारण (2) : पा क तान म अं तम श याँ सै य अ धका रय के पास होती ह

क) 1 और 2 दोन सही ह और 2, 1 क सही ा या करता है


ख) 1 और 2 दोन सही ह और 2, 1 क सही ा या नह करता है
ग) 1 सच है ले कन 2 झूठा है
घ) 1 झूठा है ले कन 2 सच है

5. Assertion (A) : Since its independence in 1980 Robert Mugabe has been elected
as a President of Zimbabwe
Reason (R) : ZANU- PF led the struggle for independence

a) Both A and R are true and R is the correct explanation of A


b) Both A and R are true and R is not the correct explanation of A
c) A is true but R is false
d) A is false but R is true
e)
अ भकथन (1): 1980 म अपनी वतं ता के बाद से रॉबट मुगाबे को ज बा वे के
रा प त के प म चुना गया है
कारण (2) : ज़ानू-पीएफ ने वतं ता के लए संघष का नेतृ व कया
क) 1 और 2 दोन सही ह और 2, 1 क सही ा या करता है
ख) 1 और 2 दोन सही ह और 2, 1 क सही ा या नह करता है
ग) 1 सच है ले कन 2 झूठा है
घ) 1 झूठा है ले कन 2 सच है
6. Assertion (A) : Democracy enhances its dignity of the citizens.
Reason (R) : People are not the subject of ruler but are the rulers themselves.

a) Both A and R are true and R is the correct explanation of A


b) Both A and R are true and R is not the correct explanation of A
c) A is true but R is false
d) A is false but R is true

अ भकथन (1): लोकतं नाग रक क ग रमा को बढ़ाता है।


कारण (2) : लोग शासक क जा नह ब क वयं शासक होते ह।

क) 1 और 2 दोन सही ह और 2, 1 क सही ा या करता है


ख) 1 और 2 दोन सही ह और 2, 1 क सही ा या नह करता है
ग) 1 सच है ले कन 2 झूठा है
घ) 1 झूठा है ले कन 2 सच है
III. Case Studies Questions

I. Democracy is a form of government in which the rulers are elected by the


people. A referendum is a direct vote in which the electorate is asked to
either accept or reject a proposal.

In a democracy, before the elections, adequate leeway needs to be given for


normal political activity and opposition. The citizens should be free to think
and form opinions, to be able to express these opinions in public, form
associations, and take other political actions.

Four features of democracy as a form of government emerge on the basis of


examples:
1. The elected representatives of the people should have the final decision-making
power.
2. The foundation of a democracy is free and fair elections, in which the ruling party
also faces a fair chance of losing.
3. This choice and opportunity is available to all the people on an equal basis.
4. A democratic government rules within the limits set by constitutional law and
citizens’ rights.

Answer the following questions:


1. What is democracy?

2. What is referendum?

3. Name the country which have democratic form of government.

4. Give two features of democracy.

लोकतं सरकार का एक प है जसम शासक लोग ारा चुने जाते ह। जनमत सं ह एक य वोट है
जसम मतदाता को कसी ताव को वीकार या अ वीकार करने के लए कहा जाता है। लोकतं म,
चुनाव से पहले, सामा य राजनी तक ग त व ध और वरोध के लए पया त छू ट द जानी चा हए। नाग रक
को सोचने और राय बनाने, सावज नक प से इन वचार को करने, संघ बनाने और अ य
राजनी तक कारवाई करने म स म होने के लए वतं होना चा हए। सरकार के प म लोकतं क चार
वशेषताएं उदाहरण के आधार पर उभरती ह:जनता के चुने ए त न धय के पास अं तम नणय लेने क
श होनी चा हए। लोकतं क न व वतं और न प चुनाव है, जसम स ाधारी दल को भी हारने क
उ चत संभावना का सामना करना पड़ता है। यह वक प और अवसर सभी लोग को समान आधार पर
उपल है। एक लोकतां क सरकार संवैधा नक कानून और नाग रक के अ धकार ारा नधा रत
सीमा के भीतर शासन करती है।

न न ल खत सवाल का जवाब द:
1. लोकतं या है?
2. जनमत सं ह या है?
3. उस दे श का नाम बताइए जसम सरकार का लोकतां क व प है।
4. लोकतं क दो वशेषताएँ बताइए।

II. In China, elections are regularly held after every five years for electing the
country’s parliament,called Quanguo Renmin Daibiao Dahui (National People’s
Congress).The National People’s Congress has the power to appoint the
President of the country. It has nearly 3,000 members elected from all over
China. Some members are elected by the army. Before contesting elections, a
candidate needs the approval of the Chinese Communist Party. Only those
who are members of the Chinese Communist Party or eight smaller parties
allied to it were allowed to contest elections held in 2002-03. The government
is always formed by the Communist Party.

Answer the following questions:

1. After how many years elections will be held in China?


2. Name the parliament of China.
3. How many members were elected all over the China?
4. Who were allowed to contest elections held in 2002-03?
चीन म, दे श क संसद के चुनाव के लए हर पांच साल के बाद नय मत प से चुनाव होते ह, जसे
वांगुओ रेन मन दाइ बयाओ द ई (नेशनल पीपु स कां स े ) कहा जाता है। नेशनल पीपु स कां से के
पास दे श के रा प त को नयु करने क श है। इसके लगभग 3,000 सद य पूरे चीन से चुने गए ह।
कु छ सद य सेना ारा चुने जाते ह। चुनाव लड़ने से पहले, एक उ मीदवार को चीनी क यु न ट पाट क
मंजूरी क आव यकता होती है। के वल वे लोग जो चीनी क यु न ट पाट या उससे संब आठ छोट
पा टय के सद य ह, उ ह 2002-03 म ए चुनाव लड़ने क अनुम त द गई थी। सरकार हमेशा क यु न ट
पाट ारा बनाई जाती है।

न न ल खत सवाल का जवाब द:

1. चीन म कतने साल बाद चुनाव ह गे?


2. चीन क संसद का नाम बताइए।
3. पूरे चीन म कतने सद य चुने गए?
4. 2002-03 म ए चुनाव लड़ने क अनुम त कसे द गई थी?

III. In Pakistan, General Pervez Musharraf led a military coup in October 1999. He
overthrew a democratically elected government and declared himself the ‘Chief
Executive’ of the country. Later he changed his designation to President and in 2002
held a referendum in the country that granted him a five year extension. Pakistani media,
human rights organizations and democracy activists said that the referendum was
based on malpractices and fraud. In August 2002 he issued a ‘Legal Framework Order’
that amended the Constitution of Pakistan. According to this Order, the President can
dismiss the national and provincial assemblies. The work of the civilian cabinet is
supervised by a National Security Council which is dominated by military officers. After
passing this law, elections were held to the national and provincial assemblies. So
Pakistan has had elections, elected representatives have some powers. But the final
power rested with military officers and General Musharraf himself.

Answer the following questions:-

1. Who led the military coup in Pakistan during 1999?


2. In which year the Constitution of Pakistan was amended?
3. What is referendum?
4. After amendment, who has the final powers in Pakistan?

पा क तान म, जनरल परवेज मुशरफ ने अ टू बर 1999 म एक सै य त तापलट का नेतृ व कया। उ ह ने


लोकतां क प से चुनी गई सरकार को उखाड़ फका और खुद को दे श का 'मु य कायकारी' घो षत कया। बाद म
उ ह ने रा प त के लए अपना पद बदल दया और 2002 म दे श म एक जनमत सं ह कराया जसने उ ह पांच
साल का व तार दया। पा क तानी मी डया, मानवा धकार संगठन और लोकतं कायकता ने कहा क जनमत
सं ह कदाचार और धोखाधड़ी पर आधा रत था। अग त 2002 म उ ह ने एक 'कानूनी ढांचा आदे श' जारी कया
जसने पा क तान के सं वधान म संशोधन कया। इस आदे श के अनुसार रा प त रा ीय और ांतीय
वधानसभा को बखा त कर सकता है। नाग रक कै बनेट के काम क नगरानी एक रा ीय सुर ा प रषद करती
है जसम सै य अ धका रय का वच व होता है। इस कानून को पा रत करने के बाद, रा ीय और ांतीय
वधानसभा के चुनाव ए। इस लए पा क तान म चुनाव ए ह, नवा चत त न धय के पास कु छ श यां ह।
ले कन अं तम श सै य अ धका रय और वयं जनरल मुशरफ के पास थी।

न न ल खत सवाल का उ र द:-

1. 1999 के दौरान पा क तान म सै य त तापलट का नेतृ व कसने कया?

2. पा क तान के सं वधान म कस वष संशोधन कया गया था?

3. जनमत सं ह या है?

4. संशोधन के बाद, पा क तान म अं तम श यां कसके पास ह?

IV. This conversation has most of the arguments that we routinely hear
against democracy. Let us go over some of these arguments: Leaders keep
changing in a democracy. This leads to instability. Democracy is all about
political competition and power play. There is no scope for morality. So many
people have to be consulted in a democracy that it leads to delays. Elected
leaders do not know the best interest of the people. It leads to bad decisions.
Democracy leads to corruption for it is based on electoral competition.
Ordinary people don’t know what is good for them; they should not decide
anything.

Answer the following questions:-

1. Elected people know the best interest of the people.


a. True
b. False

2. Leaders keep changing in a democracy. This leads to


a. Stability
b. Delays
c. Instability
d. None of the above
3. Which of the following is not correct against democracy.
a. Leaders do not change
b. Democracy is all about political competition and power play.
c. Democracy leads to corruption
d. Democracy is a more accountable form of government

4. Ordinary people ____________know what is good for them.


a. Do not
b. Often
c. Definitely
d. None of above

5. How many people have to consulted in a democracy?


a. Few
b. So many
c. All
d. Not sure
e.

इस बातचीत म अ धकांश तक ह जो हम नय मत प से लोकतं के खलाफ सुनते ह। आइए इनम


से कु छ तक पर गौर कर: लोकतं म नेता बदलते रहते ह। इससे अ रता पैदा होती है। लोकतं
राजनी तक त धा और स ा के खेल के बारे म है। नै तकता क कोई गुज ं ाइश नह है। लोकतं म
इतने लोग से परामश करना पड़ता है क इससे दे री होती है। चुने ए नेता को लोग के सव म
हत का पता नह होता है। यह बुरे फै सल क ओर ले जाता है। लोकतं ाचार क ओर ले जाता है
य क यह चुनावी त धा पर आधा रत है। साधारण लोग नह जानते क उनके लए या अ ा है;
वे कु छ भी तय नह करना चा हक)
न न ल खत सवाल का जवाब द:-
1. नवा चत लोग जनता के सव म हत को जानते ह।
क) स य
ख) अस य
2. लोकतं म नेता बदलते रहते ह। इससे ये होता है
क) रता
ख) दे री
ग) अ रता
घ) इनमे से कोई भी नह

3. न न ल खत म से कौन लोकतं के व सही नह है।


क) नेता नही बदलते
ख) लोकतं राजनी तक त धा और स ा के खेल के बारे म है।
ग)लोकतं ाचार क ओर ले जाता है
घ) लोकतं सरकार का अ धक जवाबदे ह प है

4. आम लोग ____________ जानते ह क उनके लए या अ ा है।


क) नह
ख) अ सर
ग) न त प से
घ) इनम से कोई भी नह

5. लोकतं म कतने लोग से परामश करना पड़ता है?


क) कु छ
ख) ब त सारे
ग) सभी
घ) प का नह

V. A democratic government is a better government because it is a more


accountable form of government. Democracy is based on consultation and
discussion. Thus democracy improves the quality of decision-making.
Democracy provides a method to deal with differences and conflicts. In any
society people are bound to have differences of opinions and interests. In
democracy, no one is a permanent winner. No one is a permanent loser.
Different groups can live with one another peacefully. In a diverse country like
India, democracy keeps our country together. . Even if democracy does not
bring about better decisions and accountable government, it is still better
than other forms of government. Democracy enhances the dignity of citizens.

Answer the following questions:-

1. Which of the following is not correct for democracy?


a. People elect their rulers.
b. Accountable government
c. Elected leaders know the best interest of the people.
d. Democracy improves the quality of decision making

2. Which of the following a democratic country?


a. India
b. China
c. Pakistan
d. Saudi Arabia
e.

3. People elect their leaders in __________


a. Monarchy
b. Dictatorship
c. Democracy
d. None of these

4. Is there any permanent winner in democracy?


a. No
b. Yes
c. Not sure
d. None of above

एक लोकतां क सरकार एक बेहतर सरकार है य क यह सरकार का अ धक जवाबदे ह प है। लोकतं परामश


और चचा पर आधा रत है। इस कार लोकतं नणय लेने क गुणव ा म सुधार करता है। लोकतं मतभेद और
संघष से नपटने का एक तरीका दान करता है। कसी भी समाज म लोग के वचार और चय म मतभेद होना
लाजमी है। लोकतं म कोई भी ायी वजेता नह होता। कोई भी ायी हारने वाला नह है। व भ समूह एक
सरे के साथ शां त से रह सकते ह। भारत जैसे व वधतापूण दे श म लोकतं हमारे दे श को एक साथ रखता है। .
भले ही लोकतं बेहतर नणय और जवाबदे ह सरकार नह लाता है, फर भी यह सरकार के अ य प से बेहतर है।
लोकतं नाग रक क ग रमा को बढ़ाता है।

न न ल खत सवाल का जवाब द:-

1. न न ल खत म से कौन लोकतं के लए सही नह है?


क) जनता अपने शासक का चुनाव करती है ।
ख) जवाबदे ह सरकार
ग) चुने ए नेता लोग के सव म हत को जानते ह।
घ) लोकतं नणय लेने क गुणव ा म सुधार करता है

2. न न ल खत म से कौन एक लोकतां क दे श है?


क) इं डया
ख) चीन
ग) पा क तान
घ) सऊद अरब
3. लोग __________ म अपने नेता का चुनाव करते ह
क) सा ा य
ख) अ धनायक व
ग) लोकतं
घ) इनम से कोई नह

4. या लोकतं म कोई ायी वजेता होता है?


क) नह
ख) हाँ
ग) प का नह
घ) इनम से कोई भी नह

VI. The drafting of the document called the constitution was done by an assembly of
elected representatives called the Constituent Assembly. Elections to the Constituent
Assembly were held in July 1946. Its first meeting was held in December 1946. The
Constituent Assembly that wrote the Indian constitution had 299 members. The
Assembly adopted the Constitution on 26 November 1949 but it came into effect on 26
January 1950. To mark this day we celebrate January 26 as Republic Day every
year.First some basic principles were decided and agreed upon. Then a Drafting
Committee chaired by Dr. B.R. Ambedkar prepared a draft constitution for discussion.
Several rounds of thorough discussion took place on the Draft Constitution, clause by
clause. More than two thousand amendments were considered. Every document
presented and every word spoken in the Constituent Assembly has been recorded and
preserved. These are called ‘Constituent Assembly Debates’. When printed, these
debates are 12 bulky volumes! These debates provide the rationale behind every
provision of the Constitution. These are used to interpret the meaning of the
Constitution.

Answer the following questions:

Q1. How many members were there in a constituent Assembly?


a. 399 b. 299 c. 350 d. 498

Q2. When did the constitution came into effect?


a. 26 Jan 1949
b. 26 Nov 1949
c. 26 Jan 1950
d. 26 Nov 1950
Q3. Who chaired the drafting committee?

a. M.K.Gandhi
b. Jwahar Lal Nehru
c. Subhash Chander Boss
d. B.R.Ambedhkar
e.
Q4. How many volumes printed of the constitution?
a. 10
b. 15
c. 12
d. 20

सं वधान नामक द तावेज का ा पण नवा चत त न धय क एक सभा ारा कया जाता था जसे सं वधान
सभा कहा जाता है। सं वधान सभा के चुनाव जुलाई 1946 म ए थे। इसक पहली बैठक दसंबर 1946 म ई
थी। भारतीय सं वधान लखने वाली सं वधान सभा म 299 सद य थे। वधानसभा ने 26 नवंबर 1949 को
सं वधान को अपनाया ले कन यह 26 जनवरी 1950 को लागू आ। इस दन को च त करने के लए हम हर
साल 26 जनवरी को गणतं दवस के प म मनाते ह। पहले कु छ बु नयाद स ांत को तय कया गया और उन
पर सहम त बनी। फर डॉ. बी.आर. क अ य ता म एक मसौदा स म त। अ बेडकर ने चचा के लए सं वधान का
मसौदा तैयार कया। मसौदा सं वधान, खंड दर खंड पर कई दौर क गहन चचा ई। दो हजार से अ धक संशोधन
पर वचार कया गया। सं वधान सभा म तुत कए गए येक द तावेज और बोले गए येक श द को रकॉड
और संर त कया गया है। इ ह 'सं वधान सभा वाद- ववाद' कहा जाता है। मु त होने पर, ये बहस १२ भारी मा ा
म होती ह! ये बहस सं वधान के हर ावधान के पीछे तक दान करती ह। इनका उपयोग सं वधान के अथ क
ा या करने के लए कया जाता है।

न न ल खत सवाल का जवाब द:

० 1. एक सं वधान सभा म कतने सद य थे?

一) 399 ख) 299 ग) 350 घ) 498


० 2. संघटक कब लागू आ?
क) 26 जनवरी 1949 ख) 26 नवंबर 1949
ग) 26 जनवरी 1950 घ) 26 नवंबर 1950

० 3. मसौदा स म त क अ य ता कसने क ?
क) एम.के .गांधी

ख) जवाहर लाल नेह

ग) सुभाष चंदर बॉस

घ) बी.आर.अ बेडकर

० 4. सं वधान के कतने खंड छपे?


一) 10
二) 15
三) 12
四) 20

VERY SHORT ANSWERS

Ans.1. Democratia.
उ र 1 डेमो े टया

Ans2.. Pervez Musharraf

उ र2.- परवेज मुशरफ।

Ans3. Institutional Revolutionary Party.

उ र3. सं ागत ां तकारी पाट ।

Ans.4.1980, from White minority rule.

उ र4.1980, ेत अ पसं यक शासन से।

Ans.5. Provisions given in the constitution.

उ र5. सं वधान म दए गए ावधान।

Ans6.. It is more accountable form of government.

उ र यह सरकार का अ धक जवाबदे ह प है।

Ans7. The rules can be changed.

उ र7. नयम बदले जा सकते ह।

Ans8. Gram Sabha.

उ र8. ाम सभा।

Ans9. In China elections are held after every five years for electing the country’s
Parliament called National People’s Congress.

उ र9 चीन म दे श क संसद के चुनाव के लए हर पांच साल के बाद चुनाव होते ह जसे नेशनल पीपु स कां ेस
कहा जाता है |

Ans10. Since its independence in 1930, Mexico holds elections after every’ six years for
electing the President.

उ र 10. 1930 म अपनी वतं ता के बाद से, मे सको म रा प त के चुनाव के लए हर छह साल बाद चुनाव
होते ह |

Answers: 1. (a) 2. (a) 3.(b) 4. (d) 5. (b) 6. (a)

CASE STUDY QUESTIONS


Question 1

.Ans 1. Democracy is a form of government in which the rulers are elected by the
people.

2.Ans. A referendum is a direct vote in which the electorate is asked to either accept or
reject a proposal.
Ans3. India
Ans 4. The elected representatives of the people should have the final decision-making
power.

उ र 1. लोकतं सरकार का एक प है जसम लोग ारा शासक का चुनाव कया जाता है।

उ र2. जनमत सं ह एक य वोट है जसम मतदाता को या तो वीकार करने के लए कहा जाता है या एक


ताव को अ वीकार कर।

उ र3. इं डया

उ र 4. जनता के चुने ए त न धय के पास अं तम नणय लेने क श होनी चा हए।

. Question 2

Ans1.. After 5 years


Ans2. Quanguo Renmin Daibiao Dahui (National People’s Congress)

Ans3.. 3000

Ans4.. Only those who are members of the Chinese Communist Party or eight smaller
parties allied to it were allowed to contest elections held in 2002-03.

उ र1. 5 साल बाद

उ र 2. वांगओ
ु रेन मन दाइ बयाओ द ई (नेशनल पीपु स कां ेस)

उ र 3.3000
उ र 4. के वल वे लोग जो चीनी क यु न ट पाट या उससे संब आठ छोट पा टय के सद य ह, उ ह 2002-03
म ए चुनाव लड़ने क अनुम त द गई थी।

Question 3

Ans1. Pervez Musharraf.

Ans2. August 2002

Ans3. A referendum is a direct vote in which the electorate is asked to either accept or
reject a proposal

Ans4. Military Officers and General Pervez Musharraf himself.

उ र1- परवेज मुशरफ।

उ र2- अग त 2002

उ र3- जनमत सं ह एक य वोट है जसम मतदाता को या तो वीकार करने के लए कहा जाता है या एक


ताव को अ वीकार कर

उ र4- सै य अ धकारी और खुद जनरल परवेज मुशरफ।


Question 4

Ans1 -b-False
Ans2 -c -Instability
Ans3 - a Leaders do not change

Ans4-a-Do not

Ans4 - b- So many

उ र1- अस य
उ र2- अ रता
उ र3- नेता नही बदलते
उ र4- नह
उ र5- ब त सारे

Question 5
Ans1- c-Elected leaders know the best interest of the people
Ans2- a-India
Ans3- c-Democracy

Ans4-a- No

उ र1- चुने ए नेता लोग के सव म हत को जानते ह।

उ र2- इं डया
उ र3- लोकतं
उ र4- नह

Question 5

Answer1- b. 299

Answer2- c-26 Jan 1950

Answer3-d- B.R.Ambedhkar

Answer4-c-12

उ र 1- 299
उ र 2- 26 जनवरी 1950

उ र 3- घ) बी.आर.अ बेडकर
उ र 4- 12

CLASS –IXth
SUBJECT- SOCIAL SCIENCE
BOOK- POLITICAL SCIENCE
CHAPTER: CONSTITUTIONAL DESIGN
क ा-नौव
वषय- सामा जक व ान
पु तक- राजनी तक व ान
अ याय: संवैधा नक डजाइन
MULTIPLE CHOICE QUESTIONS:
ब वक पी :
Q.1. When was the constitution of India drafted at first?
A. 1930
B. 1931
C. 1928
D. 1929

1. भारत का सं वधान सबसे पहले कब बनाया गया था?


A. 1930
B. 1931
C. 1928
D. 1929

Q.2. Indian constitution adopted many institutional details and procedures from
colonial laws like the Government of India Act……?
A. 1935
B. 1934
C. 1931
D. 1930

2. भारतीय सं वधान ने भारत सरकार अ ध नयम …… जैसे औप नवे शक कानून से कई सं ागत ववरण
और या को अपनाया?
A. 1935
B. 1934
C. 1931
D. 1930

Q.3. The drafting of the document called the constitution was done by an assembly of
elected representatives called the Constituent Assembly.
A. Constitutional Assembly
B. Constituent Assembly
C. Electoral Assembly
D. None

3. सं वधान नामक द तावेज का ा पण नवा चत त न धय क एक सभा ारा कया जाता था जसे


सं वधान सभा कहा जाता है।
A.संवैधा नक सभा
B. सं वधान सभा
C. चुनावी सभा
D.कोई नह
Q.4. When were Elections to the Constituent Assembly held?
A. 1945
B. 1946
C. 1944
D. 1943

4. सं वधान सभा के चुनाव कब ए थे?


A. 1945
B. 1946
C. 1944
D.. 1943

Q.5. The Constituent Assembly that wrote the Indian constitution had ….. members.
A. 268
B. 244
C. 299
D. 276

5. भारतीय सं वधान लखने वाली सं वधान सभा म ….. सद य थे।


A. 268
B. 244
C. 299
D. 276

Q.6. Who chaired the Drafting Committee ?


A . Jawaharlal Nehru
B. G. Durgabai Deshmukh
C. Dr. B.R. Ambedkar
D. H. C. Mookherjee

6. मसौदा स म त क अ य ता कसने क ?
A. जवाहरलाल नेह
B. जी गाबाई दे शमुख
C. डॉ. बी.आर. अ बेडकर
D. एच सी मुखज

Q.7. Every document presented and every word spoken in the Constituent Assembly
has been recorded and preserved. These are called…….?
A. Constituent Assembly Debates
B. Constituent Assembly Minutes
C. Constituent Assembly Agenda
D. Constituent Assembly Daily
7. सं वधान सभा म तुत कए गए येक द तावेज और बोले गए येक श द को रकॉड और संर त
कया गया है। इ ह कहते ह…….?
A. सं वधान सभा वाद- ववाद
B. सं वधान सभा के कायवृ
C. सं वधान सभा एजडा
D. सं वधान सभा दै नक

Q.8. How many volumes were the recordings of the constituent assembly when
printed?
A. 6
B. 10
C. 12
D. 8

8. मु त होने पर सं वधान सभा क रकॉ डग कतने खंड म थी?


A. 6
B. 10
C. 12
D. 8

Q9. What is called the soul of the Indian Constitution?


A. Legislature
B. Judiciary
C. Executive
D. Preamble
9. भारतीय सं वधान क आ मा कसे कहा जाता है?
A. वधा यका
B. यायपा लका
C. कायकारी
D. तावना
Q.10. What is meant by Secular?
A. Citizens have complete freedom to follow any religion. But there is no official
religion. Government treats all religious beliefs and practices with equal respect
B. All are equal before the law. The traditional social inequalities have to be ended.
The government should ensure equal opportunity for all.
C. There are no unreasonable restrictions on the citizens in what they think, how
they wish to express their thoughts and the way they wish to follow up their thoughts in
action
D. People have the supreme right to make decisions on internal as well as
external matters. No external power can dictate the government of India

.10. से युलर से या ता पय है?


A. नाग रक को कसी भी धम का पालन करने क पूण वतं ता है। ले कन कोई आ धका रक धम
नह है। सरकार सभी धा मक व ास और था को समान स मान के साथ मानती है
B. कानून के सम सभी समान ह। पारंप रक सामा जक असमानता को समा त करना होगा। सरकार
को सभी के लए समान अवसर सु न त करना चा हए।
C. नाग रक पर कोई अनु चत तबंध नह है क वे या सोचते ह, वे अपने वचार को कै से करना
चाहते ह और जस तरह से वे अपने वचार का पालन करना चाहते ह।
D. लोग को आंत रक और बाहरी मामल पर नणय लेने का सव अ धकार है। कोई बाहरी श
भारत सरकार को म नह दे सकती
Q11. What is meant by Fraternity?
A. All of us should behave as if we are members of the same family. No one
should treat a fellow citizen as inferior.
B. All are equal before the law. The traditional social inequalities have to be
ended. The government should ensure equal opportunity for all.
C. A form of government where people enjoy equal political rights, elect their
rulers and hold them accountable. The government is run according to some
basic rules
D. There are no unreasonable restrictions on the citizens in what they think,
how they wish to express their thoughts and the way they wish to follow up their
thoughts in action.

11. बंधु व से या ता पय है?


A. हम सभी को ऐसा वहार करना चा हए जैसे हम एक ही प रवार के सद य ह। कसी को भी अपने साथी
नाग रक को हीन नह समझना चा हए।
B. कानून के सम सभी समान ह। पारंप रक सामा जक असमानता को समा त करना होगा। सरकार को
सभी के लए समान अवसर सु न त करना चा हए।
C. सी. सरकार का एक प जहां लोग समान राजनी तक अ धकार का आनंद लेते ह, अपने शासक का
चुनाव करते ह और उ ह जवाबदे ह ठहराते ह। सरकार कु छ बु नयाद नयम के अनुसार चलती है
D. नाग रक पर कोई अनु चत तबंध नह है क वे या सोचते ह, वे अपने वचार को कै से करना
चाहते ह और जस तरह से वे अपने वचार को कारवाई म पालन करना चाहते ह
Q12. A distinct section of a document is called …?
A. Phrase
B. Clause
C. Remark
D. Subsection

12. द तावेज़ के एक व श खंड को कहा जाता है ...?


A. कहावत
B. लॉज
C. ट पणी
D. उपखंड

Q.13.A preliminary version of a legal document is called ….?


A. Blueprint
B. Version
C. Draft
D. Plan

Q.13.एक कानूनी द तावेज के ारं भक सं करण को ….? कहा जाता है?


A. लू ट
B. सं करण
C. मसौदा
D. योजना

Q14. Who prepared the Constitution for India in 1928?


(a) Motilal Nehru
(b) B. R. Ambedkar
(c) Dr Rajendra Prasad
(d) Jawaharlal Nehru
14. 1928 म भारत के लए सं वधान कसने तैयार कया था?
A. मोतीलाल नेह
B. बी आर अंबेडकर
C. डॉ राज साद
D. जवाहरलाल नेह
Q.15. Mahatma Gandhi’s vision about the Indian Constitution was published in which
magazine?

(a) Discovery of India

(b) Young India

(c) New India

(d) Indian Gazette

.15. भारतीय सं वधान के बारे म महा मा गांधी का कोण कस प का म का शत आ था?


(A) ड कवरी ऑफ इं डया
(B) यंग इं डया
(C) यू इं डया
(D) भारतीय राजपत्
Q16. Which of these features were accepted by all the Indian leaders much before they
sat down to make the Constitution?

(a) Universal adult franchise

(b) Right to freedom

(C) Protection of the rights of minorities

(D) All of the above

16. सं वधान बनाने के लए बैठने से ब त पहले सभी भारतीय नेता ारा इनम से कौन सी वशेषता वीकार
क गई थी?
(A) सावभौ मक वय क मता धकार
(B) वतं ता का अ धकार
(C) अ पसं यक के अ धकार का संर ण
(D). उपरो सभी
Q17. Which of these inspired our leaders while framing the Constitution?

(A) Ideals of the French Revolution


(B) Parliamentary democracy in Britain

(C) Bill of Rights in US

(D) All of the above

17. इनम से कसने सं वधान बनाते समय हमारे नेता को े रत कया?


(A) ांसीसी ां त के आदश
(B) टे न म संसद य लोकतं
(C) यूएस म बल ऑफ राइट् स
(D). उपरो सभी
Q18. According to which Act were the elections held to provincial legislatures in India in
1937?

(A) Government of India Act, 1935

(B) Government of India Act, 1919

(C) Government of India Act, 1909

(D) None of the above

18. 1937 म भारत म ांतीय वधानसभा के चुनाव कस अ ध नयम के तहत ए थे?


(A) भारत सरकार अ ध नयम, 1935
(B) भारत सरकार अ ध नयम, 1919
(C) भारत सरकार अ ध नयम, 1909
(D) उपरो म से कोई नह

Q.19. Which revolution in the world inspired the Indians to set up a socialist economy?

(A) French Revolution

(B) Turkish Revolution

(C) Russian Revolution

(D) American War of Independence

.19. व म कस ां त ने भारतीय को समाजवाद अथ व ा ा पत करने के लए े रत कया?


(A) ांसीसी ां त
(B) तुक ां त
(C) सी ां त
(D) अमे रक वतं ता सं ाम
Q.20. The Constitution begins with a short statement of its basic values. What is it
called?

(A) Preface

(B) Preamble
(C) Introduction

(D) Article

.20. सं वधान क शु आत अपने बु नयाद मू य के सं त ववरण से होती है। इसे या कहते ह?


(A) तावना
(B) तावना
(C) प रचय
(D) लेख
Q21. When did the Assembly adopt the Constitution?

(A) 26 November, 1949

(B) 26 December, 1949

(C) 26 January, 1950

(D) 26 January, 1949

21. वधानसभा ने सं वधान को कब अपनाया?


(A) 26 नवंबर, 1949
(B) 26 दसंबर, 1949
(C) 26 जनवरी, 1950
(D) 26 जनवरी, 1949

Q22. Which of the following combinations is correct regarding the ideals of our
Constitution?

(A) Secular, Sovereign, Socialist, Democratic Republic

(B) Socialist, Sovereign, Secular, Democratic Republic

(C) Sovereign, Socialist, Secular, Democratic Republic

(D) Secular, Socialist, Sovereign, Democratic Republic

22. हमारे सं वधान के आदश के संबंध म न न ल खत म से कौन सा संयोजन सही है?


(A) धम नरपे , सं भु, समाजवाद , लोकतां क गणरा य
(B) समाजवाद , सं भु, धम नरपे , लोकतां क गणरा य
(C) सं भु, समाजवाद , धम नरपे , लोकतां क गणरा य
(D) धम नरपे , समाजवाद , सं भु, लोकतां क गणरा य

Q23. Where was the 1931 session of Indian National Congress held?

A. Nagpur (B) Karachi (C) Calcutta (D) Delhi

23. भारतीय रा ीय कां स


े का 1931 का अ धवेशन कहाँ आयो जत कया गया था?
A. नागपुर B. कराची C. कलक ा D. द ली

Q24. How many amendments were considered before adopting the Constitution?

A. Around 500 (B) Around 2000 (C) Around 1550 (D) Around 1000

२4. सं वधान को अपनाने से पहले कतने संशोधन पर वचार कया गया था?
A. लगभग 500
B. लगभग 2000
C. लगभग 1550
D. लगभग 1000
Q25. Who said the following?“I shall work for an India in which the poorest shall feel
that it is their country ….. InWhich all communities shall live in perfect harmony?”

A. Mahatma Gandhi (b) Jawaharlal Nehru

(c) Dr. B.R. Ambedkar (d) Sarojini Naidu

25. न न ल खत म से कसने कहा?

"म एक ऐसे भारत केलए काम क ं गा जसम सबसे गरीब महसूस करेगा क यह उनका दे श है ... .. जसम सभी
समुदाय पूण स ाव म रहगे?"

A. महा मा गांधी
B. जवाहरलाल नेह
C. डॉ बी.आर. अ बेडकर
D. सरो जनी नायडू
Q26. Match these guiding values with their meanings.

(A) Sovereign (i) government will not favour any particular


religion
(B) Republic (ii) People will have supreme right to make
decisions without outside Interference
(C) Fraternity (iii) Head of the state is an elected person
(D) Secular (IV) There should be a feeling of brotherhood
among
all the people
(A) . A-(iii), B-(i), C-(ii), D-(IV)

(B) A-(ii), B-(iii), C-(iv), D-(i)


(C) A-(i), B-(IV), C-(iii), D-(ii)

(D) A-(IV), B-(iii), C-(i), D-(ii)

26. इन मागदशक मू य को उनके अथ से मलाइए।

A. सं भु (i) सरकार कसी वशेष धम का प नह लेगी


B. गणतं (ii) लोग को बाहरी ह त पे के बना नणय लेने का सव अ धकार होगा
C. बंधु व (iii) रा य का मु खया एक नवा चत है
D. धम नरपे (IV) सभी लोग म भाईचारे क भावना होनी चा हए

(A) A-(iii), B-(i), C-(ii), D-(IV)


(B) A-(ii), B-(iii), C-(iv), D-(i)
(C) A-(i), B-(IV), C-(iii), D-(ii)
(D) A-(IV), B-(iii), C-(i), D-(ii)

Q27. what kind of ‘Justice’ does our Preamble provide?

A. Economic Justice (B) Political Justice

(C) Social Justice (D) All the above

27. हमारी तावना कस कार का ' याय' दान करती है?


A. आ थक याय
B. राजनी तक याय
C. सामा जक याय
D.उपरो सभी
Q28. which of the following days is celebrated to mark the enforcement of the
constitution?

(A) Republic Day (B) Independence Day

(C) Gandhi Jayanti (D) Constitution Enforcement Day

28. न न ल खत म से कौन सा दन सं वधान के वतन को च त करने के लए मनाया जाता है?


A.गणतं दवस
B. वतं ता दवस
C. गांधी जयंती
D.सं वधान वतन दवस
Q29. When did the Indian constitution come into force?

(A) 26th Nov, 1949 (B) 15th August, 1947

(C) 26th Jan, 1950 (D) 26th Jan, 1930

29. भारतीय सं वधान कब लागू आ?


A. 26 नवंबर, 1949
B. 15 अग त, 1947
C. 26 जनवरी, 1950
D.26 जनवरी, 1930

Q.30. Who amongst the following was not a member of the Constituent Assembly of
India?
A) Dr Rajendra Prasad
B) Dr B. R. Ambedkar
C) H. C. Mukherjee
D) Mahatma Gandhi

30. न न ल खत म से कौन भारत क सं वधान सभा का सद य नह था?


A) डॉ राज साद
B) डॉ बी आर अंबेडकर
C) एच. सी. मुखज
D) महा मा गांधी

Q31. Who was the President of the Indian Constituent Assembly?


A) Motilal Nehru
B) Dr Rajendra Prasad
C) Sarojini Naidu
D) B. R. Ambedkar

31. भारतीय सं वधान सभा के अ य कौन थे?


A) मोतीलाल नेह
B) डॉ राज साद
C) सरो जनी नायडू
D) बी आर अंबेडकर
Q32. The first captain of the National Hockey Team who was also the member of the
Constituent Assembly was?
A) Baldev Singh
B) Somnath Lahiri
C) Jaipal Singh
D) K. M. Munshi
32. रा ीय हॉक ट म के पहले क तान जो सं वधान सभा के सद य भी थे?
A) बलदे व सह
B) सोमनाथ ला हड़ी
C) जयपाल सह
D) के .एम. मुंशी
Q33. Match the following leaders with their roles in the making of the Constitution of
India:
1. (a) Motilal Nehru - (a) President of the Constituent Assembly
2. (b) B.R. Ambedkar - (b) Member of the Constituent Assembly
3. (c) Rajendra Prasad - (c) Chairman of the Drafting Committee
4. (d) Sarojini Naidu - (d) Prepared a Constitution for India in 1928

33. न न ल खत नेता को भारत के सं वधान के नमाण म उनक भू मका के साथ सुमे लत क जए:
1. (a) मोतीलाल नेह - (b) सं वधान सभा के अ य
2. (b) बी.आर. अ बेडकर - (b) सं वधान सभा के सद य
3. (c) राज साद - (c) मसौदा स म त के अ य
4. (d) सरो जनी नायडू - (d) 1928 म भारत के लए एक सं वधान तैयार कया

Q34. India is a secular state because


(A) there is no state religion
(B) every citizen has the right to adopt and practice any religion
(C) no discrimination can be made among citizens on the basis of religion
(D) all of the above

34. भारत एक धम नरपे रा य है य क


(A) कोई रा य धम नह है
(B) येक नाग रक को कसी भी धम को अपनाने और अ यास करने का अ धकार है
(C) धम के आधार पर नाग रक के बीच कोई भेदभाव नह कया जा सकता है
(D). उपरो सभी

Q35. Assertion: A constitution is the supreme law of the country.

Reason: constitution determines the relationship among people living in a territory


and the relationship between people and government.

A. Both A and R are true and R is the correct explanation of A.

B. Both A and R are true and R is not the correct explanation of A.

C. A is true but R is false.


D. A is false but R is true.

35. अ भकथन: एक सं वधान दे श का सव कानून है।


कारण: सं वधान एक े म रहने वाले लोग के बीच संबंध और लोग और सरकार के बीच संबध
ं को नधा रत
करता है।
A. A और R दोन स य ह और R, A क सही ा या करता है।
B. A और R दोन स य ह और R, A क सही ा या नह करता है।
C. Aसच है ले कन R झूठा है।
D. A गलत है ले कन R स य है।

Q.36. Assertion: Dr. B. R. Ambedkar was the chairman of the constituent assembly.

Reason: Constitution assembly was formed by the elections held in july 1946.

A. Both A and R are true and R is the correct explanation of A.

B. Both A and R are true and R is not the correct explanation of A.

C. A is true but R is false.

D. A is false but R is true.

36. अ भकथन: डॉ बी आर अंबडे कर सं वधान सभा के अ य थे।


कारण: जुलाई 1946 म ए चुनाव ारा सं वधान सभा का गठन कया गया था।
A. A और R दोन स य ह और R, A क सही ा या करता है।
B. A और R दोन स य ह और R, A क सही ा या नह करता है।
C. A सच है ले कन R गलत है।
D. A झूठा है ले कन R सच है।
Q.37.Assertion: Constitutional amendment is a change in constitution made by the
supreme legislative made by the supreme legislative body in a country.

Reason: To keep constitution updated, regular amendment is required.


A. Both A and R are true and R is the correct explanation of A.
B. Both A and R are true and R is not the correct explanation of A.
C. A is true but R is false.
D. A is false but R is true.
Q.37.अ भकथन: संवैधा नक संशोधन कसी दे श म सव वधायी नकाय ारा कए गए सव वधायी ारा
कए गए सं वधान म प रवतन है।
कारण: सं वधान को अ तन रखने के लए नय मत संशोधन आव यक है।
A A और R दोन स य ह और R, A क सही ा या करता है।
B. A और R दोन स य ह और R, A क सही ा या नह करता है।
C. A सच है ले कन R गलत है।
D. A झूठा है ले कन R सच है।
Q.38. Assertion: A political institutions of the colonial rule serve as a blueprint for the
setting up of Indian constitution.

Reason: The Indian constitution adopted many institutional details and


procedures from colonial laws like the Government of India Act 1935.

A. Both A and R are true and R is the correct explanation of A.

B. Both A and R are true and R is not the correct explanation of A.

C. A is true but R is false.

D. A is false but R is true.

38. अ भकथन:: औप नवे शक शासन क एक राजनी तक सं ा भारतीय सं वधान क ापना के


लए एक खाका के प म काम करती है।

कारण: भारतीय सं वधान ने भारत सरकार अ ध नयम 1935 जैसे औप नवे शक कानून से कई सं ागत
ववरण और या को अपनाया।
A. A और R दोन स य ह और R, A क सही ा या करता है।
B. A और R दोन स य ह और R, A क सही ा या नह करता है।
C. A सच है ले कन R गलत है।
D. A झूठा है ले कन आर सच है।
Q.39. What is considered as the supreme law of the country?

A. Judiciary
B. Parliament
C. Constitution
D. Secularism
39. दे श का सव कानून कसे माना जाता है?
A. यायपा लका
B. संसद
C. सं वधान
D. धम नरपे ता
Q.40. Which country inspired the world to start their constitution with the preamble?
A. France
B. Germany
C. America
D. India

.40. कस दे श ने नया को अपना सं वधान तावना के साथ शु करने के लए े रत कया?


A. ांस
B. जमनी
C. अमे रका
D. भारत

Q.41. Which of these is a provision that a democratic constitution does not have?
A. Powers of the head of the state
B Name of head of state
C. Power of legislature
D. Name of country

41. इनम से कौन सा ऐसा ावधान है जो एक लोकतां क सं वधान म नह है?


A. रा य के मु खया क श यां
B. रा य के मुख का नाम
C. वधा यका क श
D. दे श का नाम

Q.42. The Indian constitution is called as institutional because-

A. It contains all the values that institutions have for growth


B. Preamble does not describe its values
C. It was drafted in 1928
D. None of these
42. भारतीय सं वधान को सं ागत कहा जाता है य क-

A. इसम वे सभी मू य शा मल ह जो सं ान के वकास के लए ह


B. तावना इसके मू य का वणन नह करती है
C. इसे 1928 म तैयार कया गया था
D. इनम से कोई नह

Q.43. How much time was taken to complete the constitution of India?

A. 1 year 15 days
B. 2 Years 56 days

C. 2Years 14 month 17 days

D. 2years, 11 months and 18 days

43. भारत के सं वधान को बनने म कतना समय लगा?


A. 1 वष 15 दन
B. 2 साल 56 दन
C. 2वष 14 माह 17 दन
D. 2 साल, 11 महीने और 18 दन

Q.44. Assertion (A) : The Indian Constitution is the bulkiest Constitution in the world,
about five times the size of the Constitution of U.S.A.
Reason (R ) : The fathers of the Constitution wanted to include all good elements
from about four score Constitutions of the free world at that time and special clauses
have been provided for laying down the Fundamental Right, Directive Principles, rules
for elections, a list of national languages and details about the conduct of the
government.

A). Both A and R are true and R is the correct explanation of A


B). Both A and R are true and R is not the correct explanation of A
C). A is true but R is false
D). A is false bur R is true

44अ भकथन(A)भारतीय सं वधान नया का सबसे बड़ा सं वधान है, जो यू.एस.ए. के सं वधान के आकार
का लगभग पांच गुना है।
कारण (R) : सं वधान के जनक उस समय वतं व के लगभग चार अंक वाले सं वधान म से
सभी अ े त व को शा मल करना चाहते थे और मौ लक अ धकार, नदशक स ांत,
चुनाव के नयम, एक सूची नधा रत करने के लए वशेष खंड दान कए गए ह।
रा ीय भाषा और सरकार के आचरण के बारे म ववरण।
A. A और R दोन स य ह और R, A क सही ा या करता है
B A और R दोन स य ह और R, A क सही ा या नह करता है
C. A स य है ले कन R गलत है
D. Aझूठा है ले कन R सच है
Q.45.Assertion (A) : The Constitution has ensured the independence of the Comptroller and Auditor General.
Reason (R ) His salary is determined by the President.

A). Both A and R are true and R is the correct explanation of A


B). Both A and R are true and R is not the correct explanation of A
C). A is true but R is false
D). A is false bur R is true
45. दावा (ए): सं वधान ने नयं क और महालेखा परी क क वतं ता सु न त क है।
कारण (R) : उसका वेतन रा प त ारा नधा रत कया जाता है।
A. A और R दोन स य ह और R, A क सही ा या करता है
B. A और R दोन स य ह और R, A क सही ा या नह करता है
C. A स य है ले कन R गलत है
D. Aझूठा है ले कन R सच है

Q.46. . Assertion (A) : Equality before the law is not applicable to the President of India.
Reason (R ) : The President of India enjoys special powers and privileges under the Constitution.

A). Both A and R are true and R is the correct explanation of A


B). Both A and R are true and R is not the correct explanation of A
C). A is true but R is false
D). A is false bur R is true

46. . दावा (A): कानून के सम समानता भारत के रा प त पर लागू नह होती है।


कारण (R): भारत के रा प त को सं वधान के तहत वशेष श यां और वशेषा धकार ा त ह।
A. A और R दोन स य ह और R, A क सही ा या
करता है
B. Aऔर R दोन स य ह और R, A क सही ा या
नह करता है
C. A स य है ले कन R गलत है
D. Aझूठा है ले कन R सच है

Q.47. Assertion (A) : The Constitution of India is very flexible.


Reason (R ) : Since its inception, the Constitution has been amended 100 times.

A). Both A and R are true and R is the correct explanation of A


B). Both A and R are true and R is not the correct explanation of A
C). A is true but R is false
D). A is false bur R is true

47. अ भकथन (A) : भारत का सं वधान ब त लचीला है।


कारण (R) : इसक ापना के बाद से, सं वधान म 100 बार संशोधन कया गया है।
A. A और R दोन स य ह और R, A क सही ा या करता है
B. A और R दोन स य ह और R, A क सही ा या नह करता है
C. A स य है ले कन R गलत है
D. Aझूठा है ले कन R सच है
Q.48. . Assertion (A) : The Indian Constitution provides for single citizenship.
Reason (R ) : The Indian Constitution is unitary in nature.

A). Both A and R are true and R is the correct explanation of A


B). Both A and R are true and R is not the correct explanation of A
C). A is true but R is false
D). A is false bur R is true

48. . दावा (ए): भारतीय सं वधान एकल नाग रकता दान करता है।
कारण (R) : भारतीय सं वधान कृ त म एका मक है।
A. A और R दोन स य ह और R, A क सही ा या
करता है
B. A और R दोन स य ह और R, A क सही ा या
नह करता है
C. A स य है ले कन R गलत है
D. Aझूठा है ले कन R सच है

Q.49. Assertion (A) : The Indian Constitution came into force with effect from 26th January. 1950.
Reason (R) : 26th January is celebrated as the Republic Day.

A. Both A and R are true and R is the correct explanation of A.


B. Both A and R are true but R is NOT the correct explanation of A.
C. A is true but R is false.
D. A is false but R is true.
49. अ भकथन (A) : भारतीय सं वधान 26 जनवरी 1950.से लागू आ।
कारण (R) : 26 जनवरी को गणतं दवस के प म मनाया जाता है।
A. A और R दोन स य ह और R, A क सही ा या करता है।
B. A और R दोन स य ह ले कन R, A क सही ा या नह है
C. A सच है ले कन R झूठा है।
D A झूठा है ले कन R सच है।

Q.50. Why do we need constitution?


A. It generates degree of trust
B. I t specifies how the government will be constituted.
C. It lays down limits on the powers of the government.
D. All of the above
50. हम सं वधान क आव यकता य है?
A. यह व ास क ड ी उ प करता है
B. म न द नह करता क सरकार का गठन कै से कया जाएगा।
C. यह सरकार क श य क सीमा नधा रत करता है।
D. उपरो सभी

ANSWER KEY:
1 C 11 D 21 A 31 B 41 B
2 A 12 B 22 D 32 C 42 A
3 B 13 C 23 B 33 (1)a+d 43 D
(2)b+c
(3)c+a
(4)d+b
4 B 14 A 24 B 34 D 44 A
5 C 15 A 25 A 35 A 45 C
6 C 16 D 26 B 36 D 46 A
7 A 17 D 27 D 37 A 47 C
8 C 18 A 28 A 38 A 48 C
9 D 19 C 29 C 39 C 49 A
10 A 20 B 30 D 40 C 50 D
THE STORY OF VILLAGE PALAMPUR
MCQ’S
Q.1. Which of the following is fixed capital?
(a) Tools and machines (b) Fertilisers and pesticides
(c) Soil (d) Seeds
Q.2. Which of the following is a standard unit of measurement of land?
(a) Bigha (b) Hectare
(c) Acre (d) Guintha
Q.3. The minimum wages for a farm labourer set by the government is
(a) Rs. 50 (b) Rs. 60
(c) Rs. 70 (d) Rs. 80
Q.4. Money in hand is an example of
(a) Human capital (b) Fixed capital
(c) Working capital (d) Physical capital
Q.5. HYV seeds stands for
(a) Heavy yielding variety seeds (b) High yielding variety seeds
(c) Half yielding variety seeds (d) None
Q.6. What is the main production activity in Palampur village?
(a) Farming (b) Animal husbandry c) a and b d) None of these
Q.7. Which of the following is fixed capital?
(a) Tools and machines (b) Fertilisers and pesticides
(c) Soil (d) Seeds
Q.8. Which of the following is a standard unit of measurement of land?
(a) Bigha (b) Hectare
(c) Acre (d) Guintha
Q.9. The minimum wages for a farm labourer set by the government is
(a) Rs. 50 (b) Rs. 60
(c) Rs. 70 (d) Rs. 80
Q.10. Money in hand is an example of
(a) Human capital (b) Fixed capital
(c) Working capital (d) Physical capital
Q.11. HYV seeds stands for
(a) Heavy yielding variety seeds (b) High yielding variety seeds
(c) Half yielding variety seeds (d) None
Q.12. What is the main production activity in Palampur village?
(a) Farming (b) Animal husbandry
(c) Transport (d) Small-scale manufacturing
Q.13. Multiple cropping means growing
(a) only two crops (b) only three crops
(c) upto four crops (d) more than one crop
Q.14. Land under cultivation (in million hectares) in India in the year 2000 was
(a) 120 (b) 130
(c) 140 (d) 150
Q.15. Which area in India has a low level of irrigation?
(a) Deccan plateau (b) Coastal regions
(c) Riverine plains (d) Both (a) and (b)
Q.16. Modern farming methods were tried in India for the first time in
(a) Punjab ( b) Western U.P.
(c) Haryana (d) All the above
Q.17. Which of the following is a modern farming method?
(a) Multiple cropping (b) Use of HYV seeds
(c) Use of chemical fertilisers (d) Both (b) and (c)
Q.18. Production of pulses (in million tonnes) in India during 2000-01 was
(a) 10 (b) 11
(c) 14 (d) 12

Q.19. Which one is a natural resource?


(a) Labour (b) Raw materials
(c) Mineral (d) None of the above
Q.20. High yielding variety seeds (HYV) were introduced to Indian farmers as a result
of
(a) White Revolution (b) Green Revolution
(c) IT Revolution (d) None of the above
Q.21. Which Kharif crop is used for cattle feed?
(a) Sugarcane (b) Potato
(c) Jowar and bajra (d) Wheat
Q.22. The activities such as small manufacturing, transport, shopkeeping are referred
to as
(a) Non-economic activities (b) Non-farming activities
(c) Non-traditional activities (d) Non-market activities
Q.23. High Yielding Variety (HYV) seeds are developed in
(a) Research institutes (b) Factories
(c) Krishak Bharati Cooperatives (d) None of the above
Q.24. The concept of White Revolution is associated with
(a) food crops (b) milk
(c) cotton (d) pesticides
Q.25. Who is a person who puts together land, labour and capital?
(a) Moneylender (b) Entrepreneur
(c) Zamindar (d) Manager
Q.26. A farmer who works on a piece of 1 hectare of land is treated as
(a) Medium farmer (b) small farmer
(c) large farmer (d) none of the above
Q.27. Scope of farming activity is limited in Palampur due to
(a) Fixed amount of land (b) lack of irrigation
(c) lack of labour (d) none of the above
Q.28. What is done to surplus wheat in Palampur?
(a) Sold in the market (b) Destroyed
(c) Stocked by self (d) Given in charity
Q.29. Consumption of chemical fertilisers is highest in which state of India?
(a) Punjab (b) Haryana (c) Rajasthan (d) Himachal Pardesh

Q.30. People of Palampur sell milk in the near by large village named:
(a) Pitampura (b) Siliguri
(c) Shahpur (d) Raiganj
Q.31. Out of the total cultivated areas in the country, how much area is irrigated today :
(a) less than 40% (b) less than 30%
(c) less than 60% (d) less than 70%
Q.32. ‘Operation Flood’ is related to :
(a) control flood (b) produce fish
(c) milk production (d) grain production
Q.33. Green Revolution is related to :
(a) Milk Production (b) Grain production
(c) Fish production (d) none of these
Q.34. Where do most of the small farmers borrow money to arrange for the capital in
Palampur?
(a) Banks (b) Co-operative Societies
(c) Village money lenders (d) Friends and relatives
Q.35. Which one among the following is not fixed capital?
(a) Machines (b) Buildings
(c) Tools (d) Raw materials
Q.36. Why do the farmers of Palampur follow multiple cropping? Choose the correct
answer.
(a) Because the water consumption is less in this method
(b) Because this method consumes less chemical fertilisers
(c) Because this method doesn’t require fertile soils
(d) Because this method is the most common way of increasing production
Q.37. Which of the following transformed the system of irrigation in Palampur?
(a) Tubewells (b) Persian wheel
(c) Rainwater harvesting (d) None of these

Q.38. How many families lives in Village Palampur?


(a) 150 (b) 250
(c) 350 (d) 450
Q.39. Which one among the following is a non-farm activity?
(a) Multiple croppping (b) Crop rotation
(c) Dairy farming (d) Modern farming
Q.40. Which one of the following is not an effect of the modern farming?
(a) Soil degradation (b) Deforestation
(c) Decrease in groundwater (d) Water pollution
Q.41. Marginal farmers are those :
(a) who use modern methods for farming
(b) who practice crop rotation for farming
(c) who did not have sufficient land for farming
(d) who use modern methods of irrigation
Q.42. Working capital stands for :
(a) Tools, machines and buildings
(b) raw materials and money in hand
(c) total share capital
(d) fixed deposits in financial institutions
Q.43. Which is the most abundant factor of production in India?
(a) Land (b) Capital
(c) Labour (d) Tools and machines
Q.44. Multiple Cropping refers to :
(a) Cultivation of wheat and rice
(b) cultivation of two crops in alternate rows
(c) cultivating more than one crop on the same field each year
(d) cultivating crops and rearing animals on the same farm
1. न न ल खत म से कौन सी र पूंजी है?

(ए) उपकरण और मशीन (बी) उवरक और क टनाशक

(सी) म (डी) बीज


2. न न ल खत म से कौन भू म के मापन क मानक इकाई है?
(ए) बीघा (बी) हे टे यर

(सी) एकड़ (डी) ग ा

3.एक खे तहर मज र के लए सरकार ारा नधा रत यूनतम मज री है

(ए) पये 50 (बी) पये 60

(सी) पये 70 (डी) पये 80

4. हाथ म पैसा इसका उदाहरण है

(ए) मानव पूंजी (बी) न त पूंजी

(सी) कायशील पूंजी (डी) भौ तक पूंजी

5 HYV . का पूण प या है

(ए) भारी उपज क म के बीज (बी) उ उपज क म के बीज

(सी) आधा उपज क म के बीज (डी) कोई नह

6पालमपुर गांव म मु य उ पादन ग त व ध या है?

(ए) खेती (बी) पशुपालन


7न न ल खत म से कौन अचल पूंजी है?
(ए) उपकरण और मशीन (बी) उवरक और क टनाशक

(सी) म (डी) बीज

8 न न ल खत म से कौन भू म के मापन क मानक इकाई है?

(ए) बीघा (बी) हे टे यर

(सी) एकड़ (डी) ग ा

.9 एक खे तहर मज र के लए सरकार ारा नधा रत यूनतम मज री है

(ए) पये 50 (बी) पये 60

(सी) पये 70 (डी) पये 80

.10 हाथ म पैसा इसका उदाहरण है

(ए) मानव पूंजी (बी) न त पूंजी

(सी) कायशील पूंजी (डी) भौ तक पूंजी

11.HYV का पूण प या है

(ए) भारी उपज क म के बीज (बी) उ उपज क म के बीज

(सी) आधा उपज क म के बीज (डी) कोई नह

12. पालमपुर गांव म मु य उ पादन ग त व ध या है?

(ए) खेती (बी) पशुपालन

(सी) प रवहन (डी) छोटे पैमाने पर व नमाण

13. एका धक फसल का अथ है बढ़ना

(ए) के वल दो फसल (बी) के वल तीन फसल


(सी) चार फसल तक (डी) एक से अ धक फसल

14. वष 2000 म भारत म खेती यो य भू म ( म लयन हे टे यर म) थी

(ए) 120 (बी) 130

(सी) 140 (डी) 150

.15. भारत के कस े म सचाई का तर कम है?

(ए) द कन पठार (बी) तट य े

(सी) नद के मैदान (डी) दोन (ए) और (बी)

16. भारत म पहली बार आधु नक कृ ष प तय का योग कया गया

(ए) पंजाब ( बी) प मी यूपी

(सी) ह रयाणा (डी) उपरो सभी

17. न न ल खत म से कौन एक आधु नक कृ ष प त है?

(ए) एका धक फसल (बी) एचवाईवी बीज का उपयोग

(सी) रासाय नक उवरक का उपयोग (डी) दोन (बी) और (सी)

18. 2000-01 के दौरान भारत म दाल का उ पादन ( म लयन टन म) था

(ए) 10 (बी) 11

(सी) 14 (डी) 12

.19. ाकृ तक संसाधन कौन सा है?


(ए) म (बी) क ा माल

(सी) ख नज (डी) उपरो म से कोई नह

.20. भारतीय कसान को उ उपज दे ने वाले क म के बीज (HYV) कसके प रणाम व प पेश कए गए?

(ए) ेत ां त (बी) ह रत ां त

(सी) आईट ां त (डी) उपरो म से कोई नह

21. मवे शय के चारे के लए कस खरीफ फसल का उपयोग कया जाता है?

(ए) ग ा (बी) आलू

(सी) वार और बाजरा (डी) गे ं

22. लघु नमाण, प रवहन, कानदारी जैसी ग त व धय को कहा जाता है

(ए) गैर-आ थक ग त व धयां (बी) गैर-कृ ष ग त व धयां

(सी) गैर-पारंप रक ग त व धयां (डी) गैर-बाजार ग त व धयां

23. उ उपज दे ने वाली क म (HYV) के बीज को वक सत कया जाता है

(ए) अनुसंधान सं ान (बी) कारखाने

(सी) कृ षक भारती सहकारी स म तयां (डी) उपरो म से कोई नह

24. ेत ां त क अवधारणा संबं धत है

(ए) खा फसल (बी) ध

(सी) कपास (डी) क टनाशक

.25. भू म, म और पूंजी को एक साथ रखने वाला कौन है?


(ए) सा कार (बी) उ मी

(सी) जम दार (डी) बंधक

26. एक कसान जो 1 हे टे यर भू म के टु कड़े पर काम करता है उसे माना जाता है

(ए) म यम कसान (बी) छोटे कसान

(सी) बड़े कसान (डी) उपरो म से कोई नह

27 . पालमपुर म कृ ष ग त व ध का दायरा सी मत होने के कारण

(ए) भू म क न त मा ा (बी) सचाई क कमी

(सी) म क कमी (डी) उपरो म से कोई नह

28. पालमपुर म अ धशेष गे ं के लए या कया जाता है?

(ए) बाजार म बेचा (बी) न

(सी) वयं ारा टॉक (डी) दान म दया गया

29. भारत के कस रा य म रासाय नक उवरक क खपत सबसे अ धक है?

(ए) पंजाब (बी) ह रयाणा (सी) राज ान (डी) हमाचल परदे श

30. पालमपुर के लोग पास के बड़े गांव म ध बेचते ह जसका नाम है:

(ए) पीतमपुरा (बी) सलीगुड़ी

(सी) शाहपुर (डी) रायगंज

31. दे श म कु ल कृ ष यो य े म से आज कतना े फल स चत है :

(ए) 40% से कम (बी) 30% से कम


(सी) 60% से कम (डी) 70% से कम

32. 'ऑपरेशन लड' संबं धत है :

(ए) बाढ़ को नयं त कर (बी) मछली का उ पादन कर

(सी) ध उ पादन (डी) अनाज उ पादन

33 ह रत ां त का संबध
ं है :

(ए) ध उ पादन (बी) अनाज उ पादन

(सी) मछली उ पादन (डी) इनम से कोई नह

34. पालमपुर म राजधानी क व ा करने के लए अ धकांश छोटे कसान पैसे कहाँ से उधार लेते ह?

(ए) बक (बी) सहकारी स म तयां

(सी) गांव के सा कार (डी) दो त और र तेदार

35. न न ल खत म से कौन र पूंजी नह है?


(ए) मशीन (बी) इमारत
(सी) उपकरण (डी) क े माल
36. पालमपुर के कसान ब -फसल का पालन य करते ह? सही उ र का चयन कर।

(ए) य क इस व ध म पानी क खपत कम होती है

(बी) य क इस व ध म कम रासाय नक उवरक क खपत होती है

(सी) य क इस व ध म उपजाऊ म क आव यकता नह होती है

(डी) य क यह व ध उ पादन बढ़ाने का सबसे आम तरीका है

37. न न ल खत म से कसने पालमपुर म सचाई णाली को बदल दया?

(ए) बू वेल (बी) फारसी प हया


(सी) वषा जल संचयन (डी) इनम से कोई नह

38. पालमपुर ाम म कतने प रवार रहते ह ?

(ए) 150 (बी) 250

(सी) 350 (डी) 450

39. न न ल खत म से कौन-सा एक गैर-कृ ष ग त व ध है?

(ए) एका धक फसल (बी) फसल रोटे शन

(सी) डेयरी फा मग (डी) आधु नक खेती

.40. न न ल खत म से कौन-सा एक आधु नक कृ ष का भाव नह है?

(ए) मृदा रण (बी) वन क कटाई

(सी) भूजल म कमी (डी) जल षण

41. सीमांत कसान वे ह:

(ए) जो खेती के लए आधु नक तरीक का इ तेमाल करते ह

(बी) जो खेती के लए फसल च का अ यास करते ह

(सी) जनके पास खेती के लए पया त जमीन नह थी

(डी) जो सचाई के आधु नक तरीक का उपयोग करते ह

42. कायशील पूंजी का अथ है:


(ए) उपकरण, मशीन और भवन
(बी) क े माल और हाथ म पैसा
(सी) कु ल शेयर पूंजी
(डी) व ीय सं ान म साव ध जमा

43. भारत म उ पादन का सबसे चुर कारक कौन सा है?

(ए) भू म (बी) पूंजी


(सी) म (डी) उपकरण और मशीन

44. ब फसल का ता पय है :

(ए) गे ं और चावल क खेती

(बी) वैक पक पं य म दो फसल क खेती

(c) येक वष एक ही खेत म एक से अ धक फसल उगाना

(डी) एक ही खेत म फसल क खेती और जानवर का पालन करना

(सी) चरखी णाली (डी) छड़काव णाली

THE STORY OF VILLAGE PALAMPUR


ANSWER KEY
1. ANSWER:A
2. ANSWER:B
3. ANSWER: 60 RUPEES
4. ANSWER:C
5. ANSWER: B
6. ANSWER:A
7. ANSWER: A
8. ANSWER:B
9. ANSWER: Rs. 60
10. ANSWER: C
11.ANSWER : B
12.ANSWER : A
13.ANSWER: D
14.ANSWER: C
15.ANSWER: A
16.ANSWER :D
17.ANSWER : D
18.ANSWER: B
19.ANSWER:C
20.ANSWER:B

21.ANSWER: C
22.ANSWER:B
23.ANSWER:A
24.ANSWER:B
25.ANSWER: B
26.ANSWER:B
27.ANSWER: A
28.ANSWER:A
29.ANSWER:A
30.ANSWER:D
31.ANSWER:A
32.ANSWER:C
33.ANSWER:B
34.ANSWER:C
35.ANSWER:D
36.ANSWER:D
37.ANSWER: A
38.ANSWER:D
39.ANSWER:C
40.ANSWER:B
41.ANSWER:C
42.ANSWER:B
43.ANSWER:C
44.ANSWER:C

45. Classify the major crops based on the time they are sown in the field to explain
the months in which Kharif and Rabi crop are cultivated

A group of students went on two school trips to a field in the same year. They observed
the different crops on each trip as shown
FIELD TRIP SEASON CROP
1 JUNE SOYA BEAN,GROUNDNUT
2 NOVEMBER MUSTARD,PEA

Why did they observe different crops?


(a)Soyabean and groundnut are kharif crops which are sown in rainy season while
mustard and pea are rabi crops that are sown in winter season.(b)Soyabean and
groundnut are kharif crops which are sown in winter season while mustard and pea are
rabi crops that are sown in rainy season.(c)Soyabean and groundnut are rabi crops
which are sown in rainy season while mustard and pea are kharif crops that are sown in
winter season.(d)Soyabean and groundnut are rabi crops which are sown in winter
season while mustard and pea are kharif crops that are sown in rainy season.
Correct Answer:Option (a)
46 ) The table lists some climatic data of an area
HEAVY RAINFALL
HIGH HUMIDITY
HIGH AVERAGE TEMPERATURE

Which crop can be sown in such conditions?


(a)Gram (b)Maize (c)Wheat (d)Mustard
Correct Answer:Option (b)
47. Sequence the tasks involved in cultivating the crop to list major steps of agricultural
practices
1) The table lists some practices for cultivation of crops.
A. PROTECTION FROM WEEDS
B. SOWING
C. HARVESTING
D. PREPARATION OF SOIL
E.IRRIGATION
F. STORAGE
G. ADDING MANURE AND FERTILISERS

What is the correct sequence of these agricultural practices?


(a) A→D→F→G→B→C→E (b) G→B→A→C→F→E→D
(c) D→B→G→E→A→C→F (d) B→D→E→G→C→F→A
Correct Answer: Option (c)
48. Compare the advantages of three major tools used for tilling and ploughing to justify
the variety of agricultural practices
1) Raju and Sonu both grew same crop in their fields. Raju used plough while Sonu
used hoe to increase the productivity. After few months, it was observed that both the
fields had same productivity despite using different tools.
What is the likely reason for this?
(a) both the tools are made of wood (b) both the tools are used to harvest crops
(c) both the tools are helpful for tilling the soil (d) both the tools are useful in removing
weeds Correct Answer: Option (c)
49. Compare the advantages of two major tools used for sowing to justify the variety of
agricultural practices used in the country

What is the likely advantage of using seed drill over a traditional tool?
(a) It adds nutrients in the seed.
(b)It protects the seeds from physical damage.
(c) It separates healthy seeds from damaged seeds.
(d)It sow seeds at equal distance from each other.
Correct Answer: Option (d)
50. 2) A farmer wishes to use a tool for sowing seeds that protects the seed from being
consumed by animals.
Which type of tool should the farmer choose?
(a) seed drill as it coats a chemical above the seeds
(b)traditional tool as many seeds are sown together
(c) seed drill as it covers the seeds with soil after sowing
(d)traditional tool as seeds passes through a funnel-shape tube
Correct Answer: Option (c)
51. : Describe the process of crop rotation to explain ways in which nutrients in soil is
replenished
1) Ram wishes to restore the nutrients in the soil of his field without spending money
on chemical fertilizers. Which natural method he should likely adopt for replenishment
of soil nutrients?
(a) sowing seeds that are healthy (b) growing the same crops every year
(c) growing different crops alternatively (d) supplying water to crops at regular
interval 16
Correct Answer: Option (c)
52. Compare and analyse the traditional and modern methods of irrigation based on
cost and efficiency in order to predict suitable irrigation method in real life situations

1. Why a farmer should adopt sprinkler system than pulley system?


(a) Sprinkler system reduces wastage of water.
(b) Sprinkler system decreases water requirement of the crops.
(c) Sprinkler system increases water absorption capacity of the crops.
(d) Sprinkler system provides excess water to the crops for high productivity.
Correct Answer: Option (a)
53 .Gita grows crop in a region of Rajasthan where there are no lakes and very little
rainfall. During summers, the low availability of water limits the crop production.
Which irrigation method Gita should adopt to use water economically?
(a) drip system (b)lever system
(c) pulley system (d) sprinkler system
54. Differentiate agricultural activities and non-agricultural activities, and write them

separately.
What is the example of Farming activities?
Dairy b . Transport c. Shopkeeping d. growing crops
Correct answer : d. Growing crops.
What is the example of non farming activities ?
Dairy b. Tranport c .both a and b d. non of these
Correct answer : c. both a and b
55.
1.Name the unit which is used in Palampur ?
a. Hectare b. Acre c. both a and b d. None of these
Correct answer : a. Hectare

45 . खरीफ और रबी फसल क खेती के महीन क ा या करने के लए मुख फसल को खेत म बोए जाने के
समय के आधार पर वग कृ त कर
छा का एक समूह एक ही वष म दो कू ल मण पर एक े म गया। उ ह ने येक या ा पर व भ फसल का
अवलोकन कया जैसा क दखाया गया है
फ प सीजन फसल
1 जून सोयाबीन,मूंगफली
2 नवंबर सरस , मटर

उ ह ने व भ फसल का अवलोकन य कया?


(ए) सोयाबीन और मूंगफली खरीफ फसल ह जो बा रश के मौसम म बोई जाती ह जब क सरस और मटर रबी
फसल होती ह जो स दय के मौसम म बोई जाती ह।
(बी) सोयाबीन और मूंगफली खरीफ फसल होती ह जो स दय के मौसम म बोई जाती ह जब क सरस और मटर
रबी होती ह फसल जो बरसात के मौसम म बोई जाती ह।
(सी) सोयाबीन और मूंगफली रबी फसल ह जो बा रश के मौसम म बोई जाती ह जब क सरस और मटर खरीफ
फसल होती ह जो स दय के मौसम म बोई जाती ह।
(डी) सोयाबीन और मूंगफली रबी फसल होती ह
ज ह बोया जाता है सद का मौसम है जब क सरस और मटर
खरीफ क फसल ह जो बरसात के मौसम म बोई जाती ह।
सही उ र: वक प (ए)
46) ता लका कसी े के कु छ जलवायु डेटा को सूचीब करती है
भारी वषा

उ आ ता

उ औसत तापमान

ऐसी त म कौन सी फसल बोई जा सकती है?


(ए) चना (बी) चावल (सी) गे ं (डी) सरस
सही उ र: वक प (बी)
47. कृ ष प तय के मुख चरण को सूचीब करने के लए फसल क खेती म शा मल काय को अनु मत कर
1) ता लका फसल क खेती के लए कु छ था को सूचीब करती है।
A. खरपतवार से सुर ा
B. बुवाई
C. कटाई
D. म क तैयारी
E. सचाई
F.भंडारण
G. खाद और उवरक जोड़ना

इन कृ ष प तय का सही म या है?
(ए) ए → डी → एफ → जी → बी → सी → ई (बी) जी → बी → ए → सी → एफ → ई → डी
(सी) डी → बी → जी → ई → ए → सी → एफ (डी) बी → डी → ई → जी → सी → एफ → ए
सही उ र: वक प (सी)
48. कृ
ष प तय क व वधता को सही ठहराने के लए जुताई और जुताई के लए उपयोग कए जाने वाले तीन
मुख औजार के लाभ क तुलना कर
1) राजू और सोनू दोनने अपने खेत म एक ही फसल उगाई। राजू ने हल का इ तेमाल कया जब क सोनू ने
उ पादकता बढ़ाने के लए कु दाल का इ तेमाल कया। कु छ महीन के बाद, यह दे खा गया क व भ उपकरण का
उपयोग करने के बावजूद दोन े म समान उ पादकता थी।
इसका संभा वत कारण या है?
(ए) दोन उपकरण लकड़ी के बने होते ह (बी) दोन उपकरण फसल क कटाई के लए उपयोग कए जाते ह
(सी) दोन उपकरण म क जुताई के लए सहायक ह (डी) दोन उपकरण खरपतवार हटाने म उपयोगी ह
सही उ र: वक प (सी)
49. दे श म उपयोग कजाने वाली कृ ष प तय क व वधता को सही ठहराने के लए बुवाई के लए उपयोग
कए जाने वाले दो मुख उपकरण के लाभ क तुलना कर
पारंप रक उपकरण क तुलना म सीड ल का उपयोग करने का संभा वत लाभ या है?
(ए) यह बीज म पोषक त व जोड़ता है।
(बी) यह बीज को शारी रक त से बचाता है।
(c) यह व बीज को त त बीज से अलग करता है।
(d) यह एक सरे से समान री पर बीज बोता है।
सही उ र: वक प (डी)
50. 2) एक कसान बीज बोने के लए एक उपकरण का उपयोग करना चाहता है जो बीज को जानवर ारा खाए
जाने से बचाता है।
कसान को कस कार का उपकरण चुनना चा हए?
(ए) बीज ल य क यह बीज के ऊपर एक रसायन को कोट करता है
(बी) पारंप रक उपकरण जतने बीज एक साथ बोए जाते ह
(सी) बीज ल य क यह बुवाई के बाद बीज को म से ढक दे ता है
(डी) पारंप रक उपकरण के प म बीज एक क प के आकार क बू के मा यम से गुजरता है
सही उ र: वक प (सी)
51. : मृदा म पोषक त व क पू त के तरीक क ा या करने के लए फसल च क या का वणन कर
1) राम रासाय नक उवरक
पर पैसा खच कए बना अपने खेत क म म पोषक त व को बहाल करना चाहते
ह। म के पोषक त व क पू त के लए उसे कौन-सी ाकृ तक व ध अपनानी चा हए?
(ए) व बीज बोना (बी) हर साल एक ही फसल उगाना
(सी) वैक पक प से व भ फसल को उगाना (डी) नय मत अंतराल पर फसल को पानी क आपू त करना 1
सही उ र: वक प (सी)
52. वा
त वक जीवन तय म उपयु सचाई प त क भ व यवाणी करने के लए लागत और द ता के
आधार पर सचाई के पारंप रक और आधु नक तरीक क तुलना और व षे ण कर।
1. एक कसान को चरखी णाली क तुलना म कलर णाली य अपनानी चा हए?
(ए) छड़काव णाली पानी क बबाद को कम करती है।
(बी) छड़काव णाली फसल क पानी क आव यकता को कम करती है।
(सी) छड़काव णाली फसल क जल अवशोषण मता को बढ़ाती है।
(डी) छड़काव णाली उ उ पादकता के लए फसल को अ त र पानी दान करती है।
सही उ र: वक प (ए)
53. गीता राज ान के ऐसे े म फसल उगाती है जहाँ झील नह ह और ब त कम वषा होती है। ग मय के
दौरान, पानी क कम उपल ता फसल उ पादन को सी मत कर दे ती है।
गीता को जल का आ थक प से उपयोग करने के लए कौन-सी सचाई प त अपनानी चा हए?
(ए) प स टम (बी) लीवर स टम
(सी) चरखी णाली (डी) छड़काव णाली

54. कृ ष ग त व धय और गैर-कृ ष ग त व धय म अंतर करके उ ह अलग-अलग ल खए।


1. कृ ष ग त व धय का उदाहरण या है?
ए . डेयरी बी. प रवहन (सी कानदारी डी. फसल उगाना
सही उ र : डी. फसल उगाना
2. गैर कृ ष ग त व धय का उदाहरण या है ?
ए. डेयरी बी. ॅनपोट c .दोन a और b d. . इनम से कोई नह
सही उ र : सी. a और b दोन

55. पालमपुर म यु होने वाली इकाई का नाम बताइए?


े क इकाई पांतरण इकाई

1 वग फु ट (वग फु ट) 144 वग इंच (1 फ ट 12 इंच है)


1 वग सट मीटर 0.00107639 वग फु ट

1 वग इंच 0.0069444 वग फु ट

1 वग कलोमीटर (वग कमी) 247.1 एकड़

1 वग मीटर (वग मीटर) 10.76391042 वग फु ट

1 वग मील 640 एकड़ या 259 हे टे यर

1 वग याड (वग गज) 9 वग फु ट

1 एकड़ 4840 वग गज या 100.04 सट (भू म मापने के


लए मानक इकाई)

1 हे टे यर 10000 वग मीटर या 2.49 एकड़ लगभग


45. Correct Answer:Option (a)
46 ) Correct Answer:Option (b)
47.Correct Answer: Option (c)
48.Correct Answer: Option (c)
49.Correct Answer: Option (d)
50. Correct Answer: Option (c)
51.Correct Answer: Option (c)
52.Correct Answer: Option (a)
53.Correct answer : d. Growing crops.
54.Correct answer : c. both a and b
55.Correct answer : a. Hectare
ASSERTION AND REASON
In the following questions , two statements are given-one labeled
Assertion (A) and the other labeled
Reason (R). Select the correct answer to these questions from the codes (a), (b), (c)
and (d) as given below.
(a) Assertion is true and reason is correct explanation of assertion.
(b) Assertion is true but reason is false.
(c) Assertion is false but reason is true.
(d) Both are true but reason is not correct explanation of assertion.
1. ASSERTION Till the mid1960s, the seeds used in cultivation were traditional ones
with relatively low yields. Traditional seeds needed less irrigation. Farmers used cow-
dung and other natural manure as fertilizers.
REASON As a result, the same piece of land would now produce far larger quantities of
foodgrains than was possible earlier
2. ASSERTION Small plots of lands are cultivated by the medium and large farmers . On
the other hand, more than half the area of the village is covered by plots that are quite
large in size, cultivated by the small farmers
REASON Small farmers along with their families cultivate their own fields. Thus, they
provide the labour required for farming themselves
3. ASSERTION: The small farmers , who constitute about 80 percent of total farmers in
India, find it difficult to obtain capital.
REASON: The small sizes of their plots are not able to produce enough
A. Both A & R are true & R is the correct explanation of A

B. Both A & R are true but R is not the correct explanation of A

C. A is correct but R is wrong

D. A is wrong but R is correct

4. ASSERTION : Farmers of Punjab ,Haryana and western Uttar Pradesh were the
first to try out the modern farming in India.
REASON: Farmers of these states were easily trained to use modern farming
methods .
A. Both A & R are true & R is the correct explanation of A

B. Both A & R are true but R is not the correct explanation of A

C. A is correct but R is wrong

D. A is wrong but R is correct

5. ASSERTION: Palampur has fairly well developed system of electricity .


REASON : Most of the houses have electricity connections tube wells are run in
fields and also small business run through electricity .
A. Both A & R are true & R is the correct explanation of A

B. Both A & R are true but R is not the correct explanation of A

C. A is correct but R is wrong

D. A is wrong but R is correct

6.ASSERTION: Persion wheel was used to draw water from wells for irrigation.
REASON: Use of Persian wheel for irrigation is modern method of farming.
7. ASSERTION: Multiple cropping means to grow more than one crop on different piece
of land.
REASON : Well developed system of irrigation is required for multiple cropping.
8. ASSERTION : Many non farming activities are done in Palampur.
REASON: Dairy farming is a non farming activity.
A. Both A & R are true & R is the correct explanation of A

B. Both A & R are true but R is not the correct explanation of A

C. A is correct but R is wrong

D. A is wrong but R is correct

9. ASSERTION: Food grains output in India has increased because of larger area under
cultivation.
REASON: Green Revolution brought about in India in late 1960.
अ भकथन और कारण
न न ल खत म, दो कथन दए गए ह- एक को लेबल कया गया है
अ भकथन (ए) और सरे को लेबल कया गया
कारण (आर)। नीचे दए गए कू ट (a), (b), (c) और (d) से इन के सही उ र का चयन क जए।
(ए) अ भकथन सही है और कारण दावा का सही ीकरण है।
(बी) अ भकथन सही है ले कन कारण गलत है।
(सी) अ भकथन गलत है ले कन कारण सच है।
(डी) दोन स य ह ले कन कारण कथन क सही ा या नह है।
1. अ भकथन :1960 के दशक के म य तक, खेती म इ तेमाल होने वाले बीज अपे ाकृ त कम पैदावार वाले
पारंप रक थे। पारंप रक बीज को कम सचाई क आव यकता होती है। कसान खाद के प म गाय के गोबर और
अ य ाकृ तक खाद का इ तेमाल करते थे।
कारण: प रणाम के प म, भू म का एक ही टु कड़ा अब पहले क तुलना म कह अ धक मा ा म खा ा का
उ पादन करेगा
2. अ भकथन: मयम और बड़े कसान ारा भू म के छोटे भूखंड पर खेती क जाती है। सरी ओर, गाँव का आधे
से अ धक े उन भूखंड से आ ा दत है जो आकार म काफ बड़े ह, जनक खेती छोटे कसान ारा क जाती
है।
कारण :छोटे कसान अपने प रवार के साथ अपने खेत म खेती करते ह। इस कार, वे वयं खेती के लए
आव यक म दान करते ह
3. : अ भकथन: छोटे कसान, जो भारत म कु ल कसान का लगभग 80 तशत ह, के लए पूंजी ा त करना
मु कल है।
कारण: उनके भूखंड के छोटे आकार पया त उ पादन करने म स म नह ह
A. A और R दोन स य ह और R, A क सही ा या करता है
B. A और R दोन स य ह ले कन R, A क सही ा या नह करता है
सी. ए सही है ले कन आर गलत है
D. A गलत है ले कन R सही है
4.अ भकथन : पंजाब, ह रयाणा और प मी उ र दे श के कसान भारत म आधु नक खेती को आजमाने वाले
पहले थे।
कारण: इन रा य के कसान को आधु नक कृ ष व धय का उपयोग करने के लए आसानी से श त कया
गया था।
A. A और R दोन स य ह और R, A क सही ा या करता है
B. A और R दोन स य ह ले कन R, A क सही ा या नह करता है
सी. ए सही है ले कन आर गलत है
D. A गलत है ले कन R सही है
5. अ भकथन: पालमपुर म बजली क काफ वक सत णाली है।
कारण : अ धकांश घर म बजली के कने न ह, नलकू प खेत म चलाए जाते ह और बजली के मा यम से
छोटे वसाय भी चलते ह।
A. A और R दोन स य ह और R, A क सही ा या करता है
B. A और R दोन स य ह ले कन R, A क सही ा या नह करता है
सी. ए सही है ले कन आर गलत है
D. A गलत है ले कन R सही है

6. अ भकथन: सचाई के लए कु से पानी ख चने के लए पसन हील का उपयोग कया जाता था।
कारण: सचाई के लए फारसी च का उपयोग खेती क आधु नक प त है।
7. अ भकथन: ब फसली का अथ है भू म के व भ टु कड़ पर एक से अ धक फसल उगाना।
कारण : ब फसली के लए अ तरह से वक सत सचाई णाली क आव यकता होती है।
8. अ भकथन : पालमपुर म अनेक गैर कृ ष काय कये जाते ह।
कारण: डेयरी फा मग एक गैर-कृ ष ग त व ध है।
A. A और R दोन स य ह और R, A क सही ा या करता है
B. A और R दोन स य ह ले कन R, A क सही ा या नह करता है
सी. ए सही है ले कन आर गलत है
D. A गलत है ले कन R सही है

9. अ भकथन: खेती के तहत बड़े े के कारण भारत म खा ा उ पादन म वृ ई है।


कारण: 1960 के अंत म भारत म ह रत ां त लाई गई।
ASSERTION AND REASON
1.Ans: (d) Both are true but reason is not correct explanation of assertion

2. Ans :c) Assertion is false but reason is true


3. Ans: A
4. Ans: C
5. Ans: A
6. Ans: A is true but R is false .use of Persian wheel for irrigation is traditional method of
farming.
7.Ans: A is wrong but R is correct . Multiple cropping means to grow more than one crop
on a same piece of land .
8.Ans:Both A and R are true and R is correct explanation of A.
9.Ans: A is wrong but R is correct.Green revolution brought about in late 1960 which
increased food grains output in India.

अ भकथन और कारण
1. उ र: (डी) दोन स य ह ले कन कारण कथन क सही ा या नह है
2.उ र :सी) अ भकथन गलत है ले कन कारण स य है
3. उ र: ए
4.उ र: सी
5.उ र: ए
6.उ र: ए सच है ले कन आर झूठा है। सचाई के लए फारसी प हया का उपयोग खेती क पारंप रक व ध है।
7.उ र: ए गलत है ले कन आर सही है। ब फसली का अथ है एक ही भू म पर एक से अ धक फसल उगाना।
8.उ र: A और R दोन स य ह और R, A क सही ा या करता है।
9.उ र: ए गलत है ले कन आर सही है। ह रत ां त 1960 के अंत म ई जसने भारत म खा ा उ पादन म वृ
क।
ECONOMICS (PEOPLE AS RESOURCE)

Question 1.
Which of the following is the most labour absorbing sectors of the Indian economy?
(a) Primary sector
(b) Tertiary sector
(c) Secondary sector
(d) Private sector
1
न न ल खत म से कौन भारतीय अथ व ा का सबसे अ धक म अवशो षत करने वाला े है?
(ए) ाथ मक े
(बी) तृतीयक े
(सी) मा य मक े
(डी) नजी े

Question 2.
Which of the following is included in the primary sector?
(a) Manufacturing
(b) Tourism
(c) Forestry
(d) Communication
2
न न ल खत म से कौन ाथ मक े क म शा मल है?
(ए) व नमाण
(बी) पयटन
(सी) वा नक
(डी) संचार
Question 3.
Which of the following is included in the tertiary sector?
(a) Fishing
(b) Trade
(c) Mining
(d) Health
3
न न ल खत म से कौन तृतीयक े क म शा मल है?
(ए) म य पालन
(बी) ापार
(सी) खनन
(डी) वा य

Question 4.
Which of the following is included in the secondary sector?
(a) Poultry farming
(b) Manufacturing
(c) Transport
(d) Quarrying

4.
न न म से कौन तीयक े क म शा मल है?
(ए) मुग पालन
(बी) व नमाण
(सी) प रवहन
(डी) उ खनन

Question 5.
Investment in human capital can be made through………..
(a) Education
(b) Training
(c) Medical care
(d) All the three
5.
मानव पूंजी म नवेश ……………. के मा यम से कया जा सकता है!
(ए) श ा
(बी) श ण
(सी) च क सा दे खभाल
(डी) तीन

Question 6.
The literacy rate during 2010-11 was
(a) 64%
(b) 74%
(c) 70%
(d) 80%

6.
2010-11 के दौरान सा रता दर थी-
(ए) 64%
(बी) 74%
(सी) 70%
(डी) 80%

Question 7.
Which of the following sectors faces the problem of seasonal unemployment?
(a) Agriculture
(b) Service sector
(c) Trade
(d) IT sector
7.
न न ल खत म से कौन सा े मौसमी बेरोजगारी क सम या का सामना करता है?
(ए) कृ ष
(बी) सेवा े
(सी) ापार
(डी) आईट े

Question 8.
Infant mortality rate is the death of a child under…………. Years.
(a) One year
(b) Three year
(c) Two year
(d) Four year
8.
शशु मृ यु दर ………………… से कम उ के ब े क मृ यु है
(ए) एक साल
(बी) तीन वष
(सी) दो साल
(डी) चार साल

Question 9.
The workforce population includes people from 15 years to -------
(a) 59 years
(b) 62 years
(c) 60 years
(d) 65 years
9.
कायबल क आबाद म 15 साल से लेकर ------- तक के लोग शा मल ह
(ए) 59 साल
(बी) 62 साल
(सी) 60 साल
(डी) 65 साल

Question 10.
Which age group of children does the Sarva Siksha Abhiyan aim to promote education?
(a) 6 to 10 years
(b) 5 to 3 years
(c) 6 to 14 years
(d) 7 to 12 years
10.
सव श ा अ भयान का उ े य कस आयु वग के ब क श ा को बढ़ावा दे ना है?
(ए) 6 से 10 साल
(बी) 5 से 3 साल
(सी) 6 से 14 साल
(डी) 7 से 12 साल

Question 11.
The number of females per thousand males refers to :
(a) Sex Ratio
(b) Literacy Rate
(c) Infant Mortality Rate
(d) Birth Rate
11.
त हजार पु ष पर म हला क सं या का ता पय है:
(ए) लग अनुपात
(बी) सा रता दर
(सी) शशु मृ यु दर
(डी) ज म दर

Question 12.
Decrease in IMR (Infant Mortality Rate) of a country signifies:
(a) Increase in life expectancy
(b) Increase in GNP
(c) Economic development of a country
(d) Increase in number of colleges in a country
12.
कसी दे श के आईएमआर ( शशु मृ यु दर) म कमी दशाता है:
(ए) जीवन याशा म वृ
(बी) जीएनपी म वृ
(सी) कसी दे श का आ थक वकास
(डी) एक दे श म कॉलेज क सं या म वृ

Question 13.
What is the life expectancy in India as per the census of 2000?
(a) 72 years
(b) 53 years
(c) 64 years
(d) 80 years
13.
2000 क जनगणना के अनुसार भारत म जीवन याशा या है?
(ए) 72 साल
(बी) 53 साल
(सी) 64 साल
(डी) 80 साल

Question 14.
Which one of the following is considered important to create a ‘virtuous cycle’ by the
parents?
(a) To sand their children to the school
(b) To provide goods food to their children
(c) To join their children in corporate schools
(d) To take care of the health and education of their children
14.
माता- पता ारा 'पु य च ' बनाने के लए न न ल खत म से कसे मह वपूण माना जाता है?
(ए) अपने ब को कू ल भेजने के लए
(बी) अपने ब को सामान भोजन उपल कराने के लए
(सी) कॉप रेट कू ल म अपने ब को शा मल करने के लए
(डी) अपने ब के वा य और श ा क दे खभाल करने के लए
Question 15.
Which one from the following is the most labour absorbing activity?
(a) Agriculture
(b) Fisheries
(c) Poultry farming
(d) Mining
15.
न न ल खत म से कौन सबसे अ धक म अवशो षत करने वाला े है?
(ए) कृ ष
(बी) म य पालन
(सी) मुग पालन
(डी) खनन
Question 16.
What is the aim of Sarva Siksha Abhiyan?
(a) To provide elementary education to women
(b) To provide elementary education to the rural poor
(c) To provide elementary education to all children in the age group 6-14 years
(d) To provide elementary education to the urban poor
16.
सव श ा अ भयान का उ े य या है?
(ए) म हला को ारं भक श ा दान करने के लए
(बी) ामीण गरीब को ारं भक श ा दान करने के लए
(c) ६-१४ वष के आयु वग के सभी ब को ारं भक श ा दान करना
(डी) शहरी गरीब को ारं भक श ा दान करने के लए
Question 17.
What is the expended form of PHC
(a) Public Health Club
(b) Private Health Club
(c) Primary Health Centre
(d) None of these
17.
पीएचसी का पूण प या है?
(ए) प लक हे लब
(बी) नजी वा य लब
(सी) ाथ मक वा य क
(डी) इनम से कोई नह

Question 18.
If a person cannot find jobs during some months of the year, which type of
unemployment is this called?
(a) Structural unemployment
(b) Cyclical unemployment
(c) Seasonal unemployment
(d) None of these

18.
य द कसी को वष के कु छ महीन के दौरान नौकरी नह मल पाती है, तो इसे कस कार का बेरोजगारी
कहा जाता है?
(ए) संरचना मक बेरोजगारी
(बी) च य बेरोजगारी
(सी) मौसमी बेरोजगारी
(डी) इनम से कोई नह

Question 19.
The persons who are not working by their own willing is covered under
(a) seasonal unemployment
(b) disguised unemployment
(c) educated unemployment
(d) none of the above
19.
जो अपनी मज से काम नह कर रहे ह, वे इसके अंतगत आते ह:
(ए) मौसमी बेरोजगारी
(बी) बेरोजगारी
(सी) श त बेरोजगारी
(डी) उपरो म से कोई नह

Question 20.
Where is Seasonal unemployment found
(a) urban areas
(b) rural areas
(c) in remote areas
(d) both in rural and urban areas
20.
मौसमी बेरोजगारी कहाँ पाई जाती है
(ए) शहरी े
(बी) ामीण े
(सी) रदराज के इलाक म
(डी) ामीण और शहरी दोन े म

Question 21.
The quality of population depends on
(a) literacy rate
(b) health
(c) skill
(d) all the above
21.
जनसं या क गुणव ा नभर करती है
(ए) सा रता दर
(बी) वा य
(सी) कौशल
(डी) उपरो सभी
Question 22.
Which one from the following is the primary sector activities
(i) Forestry
(ii) Poultry farming
(iii) Animal husbandry
(iv) Manufacturing
(a) (i)
(b) (i), (ii), (iii)
(c) (ii), (iii), (iv)
(d) All the above
22.
न न ल खत म से कौन-सा एक ाथ मक े क ग त व धयाँ ह:-
(i) वा नक
(ii) मुग पालन
(iii) पशुपालन
(iv) व नमाण
(ए) (i)
(बी) (i), (ii), (iii)
(सी) (ii), (iii), (iv)
(डी) उपरो सभी

Question 23.
Which group of four states have the maximum number of medical colleges in India?
(a) Andhra Pradesh, Karnataka, Maharashtra and Tamil Nadu
(b) Bihar, Bengal, UP and Haryana
(c) Andhra Pradesh, Kerala, Karnataka and Tamil Nadu
(d) Andhra Pradesh, Karnataka, Kerala and Goa
23.
भारत म चार रा य के कस समूह म सबसे अ धक मे डकल कॉलेज ह?
(ए) आं दे श, कनाटक, महारा और त मलनाडु
(बी) बहार, बंगाल, यूपी और ह रयाणा
(सी) आं दे श, के रल, कनाटक और त मलनाडु
(डी) आं दे श, कनाटक, के रल और गोवा

Question 24.
The scheme for the establishment of residential schools to impart education to talented
children from rural areas is
(a) Kendriya Vidyalayas
(b) Navodaya Vidyalayas
(c) Sarvodaya Vidyalayas
(d) None of the aboves

24.
ामीण े के तभाशाली ब को श ा दान करने के लए आवासीय व ालय क ापना क योजना है
(ए) क य व ालय
(बी) नवोदय व ालय
(सी) सव दय व ालय
(डी) उपरो म से कोई नह

Question 25.
What is India’s position in scientifically and technically manpower in the world?
(a) first
(b) second
(c) third
(d) fourth
25.
व म वै ा नक और तकनीक प से जनश म भारत का ान या है?
(ए) पहला
(बी) सरा
(सी) तीसरा
(डी) चौथा

Question 26.
Which one from the following is include in Secondary sector includes
(a) trade
(b) marketing
(c) manufacturing
(d) education
26.
न न ल खत म से कौन तीयक े क म शा मल है इसम शा मल है!
(ए) ापार
(बी) वपणन
(सी) व नमाण
(डी) श ा
Question 27.
Increase in longevity of life is an indicator of
(a) good quality of life
(b) improvement in health sector
(c) better HDI (Human Development Index)
(d) all the above

27.
जीवन क लंबी उ म वृ कसका सूचक है?
(ए) जीवन क अ गुणव ा
(बी) वा य े म सुधार
(सी) बेहतर एचडीआई (मानव वकास सूचकांक)
(डी) उपरो सभी
Question 28.
Which of the following is a significant step towards providing basic education to the
children in the age group of 6-14 years?
(a) Sarva Siksha Abhiyan
(b) Adult Education Programme
(c) Mid-day meal
(d) None

28.
न न ल खत म से कौन सा 6-14 वष के आयु वग के ब को बु नयाद श ा दान करने क दशा म एक
मह वपूण कदम है?
(ए) सव श ा अ भयान
(बी) वय क श ा काय म
(सी) म या भोजन
(डी) कोई नह

Question 29.
Why is literacy rate is low in the females?
(a) lack of equal education opportunities
(b) lack of transport facilities
(c) lack of infrastructure
(d) lack of income
29.
म हला म सा रता दर कम य है?
(ए) समान श ा के अवसर क कमी
(बी) प रवहन सु वधा क कमी
(सी) बु नयाद ढांचे क कमी
(डी) आय क कमी

Question 30.
Services of housewives are included in
(a) national income
(b) domestic income
(c) household income
(d) none of the above

30.
गृ ह णय क सेवाएं शा मल ह
(ए) रा ीय आय
(बी) घरेलू आय
(सी) घरेलू आय
(डी) उपरो म से कोई नह

Question 31.Which of the following is an economic activity?


A teacher teaching his son
b Recitation among friends
c Teacher teaching in the classroom
d All of the above
31 न न ल खत म से कौन एक आ थक ग त व ध है?
अ.एक श क अपने बेटे को पढ़ा रहा है
ब.दो त के बीच स वर पाठ
स.क ा म पढ़ाते श क
द. उपरो सभी
Question 32.
When there is investment made in the form of education, training and medical care, the
population becomes :
(a) Working capital
(b) Human capital
(c) Fixed capital
(d) Capital growth
32.
जब श ा, श ण और च क सा दे खभाल के प म नवेश कया जाता है, तो जनसं या हो जाती है:
(ए) कायशील पूंजी
(बी) मानव पूंजी
(सी) न त पूंजी
(डी) पूंजी वकास

Question33 Why countries like Japan are rich or developed despite the unavailability of
raw material?
a Because they have invested in human resources
b Because they have always been ruled by capitalists
c Because they have lots of natural resources
d All the above
33 क े माल क अनुपल ता के बावजूद जापान जैसे दे श समृ या वक सत य ह?
अ. य क उ ह ने मानव संसाधन म नवेश कया है
ब. य क उन पर हमेशा पूंजीप तय का शासन रहा है
स. य क उनके पास ब त सारे ाकृ तक संसाधन ह
द. उपरो सभी
Question 34 Urban areas have mostly …..
a Seasonal unemployment
b Disguised unemployment
c Educated unemployment
d All the above

34 शहरी े म यादातर… ..
अ.मौसमीबेरोजगारी
ब. बेरोजगारी
स. श तबेरोजगारी
द.उपरो सभी

Question 35 what is the full form of GDP?


a Global Domestic product
b Gross Domestic price
c Global Domestic price
d Gross Domestic product

35 जीडीपी का पूण प या है?


अ.वै कघरेलूउ पाद
ब.सकलघरेलूमू य
स.वै कघरेलूमू य
द.सकलघरेलूउ पाद
Question 36 What has diluted the huge expansion of the schools?
a Poor quality of schooling
b High dropout rates
c Only a
d Both a & b

36 कू ल के वशाल व तार को कस बात ने कमजोर कया है?


अ. कू ली श ा क खराब गुणव ा
ब.उ छोड़ने वाल क दर
स.के वल ए
द.दोन ए और बी
Question 37.
The New Education Policy in India was announced in the year ____.
(a) 2019
(b) 2021
(c) 2020
(d) 2018

37
भारत म नई श ा नी त क घोषणा वष ____ म क गई थी।
(ए) 2019
(बी) 2021
(सी) 2020
(डी) 2018

Assertion / Reasoning

अ भकथन / कारण
Question38

Assertion(A) The expenditure on education as a percentage of GDP has decline to 2.7%


in 2017-18.
Reason (R) In 2011 literacy rate varied from 94 % in Kerala to 62% in Bihar.
Codes
(a) Both A and R are true, and R is the correct explanation of A
(b) Both A and R are true,but R is not the correct explanation of A
(c) A is true, but R is false
(d) A is false, but R is true

38
दावा (ए) 2017-18 मसकलघरेलूउ पादके तशतके पम श ापर य 2.7% तक गरगयाहै।
कारण (R) 2011 मसा रतादरके रलम 94% से बहारम 62% तक भ थी।
कोड् स
(ए) ए और आर दोन स य ह, और आर ए क सही ा या है
(बी) ए और आर दोन स य ह, ले कन आर ए क सही ा या नह है
(सी) ए सच है, ले कन आर झूठा है
(डी) ए झूठा है, ले कन आर सच है

Question39
Assertion (A) Unemployment leads to wastage of manpower resources.
Reason (R) Increase in unemployment is an indicator of depressed economy.
Codes
(a) Both A and R are true and R is the correct explanation of A
(b) Both A and R are true but R is not the correct explanation of A
(c) A is true, but R is false
(d) A is false, but R is true

39
दावा (ए) बेरोजगारी जनश संसाधन क बबाद क ओर ले जाती है।
कारण (R) बेरोजगारी म वृ दयनीय अथ व ा का सूचक है।
कोड् स
(ए) ए और आर दोन स य ह और आर ए क सही ा या है
(बी) ए और आर दोन स य ह ले कन आर ए क सही ा या नह है
(सी) ए सच है, ले कन आर झूठा है
(डी) ए झूठा है, ले कन आर सच है
Question 40
Assertion (A) Market activities are performed for pay or profit.
Reason (R) Non market activities are performed for self consumption.
(a) Both A and R are true and R is the correct explanation of A
(b)Both A and R are true and R is not the correct explanation of A
(c)A is true, but R is false
(d)A is false, but R is true

40
अ भकथन (ए) बाजार क ग त व धयाँ वेतन या लाभ के लए क जाती ह।
कारण (आर) गैर-बाजार ग त व धय को वयं उपभोग के लए कया जाता है।
(ए) ए और आर दोन स य ह और आर ए क सही ा या है
(बी) ए और आर दोन स य ह और आर ए क सही ा या नह है
(सी) ए सच है, ले कन आर झूठा है
(डी) ए झूठा है, ले कन आर सच है

Question 41
Assertion (A) Educated population is an important input for growth of country.
Reason (R) Education opens new horizons, provide new aspiration and develop value for
life.

(a)Both A and R are true and R is the correct explanation of A


(b)Both A and R are true and R is not the correct explanation of A
(c)A is true, but R is false
(d)A is false, but R is true
41
दावा (ए) दे श के वकास के लए श त आबाद एक मह वपूण इनपुट है।
कारण (आर) श ा नए तज खोलती है, नई आकां ा दान करती है और जीवन के लए मू य वक सत करती
है।
(ए) ए और आर दोन स य ह और आर ए क सही ा या है
(बी) ए और आर दोन स य ह और आर ए क सही ा या नह है
(सी) ए सच है, ले कन आर झूठा है
(डी) ए झूठा है, ले कन आर सच है

Question 42
Assertion (A) A healthy person become an asset to the country.
Reason(R) Health increases a person's potential and ability to work.
(a) Both A and R are true and R is the correct explanation of A
(b) Both A and R are true and R is not the correct explanation of A
(c) A is true, but R is false
(d) A is false, but R is true

42
दावा (ए) एक व दे श के लए एक संप बन जाता है।
कारण (आर) वा य क मता और काम करने क मता को बढ़ाता है।
(ए) ए और आर दोन स य ह और आर ए क सही ा या है
(बी) ए और आर दोन स य ह और आर ए क सही ा या नह है
(सी) ए सच है, ले कन आर झूठा है
(डी) ए झूठा है, ले कन आर सच है
Question 43
Assertion (A) Unemployment affect the overall growth of the economy.
Reason (R) IT leads to wastage of manpower resource and increases economic
overload.
(a) Both A and R are true and R is the correct explanation of A
(b) Both A and R are true and R is not the correct explanation of A
(c) A is true, but R is false
(d)A is false, but R is true

43
दावा (ए) बेरोजगारी अथ व ा के सम वकास को भा वत करती है।
कारण (आर) आईट जनश संसाधन क बबाद क ओर जाता है और आ थक अ धभार बढ़ाता है
(ए) ए और आर दोन स य ह और आर ए क सही ा या है
(बी) ए और आर दोन स य ह और आर ए क सही ा या नह है
(सी) ए सच है, ले कन आर झूठा है
(डी) ए झूठा है, ले कन आर सच है

Question 44. Read the given passage and answer the question that follows:-

Investment in human capital (through education, training , medical care) yields a return
just like investment in physical capital. This can be seen directly in the form of higher
incomes earned because of higher productivity of the more educated or the better
trained persons as well as the higher productivity of healthier people.

(i)………….is a way of referring to a country's working people in terms of their existing


productive skills and abilities.

(a) Working population


(b)Independent population
(c)People as a resource
(d)Human capital formation

44. दए गए ग ांश को प ढ़ए और नीचे दए गए के उ र द जए:-


मानव पूंजी म नवेश ( श ा, श ण, च क सा दे खभाल के मा यम से) भौ तक पूंजी म नवेश क तरह ही
तफल दे ता है। इसे सीधे तौर पर अ धक श त या बेहतर श त य क उ उ पादकता के साथ-साथ
व लोग क उ उ पादकता के कारण अ जत उ आय के प म दे खा जा सकता है।
(i).………….एक दे श के कामकाजी लोग को उनके मौजूदा उ पादक कौशल और मता के संदभ म संद भत
करने का एक तर(ए) कामकाजी आबाद
(बी) वतं जनसं या
(सी) संसाधन के प म लोग
(डी) मानव पूंजी नमाण

(ii) When do we call human capital the human capital formation?


(a) By making population more healthy and educated.
(b) By forming population in a group.
(c) By organising a training for the population.
(d) by giving them more resources like property employment etc.

(ii). हम मानव पूंजी को मानव पूंजी नमाण कब कहते ह?


(ए) जनसं या को अ धक व और श त बनाकर।
(बी) एक समूह म जनसं या बनाकर।
(सी) आबाद के लए एक श ण आयो जत करके ।
(डी) उ ह संप रोजगार आ द जैसे अ धक संसाधन दे कर।
(iii) Choose the positive aspect of the population.
(a) Providing population with health facility.
(b) Arranging food facility for population
(c) Higher productivity with higher population.
(d) Giving education to the children of the country.
(iii) जनसं या का सकारा मक पहलू चुन।
(ए) वा य सु वधा के साथ जनसं या दान करना।
(बी) जनसं या के लए भोजन सु वधा क व ा करना
(सी) उ जनसं या के साथ उ उ पादकता।
(द) दे श के ब को श ा दे ना।
(iv)Which is an example of the greater knowledge in the form of improved production
technology?
(a) Green Revolution
(b) Human working capital
(c) Physical capital formation
(d) National income

(iv) उ त उ पादन तकनीक के प म अ धक ान का कौन सा उदाहरण है?


(ए) ह रत ां त
(बी) मानव कायशील पूंजी
(सी) भौ तक पूंजी नमाण
(डी) रा ीय आय

Question 45. Read the given passage and answer the question that follows:-
The various activities have been classified into three main sectors i.e., primary,
secondary and tertiary. Primary sector includes agriculture, forestry, animal husbandry,
fishing, poultry farming, mining and quarrying.
(i) Which sector includes trade transport, communication, tourism etc activities?
(a)Primary sector
(b) Secondary sector
(c) Tertiary sector
(d) Industrial sector

45. दए गए गांश को प ढ़ए और नीचे दए गए के उ र द जए:-


व भ ग त व धय को तीन मु य े अथात ाथ मक, मा य मक और तृतीयक म वग कृ त कया गया है।
ाथ मक े म कृ ष, वा नक , पशुपालन, म य पालन, मुग पालन, खनन और उ खनन शा मल ह।
(i) कस े म ापार प रवहन, संचार, पयटन आ द ग त व धयाँ शा मल ह?
(अ) ाथ मक े
(ब) मा य मक े
(स) तृतीयक े
(द) औ ो गक े
(ii)………….. is included in the secondary sector.
(a) Fishing
(b) Manufacturing
(c) Transportation
(d) Agriculture

(ii).………….. तीयक े क म शा मल है।


(ए) म य पालन
(बी) व नमाण
(सी) प रवहन
(डी) कृ ष

Question 46. Read the given passage and answer the question that follows:-
The basic cause of poverty is unemployment. People do not have enough money to
support their family. Inability of educated people, who are willing to work to find gainful
employment implies a great social waste. Unemployment tends to increase the
economic overload. The dependence of the unemployed on the working population
increases. The quality of life of an individual as well as of the society is adversely
affected. When a family has to live on a bare subsistence level, there is a general
decline in its health status and rising withdrawal from the school system.
46. दए गए ग ांश को प ढ़ए और नीचे दए गए का उ र द जए:-
गरीबी का मूल कारण बेरोजगारी है। लोग के पास अपने प रवार का भरण-पोषण करने के लए पया त धन नह है।
श त लोग क अ मता, जो लाभकारी रोजगार खोजने के लए काम करने को तैयार ह, का अथ है एक महान
सामा जक बबाद । बेरोजगारी आ थक अ धभार को बढ़ाती है। बेरोजगार क कामकाजी आबाद पर नभरता बढ़
जाती है। एक के साथ-साथ समाज के जीवन क गुणव ा पर तकू ल भाव पड़ता है। जब एक प रवार को
के वल नवाह तर पर रहना पड़ता है, तो उसके वा य क त म सामा य गरावट आती है और कू ल णाली
से बढ़ती वापसी होती है।
(i) Work force refers to that part of :
(a) Labour force which is employed
(b) Population which is unemployed
(c) Population which is forced to work
(d) Labour force which is unemployed

(i) काय बल का ता पय उस भाग से है:


(ए) म बल जो कायरत है
(बी) जनसं या जो बेरोजगार है
(सी) जनसं या जो काम करने के लए मजबूर है
(डी) म बल जो बेरोजगार है

(ii) Which of the following occupations would most likely be subject to seasonal
unemployment?
(a) Automobile mechanic.
(b) Appliance salesperson.
(c) Television repair-person.
(d) Farm worker.

(ii) न न ल खत म से कौन सा वसाय मौसमी बेरोजगारी के अधीन होगा?


(ए) ऑटोमोबाइल मैके नक।
(बी) उपकरण व े ता।
(सी) टे ली वजन मर मत- ।
(डी) खेत कायकता।

(iii) What are the three consequences of unemployment?


(a) High dropout rates, decrease in Dependent Population, Poor Quality of Life
(b) Poor Quality of Life, Poor Health, High dropout rates
(c) Increase in Dependent Population, Poor quality of schooling, Poor Health
(d) Poverty, Increase in Dependent Population, Poor Quality of Life

(iii) बेरोजगारी के तीन प रणाम या ह?


(ए) उ ॉपआउट दर, आ त जनसं या म कमी, जीवन क खराब गुणव ा
(बी) जीवन क खराब गुणव ा, खराब वा य, उ छोड़ने क दर
(सी) आ त जनसं या म वृ , कू ली श ा क खराब गुणव ा, खराब वा य
(डी) गरीबी, आ त जनसं या म वृ , जीवन क खराब गुणव ा

Question 47.Consider the following statements:

1. Disguised unemployment is most commonly found unemployment in farm sector in


India
2. Small size of landholdings and joint family system are some of the factors behind
disguised unemployment
Which among the above is / are correct statements?
(i) Only 1
(ii) Only 2
(iii) Both 1 and 2
(iv)Neither 1 nor 2

48. न न ल खत कथन पर वचार कर:


1. बेरोजगारी भारत म कृ ष े म सबसे अ धक पाई जाने वाली बेरोजगारी है
2. छोटे आकार क जोत और संयु प रवार व ा बेरोजगारी के पीछे कु छ कारक ह
उपरो म से कौन-सा/से सही कथन है/ह?
(i) के वल 1
(ii) के वल 2
(iii) 1 और 2 दोन
(iv) न तो 1 और न ही 2

Question 49. Choose the non-market activities


(i) Vilas sells fish in the village market (ii) Vilas cooks food for his family (iii) Sakal
works in a private firm (iv) Sakal looks after his younger brother and sister
(a) (i) & (ii)
(b) (iii) & (iv)
(c) (i) & (iii)
(d) (ii) & (iv)

49. गैर-बाजार ग त व धय का चयन कर


(i) वलास गांव के बाजार म मछली बेचता है (ii) वलास अपने प रवार के लए खाना बनाता है (iii) सकल एक
नजी फम म काम करता है (iv) सकल अपने छोटे भाई और बहन क दे खभाल करता है
(ए) (i) और (ii)
(बी) (iii) और (iv)
(सी) (i) और (iii)
(डी) (ii) और (iv)

Question 50. Bribe taking is an -------------- activity.


(a) Market activity
(b) Economic activity
(c) Non market activity
(d) Non economic activity

50. र त लेना एक -------------- ग त व ध है।


(ए) बाजार ग त व ध
(बी) आ थक ग त व ध
(सी) गैर बाजार ग त व ध
(डी) गैर आ थक ग त व ध

Answer key
1. Primary sector
2. Forestry
3. Health
4. Manufacturing
5. All of three
6. 74%
7. Agriculture
8. 1 year
9. 59 years
10. 6 to 14 years
11. Sex ratio
12. Increase in life expectancy
13. 64 years
14. To take care of the health and education of their children
15. Agriculture
16. To provide elementary education to all children in the the age group 6 to 14 years
17. Primary health centre
18. More than required
19. Seasonal unemployment
20. None of the above
21. Rural area
22. All the above
23. (b) i,ii,iii
24. Andhra Pradesh, Karnataka, Maharashtra and Tamil Nadu
25. Navodaya vidyalayas
26. Third
27. Manufacturing
28. Good quality of life
29. Sarva shiksha abhiyan
30. Lake of equal education opportunity
31. None of the above
32. Teacher teaching in the classroom
33. Human resource
34. Because they have invested in human resource.
35. Educated unemployment
36. Gross domestic product
37. Both a & b
38. 2020
39. Both A and R are true, but R is not the correct explanation of A
40. Both A and R are true but R is not the correct explanation of A
41. Both A and R are true but R does not explain A
42. Both A and R are true and R is correct explanation of A
43. Both A and R are true and R is correct explanation of A
44. A is true, but R is false
45. (i) People as a resource
(ii) By making population more healthy and educated
(III) Higher productivity with higher population
(iv) Green Revolution
46. (i) Tertiary sector
(ii) manufacturing
47. (i) Labour force which is employed
(ii) Farm worker
(iii) Poverty, Increase in Dependent Population, Poor Quality of Life
48. Both 1 and 2
49. (i) & (iii)
50. Non economic activity

You might also like